11.07.2015 Views

Revista (format .pdf, 1.2 MB) - Recreaţii Matematice

Revista (format .pdf, 1.2 MB) - Recreaţii Matematice

Revista (format .pdf, 1.2 MB) - Recreaţii Matematice

SHOW MORE
SHOW LESS

Create successful ePaper yourself

Turn your PDF publications into a flip-book with our unique Google optimized e-Paper software.

Anul X, Nr. 2Iulie – Decembrie 2008RECREAŢ IIMATEMATICEREVISTĂ DE MATEMATICĂ PENTRU ELEVI Ş I PROFESORI125 de ani de la apariţiarevistei “Recreaţii Ştiinţifice”(1883 – 1888)e iπ =−1Asociaţ ia “Recreaţ ii <strong>Matematice</strong>”IAŞ I - 2008


Osută de ani de la naşterea lui Gheorghe GheorghievGheorghe Gheorghiev a fost elev aliluştrilor savanţi A. Myller şi O. Mayer şi a devenit,pentru o bună perioadă de timp, liderulşcolii de geometrie de la Universitatea din Iaşi. Afost un reprezentant al unei generaţii de dascăliieşeni care au vegheat la destinele matematicilorromâneşti, într-o perioadă demarischimbăripolitice. În acelaşi timp, a căutat săreialegăturileîntrerupte cu matematicile din lumea întreagă. Afost un adevărat constructor de şcoală ştiinţificăla Facultatea de matematică dinIaşi, dovedinduseun merituos continuator al eforturilor şi străduinţelorprofesorului său A. Myller.S-a născut la 27 iunie 1907 la Bolgrad, judeţulIsmail. A urmat liceul din Bolgrad în anii 1916-1925, dupăcareasusţinut examenul de bacalureatla Ismail în iunie 1925. Imediat a devenit studentla Facultatea de ştiinţe din Iaşi, secţia matematică, unde a urmat cursuri predatede profesorii A. Myller, Vera Myller Lebedeff, O. Mayer, Simeon Sanielevici, PetreCulianu, Constantin Popovici, Mendel Haimovici, Gheorghe Vrănceanu ş.a. şi l-aavut asistent pe Ilie Popa, cucarearămas prieten pentru toată viaţa. În timpulultimului an de studiu şi după examenul de licenţă, în martie 1929, a devenit asistentşi s-a împrietenit cu mai mulţi tineri matematicieni: I. Schoenberg, P. Cazanachi,D. Mangeron, E. Stihi, şi cu studentul A. Climescu. Tot în 1929 a urmat cursuri despecializare la Universitatea din Hamburg, cu cunoscutul profesor W. Blaschke.În 1929 s-a declanşat criza economica pe plan mondial, care a afectat şi ţaranoastră. În perioada 1929-1938, Gh. Gheorghiev a fost profesor de matematici laliceele din Aiud, Ismail, Chilia Noua, Cetatea Alba şi Iaşi. In perioada 1938-1946 afost înscris la doctorat la Universitatea din Iaşi. În aceeaşi perioadă a fost concentratşi mobilizat timp de peste 5 ani. A participat la război ca ofiţer de geniu şi, timp decirca un an şi jumătate, a fost prizonier. A existat chiar pericolul să fieîmpuşcat înmomentul în care a fost luat prizonier, pentru că eraofiţer.În 1946 a susţinut teza de doctorat cu titlul Suprafeţe ale căror familii remarcabilede curbe sunt asemenea (comisia de doctorat, fiind compusădinA. Myller, O. Mayer,V. Myller, I. Popa).Afuncţionat la Universitatea din Iaşi ca lector (1946-1948), conferenţiar onorificde matematici generale (1946-1948), profesor suplinitor de geometrie analitică (1947-1954), profesor titular de geometrie (1955-1975).Între anii 1953-1968 a fost decan al Facultăţii de matematică dinIaşi iar, între1960-1975, a fost şeful catedrei de geometrie. În aceste posturi şi poziţii de conducere,Gh. Gheorghiev aavutoinfluenţă hotărâtoare asupra dezvoltării facultăţii în cepriveşte stabilirea unor legături cu direcţii de cercetare ştiinţifică existente atât înşcolile de matematică din occident cât şi cu cele existente în Uniunea Sovietică.95


A colaborat la elaborarea unui tratat de geometrie intitulat simplu Curs de geometrieanalitică (1951); recunoaşterea meritelor acestui tratat s-a realizat prin acordareaPremiului de Stat. A mai colaborat la realizarea unei carţi Geometrie analiticăşi diferenţială, apărută îndouă volume în 1968 şi 1969 la Editura Didactică şi Pedagogică.În această carte încep să sereflecte schimbările de concepţie şi de tehnicădeinvestigareîncepriveşte studiul geometriei diferenţiale. Ideile au fost finalizateîn alte două carţi, elaborate în colaborare, Varietăţi diferenţiabile finit şi infinit dimensionale,apărută îndouă volume în 1976 şi 1979 la Editura Academiei Româneşi Geometrie diferenţială apărută laEdituraDidactică şi Pedagogică.A scris circa 180 de lucrări ştiinţifice în domeniul geometriei diferenţiale şi numeroasearticole de popularizare a matematicii. A conferenţiat la mai multe universitaţide prestigiu din lume. A primit premiul "Gh. Ţiţeica" al Academiei Române,a primit titlul de profesor emerit, mai multe diplome şi medalii ale unor asociaţiiprofesionale şi numeroase ordine şi medalii, din partea statului român.După pensionare, în 1975, a rămas profesor consultant la Facultatea de matematică(cu unele întreruperi) până la deces, în 28 iunie 1999.A abordat teme de cercetare ştiinţifică din geometria diferenţială euclidiană(reţelepe suprafeţe, câmpuri de vectori pe suprafeţe) geometria diferenţială afină şi proiectivă(câmpuri de conuri, configuraţii Myller), geometria diferenţială avarietaţilormodelate de spaţii Banach, teoria grupurilor Lie, teoria G-structurilor şi generalizăriale acesteia. Circa 30 de tineri studioşi şi-au elaborat tezele de doctorat sub conducereaştiinţifică a profesorului Gheorghiev. A colaborat, în elaborea cărţilor, cu câţivacolegi apropiaţi: R. Miron, D. Papuc, V. Oproiu.A fost profesor vizitator şi a conferenţiat la numeroase universitaţi din Europa:Jena, Padova, Sofia, Londra, Moscova, Debrecen etc.A predat cursuri de geometrie analitică, diferenţială, teoria grupurilor Lie etc. ladiverse nivele, la Facultatea de matematică aUniversităţii din Iaşi. A mai predatşi cursuri de matematici generale pentru studenţii de la facultăţile Institutului Politehnicdin Iaşi. Lecţiilesaleeraurenumiteprindensitateainformaţiilor transmise şipentru solicitarea intelectuală extremălacareerausupuşi studenţii ce le urmăreau.În perioada cât a fost decan, în facultate s-a petrecut o extraordinară schimbareaargumentelor predate la cursuri şi a celor abordate în cercetare, realizându-se un saltde la aspectele tradiţionale existente în matematicile din perioada interbelică, cătrematematicile moderne, cercetate şi predate în stilul structuralist, promovat de cătregrupul Bourbaki. În ultimii ani ai vieţii era preocupat de istoria matematicii şi deprobleme matematice ale filosofiei fizicii.Meritele la catedră, în cercetarea ştiinţifică, de om care, pe tot parcursul vieţiisale, a practicat cultul muncii şi al cinstei au făcut din profesorul Gh. Gheorghievo personalitate în domeniul matematicilor, respectată atât în facultate şi universitatecât şi pe plan naţional şi internaţional.Activitatea sa în slujba şcolii româneşti şi în servirea matematicii a fost răsplatităprin mai multe distincţii, titlul de Profesor Emerit (1964), Premiul de Stat (1951),premiul "Gh.Ţiţeica" al Academiei Române (1981), Ordinul Muncii (1960), OrdinulMeritul Ştiinţific (1960), Medalii ale Muncii etc.96Prof. dr. Vasile OPROIU


siderăm că pentru cititorii prezentei reviste sunt mai accesibile, mai interesante şi cuun folos mai mare alte aspecte ale activităţii variate pe care a desfăşurat-o Ilie Popa.Avândunspiritpătrunzător, o gândire profundă şiomareputeredesinteză şibeneficiind de o bogatăcultură generală, Ilie Popa a abordat teme diverse din istoriamatematicii româneşti aducând clarificări într-un material faptic din trecut, care eradestinat cu precădere uitării. A scos la lumină meritele unor precursori ai matematiciiromâneşti ca Dimitrie Asachi — fiul cel mare al lui Gh. Asachi —, primul românce publică înstrăinătate (München, 1841) o lucrare de matematici superioare sauN. Şt. Botez, caredăîn1872oformulă pentru o bucată aserieiarmonice,ceaatrasatenţia matematicianului belgian E.-Ch. Catalan. Un alt studiu este destinat luiAmfilochie Hotiniul, caretipăreşte în Iaşi, la 1795, prima aritmetica românească Elementiaritmetice arătate fireşti, ocompilaţie dupăcâtevacărţi italieneşti. Întreprindeo muncădificilă de restaurare, atunci când reconstituie manuscrisul de geometrie pierdutal lui Gh. Lazăr. A pus în evidenţă roluljucatdesoţii Alexandru şi Vera Myllerîn crearea şcolii matematice ieşene, cu numele generic de Seminarul matematic. Cuprilejul centenarului Universitătii din Iaşi (1860—1960), publică, în al doilea tomconsacrat evenimentului, studiul Dezvoltarea matematicii — o amplă şi documentatăcontribuţie privind realizările şcolii matematice ieşene pe parcursul unui secol.Să mai spunem cititorilor că Ilie Popa a publicat în 1955 studiul Recreaţii Ştiinţifice— precursoare a Gazetei matematice în care evidenţiază rolul acestei reviste —prima de acest fel din ţară, străbună aactualeiRecreaţii <strong>Matematice</strong> — în dezvoltareaînvăţământului şi netezirea drumului ce duce la cercetări originale în matematică.Ca profesor, Ilie Popa a fost un maestru al artei comunicării. Lecţiile sale erauoficiate după unritualneabătut. Începeau când profesorul punea ceasul de buzunarpe masă şi se terminau când acesta era repus la locul lui. Era omisă pauzadintreore, dar timpul acordat prelegerii era respectat cu stricteţe.Cuuntimbruplăcut, ovorbire blândă, moldovenească, cu un ritm al expunerii care antrena dar nu obosea,profesorul reuşea imediat să captiveze auditoriul. Ideile şi argumentele erau expuseîntr-o înlănţuire uşor de urmărit. Erau scoase în evidenţă semnificaţiile conceptelorşi rezultatelor. Faţa magistrului se acoperea de o lumină discretă, cea a bucurieide a dărui. Câte un zâmbet involuntar, reţinut cu grijă, marca un moment cheie allecţiei sau însoţea un rezultat de o rară frumuseţe. Calmul, echilibrul, tactul erauîntotdeauna prezente. George Şt. Andonie spune inspirat: îşi alege detaliile cu grijăşi obiectivitate şi lasă impresiacă-şi sună în prealabil cuvintele, spre a-şi da seamade rezonanţa lor (Istoria matematicii în România, vol. III).A avut numeroase funcţii de conducere în diverse structuri ale învăţământuluisuperior: prorector al Universitaţii din Iaşi, director al Seminarului Matematic"Al. Myller", director general în Ministerul Învăţământului, rector al InstitutuluiPedagogic de 3 ani din Iaşi ş.a. Cu devotament şi competenţă, a folosit întreaga sacapacitate de muncă în scopul ridicării învăţământului românesc.La împlinirea unui secol de la naşterea sa, ne gândim cu profundă recunoştinţăla cel ce a fost omul şi profesorul Ilie Popa şi cu toată admiraţia la opera sa degeometru, de istoriograf al matematicii şi învăţământului matematic şi de dascăl alînvăţământului din ţara noastră.98Prof. dr. Temistocle BÎRSAN


pământ primele capitole elevate de istorie a ştiinţelor din ţară şi cele întâi lecţiipentru un învăţământ modern în domeniul ştiinţelor exacte. Peste veac s-a văzutînrâurirea covârşitoare a ideilor vehiculate în cuprinsul acestei reviste, în Şcoala detoate gradele şi în cercetare, conducând la integrarea noastră în rândul ţărilor careaveau deja tradiţii seculare.Datorită conţinutului preponderent matematic se poate afirma cu deplin temeică revista a deschis prima pagină a matematicilor româneşti. Aşa cum s-a remarcatmai târziu, dacă publicaţia s-ar fi numit Recreaţii matematice, eaarficonstituitprima publicaţie din lume în domeniu, care se adresează tineretului. Este adevăratcă, la şapte ani de la dispariţia "Recreaţiilor ştiinţifice", în 1895 a fost înfiinţatăGazeta Matematică cu adresă specială pentru tineretul român. Dar această revistăeste considerată a doua din lume ca profil şi destinaţie.Evident, apariţia "Gazetei", aşa cum sublinia Gheorghe Ţiţeica a fost impulsionatăde"Recreaţiile ştiinţifice".Acum, la 125 de ani de la înfiinţare a celebrei reviste se cuvine să exprimămomagii profunde memoriei fondatorilor: N. Culianu, C. Climescu, I. Melic de laUniversitatea "Al. I. Cuza", G. I. Lucescu, V. Paladi, G. I. Roşiu, I. D. Rallet,G. Zarifopol, I. V. Praja şi I. M. Dospinescu din învăţământul preuniversitarieşean. Prin competenţă, pasiune şi sacrificii personale făcute cu generozitate ei aureuşit să trezească interesul pentru ştiinţă în general şi să stimuleze gustul pentrumatematici în special.Sunt emoţionante cuvintele scrise într-un editorial al revistei: Credem că noiamtras cea întăi brazdă care conduce cătră lucrări originale. Brazda-i mică şi îngustă,dar există!Personalitatea fondatorilor este bine cunoscută. Ei sunt prezentaţi de George Şt.Andonie în volumul I din Istoria Matematicii în România. Majoritatea lor suntoameni de ştiinţă cu studii înalte făcute în Franţa, Italia, Olanda şi Germania. Ungând de recunoştinţă colaboratorilor, nu mai puţin celebri: M. Tzony, V. Costin,P. Tanco, C. Gogu şi rezolvitorilor pasionaţi, elevi pe vremea aceea, E. Pangratişi D. Pompeiu.Nu trebuie să-i uităm pe oamenii de ştiinţăcareaususţinut peste timp importanţarevistei şi impactul ei în cultura românească. Cităm doar câţiva dintre ei: AlexandruMyller, Octav Mayer, Ilie Popa, Gheorghe Gheorghiev, Gheorghe Bantaş, GheorgheŢiţeica, G. Şt. Andonie, N. N. Mihăileanu etc.Condiţiile istorice în care a apărut în 1883 revista "Recreaţii ştiinţifice" nu eraudintre cele mai favorabile. Unirea Principatelor abia se înfăptuise, Regatul Românieiera abia întemeiat, Războiul de Independenţă din 1877 lăsase urme adânci în conştiinţaromânilor, alfabetul chirilic fusese înlocuit cu cel latin, limba română literarăabia îşi definitivase procesul de unificare, românii îşi afirmau în mod decisiv aspiraţiaspre o societate modernă. În atari condiţii, deşi apăruseră cu23deaniînainteuniversităţile din Iaşi şi Bucureşti, şcoala de toate gradele trebuia profund reclădită.Era nevoie imperioasă de regândit programarea curriculară, de pregătit personalul100


didactic, de scris manuale bune în limba româna, de construit şcoli etc.În atari condiţii spirituale, materiale şi sociale dure, a pune bazele unei revistede cultură ştiinţifică era un act de curaj, de patriotism. El constituia o importantărealizare destinată poporului nostru. Soliditatea acestui edificiu este dată de calitateaştiinţifică, didactică şi educaţională a subiectelor publicate, de limbajul ştiinţificadoptat, de grafica de excepţie utilizată în acea vreme. Am prezentat aceste aspecteîn articolul "Centenarul revistei Recreaţii ştiinţif ice", Problemedeistoriaşi filozofiaştiinţei, vol. X, 1984, Filiala Iaşi. Valabilitatea afirmaţiilor făcute atunci îşi păstreazătemeiul şi astăzi. Din acest motiv reproduc o parte din text.Tonul întregii producţii matematice, cuprinzând mai bine de 90% din cele 1920de pagini cât însumează această revistă, a fost dat în primul rând de fondatorii ei,care, prin prestigiul lor, au atras foarte curând valoroşi colaboratori: Miltiade Tzony— profesor de mecanică teoreticălaUniversitateadinIaşi, Candide (probabil VictorCostin, pe atunci student la Paris), Iacob Solomon — inginer, Paul Tanco — profesorde matematică şi f izică laGimnaziulSuperiordinNăsăud, Constantin Gogu —profesor de geometrie analitică laUniversitateadinBucureşti, Vasile Buţureanu —profesor de mineralogie şi petrografie la Facultatea de ştiinte din Iaşi ş. a.În paginile revistei sunt publicate articole, note, probleme şi soluţii din domeniica: aritmetică, algebră, geometrie elementară, geometrie analitică şi diferenţială, calculdiferenţial şi integral, mecanică, astronomie, istoria matematicii, chimie, f izică,geograf ie. Apare prima traducere a cărţii întâi din celebrele "Elemente" ale lui Euclid.Miltiade Tzony tipăreşte în coloanele ei o remarcabilă culegere de problemede mecanică teoretică. Geometria proiectivă, domeniu de mare actualitate în aceavreme, este prezentă prin traducerea primelor opt paragrafe din vestita lucrare "Geometriade poziţie" a lui Criristian von Staudt. Întâile elemente din istoria matematicilorîn antichitate sunt transpuse în limba română de Iacob Solomon.La succesul binemeritat al Recreaţiilor ştiinţif ice a contribuit şi prezentarea graficăexcelentă. Scrisă într-o limbă literară elevată, revista are, cu excepţia unor termenimatematici în formare, ceva din culoarea şi prospeţimea revistelor actuale.Privită global, ca act de cultură ştiinţif ică, revista rivalizează cu cele mai bunepublicaţii de acest gen tipărite acum un secol pe plan mondial.Închei aici relatarea din articolul amintit. Dar ultima frază trebuie corectată cu"acum un secol şi un sfert pe plan mondial".Subliniez faptul că în tot cuprinsul celor şase volume ale revistei impresioneazăgrija pentru rigoarea prezentării, acurateţa exprimării în limba română, actualizareaexpunerilor, informaţia de ultimă oră, profunzimea raţionamentelor şi, nu în ultimulrând, atenţia acordată contribuţiilor personale ale tinerilor rezolvitori sau autori aleproblemelor propuse spre publicare.A fost realizată astfelînpremieră, o revistă românească extremdeimportantăpentru învăţământ şi cercetare în ştiinţele exacte, de acelaşi nivel cu reviste similarecosacrate şi vestite din lume. După şapte ani de la stingerea activităţiiacesteireviste,ideea de a răspândi în rândul tineretului pasiunea pentru matematică vafipreluata101


de "Gazeta Matematică".La 125 de ani de la apariţia revistei "Recreaţii ştiinţifice", generaţiile de astăziomagiază acest eveniment ca semn de adâncă recunoştinţă adusă înaintaşilor noştripentru contribuţia lor inestimabilă latezaurulştiinţei şi culturii româneşti.Şi un scurt adaos: Centenarul apariţiei revistei "Recreaţii ştiinţifice" a fost organizatîn 1983 de matematicieni ieşeni în Seminarul Matematic "Alexandru Myller",iar sărbătorirea celor 125 de ani de la apariţie a fost iniţiată tot în cadrul acestuiSeminar pregatită fiind de Asociaţia "Recreaţii <strong>Matematice</strong>", Facultatea de matematicăşi Institutul de Matematică "OctavMayer"delaFilialadinIaşi a AcademieiRomâne. Asociaţia "Recreatii <strong>Matematice</strong>" îşifaceuntitludeonoareprinreeditareaintegrală, exclusiv prin grijă proprie, a colecţiei revistei "Recreaţii ştiinţifice".Acest fapt îl datorăm prof. univ. Temistocle Bîrsan de la Universitatea Tehnică"Gheorghe Asachi" din Iaşi, cercetătorului dr. Dan Tiba de la Institutul de Matematicăal Academiei Române din Bucureşti şi doamnei Marinela Ghigea — director alfirmei Kepler Systèmes d’ In<strong>format</strong>ion. Îi asigurăm de toată preţuirea şi gratitudineanoastră.Rolul şi ponderea geometrieiîn revista „Recreaţii Ştiinţifice”Prof. dr. Vasile OPROIUÎn revista Recreaţii ştiinţifice, scrisă şi editată deungrupdeoamenideştiinţăşi culturăinimoşi (N. Culianu, C. Climescu, I. Melik, G. I. Lucescu, V. Paladi,G. I. Roşiu, I. D. Rallet, G. Zarifopol, I. V. Praja, şi I. M. Dospinescu),s-au adunat şi publicat diferite materiale din domeniile matematicii, fizicii, chimiei,geografiei, cosmografiei, topografiei, mineralogiei, istoriei matematicii etc. <strong>Revista</strong>se adresa elevilor din clasele de gimnaziu şi liceu, dar şi altor categorii de persoaneinteresate de cunoaştere: profesori, studenţi, funcţionari, militari etc.Geometria, ca ramura a matematicilor are o pondere destul de însemnata înpaginile revistei. Trebuie să menţionăm, de la început, că G.I.Roşiu publică întreanii 1883-1885 prima carte a Elementelor lui Euclid (traducere după oediţieitaliană). Am regăsit cu o anumită emoţie şi nostalgie multe formulări pe care leîntâlnisem când eram student şi apoi le citisem în cărţile lui Efimov (ediţia în limbaromână şi cea în franceză) şi în cartea lui I.Vaisman. Astfel (în volumul II al Recreaţiilor),printre definiţiile lui Euclid am regăsit formulări precum: Punctul este aceea cenu are părţi, adecă nuareniciomărime, Linia este lungime fără lărgime, Suprafaţaplană este aceea care este aşezată egal în respectul tuturor liniilor sale drepte. Postulateleşi axiomele au fost publicate anterior, în primul volum, într-o ordine diferită102


de cea cu care suntem obişnuiţi. Astfel, faimosul postulat V al lui Euclid apare caaxioma XII. Autorul prezintă şi definiţiile şi axiomele, aşa cum au fost prelucratede Legendre în Geometria sa, ediţia V, Paris, 1804. Legendre nu defineşte punctulşi, în legatură cudefiniţia, formulează următoarea aserţiune: Def iniţia unui lucrueste exprimarea raporturilor sale catră lucruri cunoscute. După care, se încumetă sădefinescă dreapta: Linia dreaptă este drumul cel mai scurt de la un punct la altul.Mai sunt prezentate comentarii critice ale diverşilor matematicieni relativ la acestedefiniţii, inclusiv noi definiţii: Cea mai simplă din toate liniile este linia dreaptă acăria noţiune este familiară latoţi şi despre care ni da o idee un f ir întins. Apoisunt prezentate propoziţiile de la I la XXII. Cum spuneam, prezentarea Elementelorcontinuă în volumul III cu propoziţiile rămase şi cu exerciţiile la cartea I.În revistă sunt prezentate numeroase aspecte ale geometriei elementare, utileelevilor, profesorilor şi altor persoane interesate: maxime şi minime geometrice, mediaşi extrema raţie, calculul lungimilor unor linii importante din triunghi, calcule pentrupatrulaterul inscriptibil (în primul volum), proprietaţi ale poligoanelor şi dreapta luiSimson (în vol. II), calcularea volumelor piramidei trunchiate şi al conului trunchiat,proprietaţi sintetice ale elipsei (în vol. III), teoria transversalelor, diviziunea armonică,fasciculul armonic, poli şi polare, geometria de poziţie a lui Staudt (în vol. IV);aceasta din urma se continua şi în volumul V. Chestiunile de geometrie analitică suntconsiderate separat şi se referă la: secţiuni plane în conul drept, construcţii de curbe,cu exemplificări din clasele curbelor celebre, tratarea acestora în coordonate polare,plane principale la suprafeţele de gradul al doilea.Osecţiune importantă înrevistă este cea a problemelor propuse (de regulă, înjur de 10 probleme la fiecare număr), la care se adugă, pe parcurs, cea cu rezolvărileşi listele de rezolvitori. Trebuie să menţionăm că, în Regatul României de atunci,existau câteva zeci de gimnazii şi licee (oricum, sub 30) şi că numărul celor carerezolvau probleme era destul de mic. Moda rezolvărilor de probleme la reviste dematematică nuprinseseîncă. Menţionăm că existauşi colaborărivenitedelaelevidin Transilvania, Banat şi alte regiuni ale viitoarei Românii Mari.Ca o apreciere cu caracter general, conţinutul revistei Recreaţii Ştiiţifice eradestul de ridicat din punct de vedere al nivelului chestiunilor de geometrie tratate.Subiectele erau interesante şi atractive pentru numeroşi cititori. Cred că, în redacţiarevistei, erau persoane care doreau să facă revista cât mai atractivă.Răsfoind cele şase volume am dat şi peste un articol fascinant de la secţia cosmografie,scris de G.I. Lucescu, în care se explica în ce manieră au fost conceputecalendarele iulian şi gregorian şi că motivul pentru care s-a făcut trecerea de la unulla altul a fost legat de ideea că, în acord cu hotărârea Conciliului de la Niceea dinanul 325, punctul de plecare pentru fixarea zilei de Paşti trebuia să fieechinocţiulde primăvară şi acesta trebuia să fie mereu la 21 martie. După aceea, se aşteaptaprima noapte cu lună plină şi Paştele se fixa în duminica imediat următoare (astfel,în 2008, noaptea cu lună plină cade exact în 21 martie şi Paştele catolic este fixat în23 martie; fixarea Paştelui ortodox este mult mai complicată şi ţine de nişte date din103


calendarul iudaic). De la data Conciliului de la Niceea până în 1582 calendarul iulianrămăsese în urmă cu 10 zile faţa de calendarul real (anul din calendarul iulian erapuţin mai lung decât anul real). Acest lucru influenţa foarte mult diverse activitaţipractice, de exemplu, unele lucrări agricole ce se făceau în strânsă legătură cusărbatorilereligioase. În 1582, papa Grigore al XIII-lea a dat o bulă princaresedecideaavansarea calendarului iulian, existent, cu 10 zile şi că anii multipli de sute (maipuţin cei multipli de 400) nu sunt bisecţi. Acest calendar mai are o mica eroare careconstă înrămânerea în urmă cu o zi în circa 3300 ani, eroare considerată rezonabilăşi care va fi corectată în viitor. În ţară la noi, calendarul grigorian a fost adoptat în1923 (s-a trecut la stilul nou!), dată când întârzierea calendarului iulian faţă de celreal, sau cel grigorian, ajunsese la 13 zile.Revenind la revista Recreaţii ştiinţifice, apreciez că un cititor interesat poate săgăsească în cuprimsul ei lucruri incitante, atât în domeniul geometriei, cât şi în altedomenii ale matematicilor şi ale altor ştiinţe.Despre problemele de mecanicăProf. dr. Dorin IEŞAN, m.c. al AcademieiPe lângă alte chestiuni interesante, în revista Recreaţii ştiinţifice au apărutşi un număr de probleme de mecanică raţională, semnate de Miltiade Tzony. Pevremea când a publicat aceste probleme, M. Tzony era profesor de mecanicăteoreticăla Universitatea din Iaşi (a funcţionat în această calitate în perioada 23.X.1869 —15.III.1898).Miltiade Tzony este autorul unui curs de mecanică, prezentat în manuscris, îndouă volume. Primul volum a fost scris în 1869, iar al doilea volum în 1881. Peprima pagină a acestui curs autorul scrie "Cursŭ de Mecanică raţionale şi aplicată;Profesat la Universitatea de Jasy; După cei mai buni autori francezi: Delaunay,Sturm, Duhamel, Bellanger, Bresse, Bour, Collignon, Mesat, Chasles; de MiltiadeTzony, Licenţiat în ştiinţe matematice de la Sorbona din Paris, Ingineriu al şcoaleide poduri din Paris, Vechiu elevu al şcoalei politechnice din acestu oraşu, Profesor alUniversităţii de Jasy şi a Lyceului Nou din Jasy". Lecţiile de mecanică raţională dela Universitatea din Iaşi se făceau după acest curs.In perioada 1885-1888, M. Tzony publică Un curs de probleme în revista "Recreaţiiştiinţifice". El mărturiseşte că acest lucru îl face "în scopul de a uşura studenţilorUniversităţilor noastre completa pricepere a cursului de mecanică raţională şi nimeritaîntrebuinţare a principiilor acestei însemnate ştiinţi" (vol. III, pp. 77-78). Săvedem care este originea problemelor şi a rezolvărilor date. M. Tzony ne spune că"problemele sunt lucrate după diverşi autori între care figurează înprimullocabatele104


Jullien, a cărui carte în această materie a devenit clasică". Am constatat că toatecele 98 de probleme publicate de Tzony în "Recreaţii ştiinţifice" sunt luate din carteacălugărului P.M. Jullien Problèmes de mécanique rationnelle, apărută laParisînanul 1855. Cartea lui P.M. Jullien conţine atât probleme originale cât şi probleme alealtor autori. In "Recreaţii ştiinţifice" M. Tzony prezintă 36 probleme ale cărui autoreste P.M. Jullien, 24 de probleme datorate lui W. Walton şi 28 probleme ale altor autori(printre care Euler, Bernoulli, Leibniz, Laplace, Gauss, Möbius). Menţionăm căproblemele datorate lui W. Walton se găsesc în cartea acestuia ACollectionofProblemsin Illustration of the Principles of Theoretial Mechanics, apărută laCambridgeîn anul 1842.La începutul prezentării problemelor, M. Tzony afirmă că "de câte ori ne vaf i posibil vom indica la f inea f iecărei probleme autorul căruia se datoreşte". Problemelepublicate de Tzony sunt şi rezolvate şi cu "toate indicaţiunile necesare pentruaputeaficuprinsecuuşurinţă detânărul public cetitoriu căruia este în specialdestinat". M. Tzony susţine,pedreptcuvânt,că "opera abatelui Jullien este scrisăîntr-un mod atît de laconic încât cetirea ei de începători este foarte laborioasă şi înunele puncte aproape cu totul neînţeleasă". Dintre problemele publicate de Tzonyîn "Recreaţii ştiinţifice" un număr de 51 sunt rezolvate în cartea lui P.M. Jullien.Celelalte sunt probleme pe care Jullien le-a propus spre rezolvare. O parte dintreproblemele nerezolvate de Jullien sunt însă însoţite de figuri şi răspunsuri în cartealui W. Walton. Am comparat aceste rezolvări şi figuri cu cele date de M. Tzony în"Recreaţii ştiinţifice". Se poate spune cu certitudine că Tzony nu s-a inspirat dincartea lui W. Walton.Fiecare capitol din culegerea de probleme este prefaţat cu "o scurtă amintirearezultatelor f inale ale teoriei, întovărăşite de câteva noţiuni istorice pe care, în lipsaaltor documente, le vom împrumuta pentru cea mai mare parte din opera ştiinţif icăde care facem menţiune". Este uşor de văzut că aceste comentarii sunt traduse dincartea lui P.M. Jullien.Menţionăm că rezolvările prezentate de M. Tzony sunt clare, iar figurile suntîngrijite şi binevenite. Un lucru remarcabil este faptul că şi în cazul problemelorrezolvate de Jullien, M. Tzony face figuri suplimentare şi adaugă explicaţii. In problemelepublicate de Tzony în "Recreaţii ştiinţifice" sunt 88 de figuri, dintre care 66nu se află în cartea lui Jullien.Referitor la repartiţia pe ani a problemelor se constată că în anul 1885 suntpublicate 18 probleme, în anul 1886 apar 33 probleme, în anul 1887 sunt publicate23 probleme, iar în anul 1888 apar 18 probleme. In anul 1888 revista "Recreaţiiştiinţifice" îşi încetează apariţia. Acest lucru a curmat publicarea firească aaltorprobleme.Menţionăm că problemele apărute se referă doar la partea de Statică acursuluipredat de Tzony la Universitate. Dintre acestea, 18 se referă la echilibrul punctuluimaterial, 39 la echilibrul corpului rigid, 6 tratează echilibrul unui sistem de barearticulate, 27 sunt dedicate echilibrului firelor, iar 8 se referă la principiul "lucrului105


mecanic virtual".CursulluiM.Tzony(înmanuscris)şi problemele publicate de el în "Recreaţiiştiinţifice" au stat la baza învăţământului Mecanicii din ţara noastră.În afară de activitatea de profesor, Miltiade Tzony s-a remarcat prin munca sadepusă în vederea propăşirii României. A fost senator, secretar de stat la MinisterulConstrucţiilor Publice, director al C.F.R. Printre altele, oraşul Iaşi îi datoreazăpavarea străzilor.Despre M. Tzony se pot spune multe lucruri. Un fost elev de-al său, Petru N. Culianu,care a urmat cursurile de mecanică delaUniversitateadinIaşi în anii 1890,îl descrie pe Miltiade Tzony astfel: "Cu mintea ageră, cu f igura frumoasă (afostluat ca model de pictorul Grigorescu pentru unele f iguri din biserica de la mănăstireaAgapia), ce corespund întru totul nobleţei caracterului său, el a fost cu totul devotatdatoriei şi înaltei misiuni a profesorului".Problematica de algebră şi analiză matematicăîn revista „Recreaţii Ştiinţifice”Prof. dr. Teodor PRECUPANUPentru matematica românească, apariţia revistei Recreaţii ştiinţifice din iniţiativaunei elite de profesori ai universităţii şi ai liceelor din Iaşi, reprezintă unmoment important, de început, pentru crearea unei atmosfere propice dezvoltării ştiinţelormatematice, de atragere a tinerilor, stimulându-i şi amplificându-le pasiunile,călăuzindu-i spre problemele moderne ale acelei perioade.Este de remarcat faptul că iniţiatorii revistei erau la curent cu multe din preocupărileexistente în matematica europeană, având o bună informare, facilitată deaccesul la o serie de reviste importante, îndeosebi din Franţa, Italia şi Germania. Suntsemnalate nu numai apariţia unor rezultate importante, ci şi diverse evenimente alecomunităţii ştiinţifice, cum ar fi, spre exemplu, apariţia revistei Acta Matematicafondată deMittag-Leff ler prin casa regală suedo-norvegiană, solemnitatea retrageriidin învăţământ la vârsta de 70 de ani a marelui matematician Eugène-CharlesCatalan, apariţia unor tratate importante de matematică, discuţiile generate deproiectul Turnului Eiffel, ceurmasăseconstruiascăînParis.Adresându-se în primul rând elevilor din învăţământul secundar, revista a stimulat,de asemenea, preocupările profesorilor pentru modernizarea învăţământuluimatematic, găzduind în paginile sale dezbateri interesante cu caracter metodic asupraprogramelor analitice şi a metodelor prin care să fieatraşi elevii pentru studiulmatematicii, să seasigureocâtmaibună accesibilitate, punând pe primul planintuiţia şi dezvoltarea abilităţilor de rezolvitori de probleme.106


miercuri sau într-o vineri.Este uşor de stabilit când avem un an bisect. Se împarte numărul anului dincalendarul iudaic prin 19. Dacă restulesteunuldinnumereledemaisus,adică0,3, 6, 8, 11, 14 sau 17, acel an este bisect. De exemplu: anul 5744 (1983 — 1984în caledarul gregorian) este un an bisect, deoarece împărţirea prin 19 dă câtul 302şi restul este 6. Mult mai dificil este de a se stabili lungimea unui an iudaic (353,354 sau 355 de zile într-un an ordinar, respectiv 383, 384 sau 385 de zile într-un anbisect). Calendarul iudaic are o întâziere de o zi în 216, 4 ani.Addendum. La catolici şi protestanţi, data Paştelui este aceeaşi. Ea se stabileştedupă oregulăsimplă: Duminica Paştelui este prima duminică de după primalunăplină ceaparedupă echinoxiul de primăvară.Luna plină Data PaşteluiLuna plină Data Paştelui2005 05.03 27.032009 09.04 12.042006 13.04 16.042010 30.03 04.042007 02.04 08.042011 18.04 24.042008 21.03 23.032012 06.04 08.04Data Paştelui pentru ortodocşi se stabileşte după un algoritm mult mai complicat.Interesant este faptul că acest algoritm se bazează tot pe ciclul de 19 ani, ca şi pentrucalendarul iudaic. De aceea, nu întâmplător, Duminica Paştelui pentru ortodocşi esteîntotdeauna în ultima zi, puţin înaintea ultimei zile sau imediat după ultima zi dincele 8 zile ale sărbătorii Paştelui (PESAH) din calendarul iudaic.Ultima zi PaşteleUltima zi Paştelede PESAH ortodox2005 dum. 01.05 01.052006 joi 20.04 23.042007 marţi 10.04 08.042008 dum. 27.04 27.04de PESAH ortodox2009 joi 16.04 19.042010 marţi 06.04 04.042011 marţi 26.04 24.042012 sâmb. 14.04 15.04Bibliografie1. G. Petrescu - Astronomie elementară, Bucureşti, 19622. G. Stănilă - Sisteme calendaristice, Bucureşti, 19803. www.jewfaq.org: Judaism 1014. LUAH 5743 (Calendar 1982 -1983), editat de Federaţia Comunităţilor evreieşti dinRomâniaNota Redacţiei. În revista Recreaţii Ştiinţifice probleme ca: măsurarea timpului,alcătuirea unui calendar, stabilirea datei Paştelui au fost îndelung dezbătute şiaprofundate. Într-un ciclu de nouă articole (apărute în vol. I(1883), vol. II(1884) şivol. III(1885)), G. I. Lucescu, profesor la Liceul Naţional din Iaşi, face un istoric alcalendarului, din cele mai vechi timpuri şi până la adoptarea calendarului gregorian.În cuprinsul a cinci scrisori publicate în vol. VI(1888), Constantin Gogu, profesorla Universitatea din Bucureşti, se ocupă deregulilepentrugăsirea zilei Paştelui.Paul Tanco din Năsăud, primul român cu titlul de doctor în matematici, ridicăproblema periodicităţii cu care Paştele cade a doua zi după Sf. Gheorghe.111


Câteva probleme de teoria numerelor a cărorrezolvare se bazează peidentităţiMarian TETIVA 1Când vorbim despre utilizarea identităţilor în teoria numerelor, probabil că negândim în primul rând la ecuaţii diofantice, în special la demonstrarea existenţeisoluţiilor unor asemenea ecuaţii. De exemplu, identităţile(m 2 − n 2 ) 2 +(2mn) 2 =(m 2 + n 2 ) 2şi(m 2 − 2mn − n 2 ) 2 +(m 2 +2mn − n 2 ) 2 =2(m 2 + n 2 ) 2arată căecuaţiile x 2 + y 2 = z 2 şi, respectiv, x 2 + y 2 =2z 2 au,fiecare,oinfinitatede soluţii întregi. Totuşi, în cele ce urmează, vom rezolva alte tipuri de probleme deteoria numerelor cu ajutorul identităţilor.Avem în minte mai ales două identităţi, binecunoscute cititorilor. Este vorba de(x − y)(x + y)(x 2 + y 2 ) ···(x 2n−1 + y 2n−1 )=x 2n − y 2n (1)şi de(x 2 + xy + y 2 )(x 2 − xy + y 2 )(x 4 − x 2 y 2 + y 4 ) ···(x 2n − x 2n−1 y 2n−1 + y 2n )== x 2n+1 + x 2n y 2n + y 2n+1 . (2)Ambele sunt valabile pentru orice numere complexe x şi y (dar, desigur că penoinevor interesa pentru numere întregi) şi orice număr natural n ≥ 1. Sejustificălafel,prin aplicarea repetată aformulei(a − b)(a + b) =a 2 − b 2 .Problema cea mai cunoscută care utilizează (1) este probabilProblema 1. Fie n ≥ 1 un număr natural. Să searatecănumărul N =11...1scris în baza 10 cu 2 n cifre de 1 are cel puţin n divizori primi distincţi.Soluţie. Într-adevăr, avemN =(10 2n − 1)/9 = (10 + 1)(10 2 +1)···(10 2n−1 +1)deci (1) ne permite să scriempeN ca produsul a n numere; dacă reuşim să arătămcă acesteasuntprimeîntreeledouă câte două problemaarfirezolvată, deoarecefiecare din aceste n numere ar aduce (cel puţin) un factor prim în descompunerea luiN ca produs de numere prime.Avem, pentru 0 ≤ i


şi să observăm că avemm 2 − m +1 2p.Astfel că 2m 2 > 2m 2 − 2m +1> 2p >p, deci produsul (2m 2 )! conţine factorii(distincţi) 2m 2 − 2m +1, 2p şi p, deci se divide cu (2m 2 − 2m +1)p 2 =(2m 2 ) 2 +1;prin urmare sunt soluţii toate numerele n =2m 2 , unde m =(x k − 1)/2, k ≥ 2.Se poate vedea prin calcul direct că soluţia cea mai mică esten =18(18! se dividecu 18 2 +1 = 5 2 · 13). Această soluţiefacepartedinşirul de mai sus; se obţine pentruk =1,cândm =3şi p =5(inegalitatea 2m 2 − 2m +1> 2p are loc şi în acest caz,chiar dacă nuarelocm 2 > 2p).Problema 4. Pentru un număr natural n ≥ 2 notăm cu h(n) cel mai maredivizor prim al lui n. Săsearatecăexistăoinfinitatedenumeren astfel încâth(n)


Soluţie. De astă dată, pe lângă identitatea (2), vom folosi şi o idee ceva maisubtilă. Anume, să fixăm un număr prim impar p şi să observăm că, la fel ca laProblema 1, oricare două dintre numerelep +1,p 2 +1,...,p 2k +1,...au cel mai mare divizor comun 2. De aceea există unuldintreelecarearemăcarun factor prim mai mare decât p. Săconsiderăm primul dintre aceste numere, adicăfie k acel număr natural pentru care h(p 2k +1) >pşi h(p 2j +1)


O caracterizare a punctului MathotCătălin ŢIGĂERU 1Punctul lui Mathot (sau anticentrul) unui patrulater inscriptibil este definit capunctul de intersecţie al perpendicularelor duse din mijlocul fiecărei laturi ale patrulateruluipe latura opusă. Numeroase proprietăţi ale acestui punct au fost puse înevidenţă, cele mai spectaculoase fiind în legătură cu cele patru triunghiuri <strong>format</strong>ede câte două laturi adiacente ale patrulaterului şi câte o diagonală.În acestănotă punem în evidenţă o caracterizare a punctului Mathot care se referăla cele patru triunghiuri <strong>format</strong>e de câte o latură şi câte două segmente determinatepe diagonale de punctul lor de intersecţie. Mai precis, demonstrămTeorema 1. Se consideră patrulaterul inscriptibil ABCD, înscris în cercul decentru O, în care se notează cuE intersecţia diagonalelor AC şi BD şi cu H 1 , H 2 ,H 3 , H 4 ortocentrele triunghiurilor AEB, BEC, CED şi respectiv DEA. Atunci:(a) Patrulaterul H 1 H 2 H 3 H 4 este paralelogram.(b) Intersecţia diagonalelor paralelogramului H 1 H 2 H 3 H 4 coincide cu punctul Mathotal patrulaterului ABCD.Pentru demonstraţie folosimLema 1. Dacă O 1 , O 2 , O 3 , O 4 sunt centrele cercurilor circumscrise triunghiurilorAEB, BEC, CED şi respectiv DEA, atunci(a) Patrulaterul O 1 O 2 O 3 O 4 este paralelogram.(b) Dacă Γ este punctul de intersecţie a diagonalelor paralelogramului O 1 O 2 O 3 O 4 ,atunci punctele O, Γ, E sunt coliniare, punctul Γ fiind mijlocul segmentului [OE].Demonstraţie. Cititorul poate verifica imediat faptulcă patrulaterul O 1 O 2 O 3 O 4 este paralelogram. Notămcu F piciorul perpendicularei din E pe AB şi unim E cuO 3 . Pedeoparteavemm( [FEA)=90 ◦ −m(\BAE); pedealtă partem( \CEO 3 )=90 ◦ −m(\CDE); cumm(\BAE) =m(\CDE), rezultă că m( [FEA) = m( \CEO 3 ), adică F ,E, O 3 sunt coliniare, deci EO 3 ⊥AB, deciO 3 E k OO 1 ;analog se demonstrează că O 1 E k OO 3 , O 4 E k OO 2 ,O 2 E k OO 4 , de unde rezultă că patrulaterele O 1 EO 3 O,O 2 EO 4 O sunt paralelograme. Cum diagonalele paralelogramelorse înjumătăţesc, rezultă că punctul Γ este mijlocul segmentului [OE].Demonstraţia Teoremei 1. Vom folosi şi următoarele rezultate:(A) Dacă UVWZ este un paralelogram, S este intersecţia diagonalelor sale şi Meste un punct oarecare din plan, atunci 4MS −−→ = MU −−→ + −−→ MV + MW −−→ + −−→ MZ ;(B) (Sylvester) Dacă M este centrul cercului circumscris triunghiului UVW şidacă S este ortocentrul triunghiului, atunci −−→ MS = MU −−→ + −−→ MV + MW −−→ ;(C) Dacă ABCD este inscriptibil, dacă O este centrul cercului circumscris şi dacăΩ este punctul Mathot al patrulaterului, atunci1 Lect. dr., Univ. "Ştefan cel Mare", Suceava−→OA + −→ −→ −−→ −→OB + OC + OD =2OΩ . (1)115


Se demonstrează imediatcă H 1 H 2 H 3 H 4 este paralelogram. Putem scrie: −→ OA =−−→OO 4 + −−→ O 4 A = −−→ OO 1 + −−→ −→ −−→O 1 A, OB = OO 1 + −−→ O 1 B = −−→ OO 2 + −−→ −→ −−→O 2 B, OC = OO 2 + −−→ O 2 C =−−→OO 3 + −−→ O 3 C, −−→ OD = −−→ OO 3 + −−→ O 3 D = −−→ OO 4 + −−→ O 4 D, de unde, prin însumare, obţinem:³ −→ −→ −→ −−→³ −−→2 OA + OB + OC + OD´=2 OO1 + −−→ OO 2 + −−→ OO 3 + −−→ OO 4´+³ −−→+ O1 A + −−→ O 1 B + −−→ ´ ³ −−→O 1 E + O 2 B + −−→ O 2 C + −−→ ´ ³ −−→O 2 E + O3 C + −−→ O 3 D + −−→ ´O 3 E +³ −−→+ O 4 D + O −−→4 A + −−→ ´O 4 E + −−→ EO 1 + EO −−→2 + −−→ EO 3 + −−→ (B)EO 4 =4X −−→4X −−−→4X −−→4X ³ −−→=2 OO i + O i H i + EO i = OOi + −−−→ 4X −−→4X −−→O i H i´+ OO i + EO i =i=1i=1=(A)+lema =i=1i=14X −−→ OHi +4 −→ OΓ +4 −→ EΓ.i=1Conform lemei, rezultă că −→ OΓ + −→ EΓ = ¯0. Dacănotăm cu Ω 0 punctul de intersecţie a diagonalelor paralelogramuluiH 1 H 2 H 3 H 4 şi, ţinând cont din nou de (A),Pavem n OH i =4OΩ 0 . Ca urmare, obţinemi=1³ −→ −→ −→ −−→2 OA + OB + OC + OD´=4 −−→ OΩ 0 . (2)Din (1) şi (2) obţinem că 4 −→ OΩ =4 −−→ OΩ 0 , de unde deducemcă Ω ≡ Ω 0 şi teorema este demonstrată. (Celespusesepot urmări pe figura alăturată.)O consecinţă imediată a teoremei este şi relaţia vectorială4X −−−→O i H i =4 −→ ΓΩ, (3)i=1care se deduce imediat folosind (A).Este posibil ca rezultatul notei să nu fie nou, dar sigur nu este trecut, de exemplu,printre proprietăţile punctului Mathot, demonstrate în capitolul consacrat subiectului,din monografia "Problems in plane and solid geometry", scrisă deViktorPrasolov,care este accesibilă pe Internet. Precizăm că autorul nu a găsit rezultatul niciîn cărţile citate în bibliografie, nici în alte cărţi clasice de geometrie, scrise în limbaromână.Bibliografie1. D. Mihalcea, I. Chiţescu, M. Chiriţă - Geometria patrulaterului, Ed. Teora,Seria Bacalaureat-Admitere, nr. 24, 1998.2. C. Mihalescu - Geometria elementelor remarcabile, Bibl.Soc.Şt. <strong>Matematice</strong> aS.S.M.R., Ed. Tehnică, Bucureşti, 2007.116i=1i=1


Unsprezece pătrate perfecteDan POPESCU 1Scopul acestei note este determinarea numerelor naturale în baza zece deforma aa...a| {z }bcc...c| {z }d, n ∈ N ∗ , care, pentru orice număr natural n, suntn cifre n cifrepătrate perfecte.Se va vedea cărezultatulobţinut are drept consecinţe un număr mare de problemepublicate în reviste de specialitate destinate elevilor (de gimnaziu).Din modul cum s-a formulat problema, rezultă căpătratele perfecte căutate segăsesc printre cele ce corespund unei valori particulare a lui n. Pentru n =4segăsesc, cu ajutorul calculatorului următoarele 11 pătrate perfecte de forma dorită:1) 1111022224 = 33332 2 ; a =1,b=0,c=2,d=4,2) 1111088889 = 33333 2 ; a =1,b=0,c=8,d=9,3) 1111155556 = 33334 2 ; a =1,b=1,c=5,d=6,4) 1111222225 = 33335 2 ; a =1,b=2,c=2,d=5,5) 4444222225 = 66665 2 ; a =4,b=2,c=2,d=5,6) 4444355556 = 66666 2 ; a =4,b=3,c=5,d=6,7) 4444488889 = 66667 2 ; a =4,b=4,c=8,d=9,8) 4444622224 = 66668 2 ; a =4,b=6,c=2,d=4,9) 9999400009 = 99997 2 ; a =9,b=4,c=0,d=9,10) 9999600004 = 99998 2 ; a =9,b=6,c=0,d=4,11) 9999800001 = 99999 2 ; a =9,b=8,c=0,d=1.Vom arăta că existăexact11 numere de forma aa...a| {z }bcc...c| {z }d care sunt pătraten nperfecte pentru orice n ∈ N ∗ , anume, acelea ce se scriu cu sistemele de cifre (a, b, c,d) pe care le-am întâlnit mai sus (în cazul n =4), adică(1)|11{z...1}022...2| {z }4=33| {z...3}2 2 , (7) 44| {z...4}88| {z...8}9=66| {z...6}7 2 ,n nnn+1 nn2(2) 11| {z...1}088...8| {z }9=33| {z...3} , (8) 44| {z ...4}622...2| {z }4=66| {z...6}8 2 ,n nn+1n nn(3) 11 ...1 55 ...5 6=33 ...3 4 2 , (9) 99 ...9 400...0 9=99 ...9 7 2 ,| {z }n+1(4) 11 ...1| {z }n(5) 44 ...4| {z }n(6) 44 ...4| {z }n| {z }n22 ...2| {z }n+122 ...2| {z }n+1355...5| {z }n| {z }n5=33 ...3| {z }n5=66 ...6| {z }n6=66 ...6| {z }n+1| {z }n5 2 , (10) 99 ...9| {z }n5 2 , (11) 99 ...92,| {z }n| {z }n600...0| {z }n800...0| {z }n| {z }n4=99 ...9| {z }n1=99 ...9| {z }n+18 2 ,2.1 Profesor, Colegiul Naţional "Ştefan cel Mare", Suceava117


În scopul propus, să notăm x n = aa...a| {z }bcc...c| {z }d, n ∈ N ∗ .Putemscrien nx n = aa...a| {z }· 10 n+1 − a · 10 n+1 + b · 10 n+1 + cc...c| {z }+ d − c =n+1n+1= aa...a| {z }· ¡10 n+1 − 1 ¢ +(b − a) ¡ 10 n+1 − 1 ¢ + aa...a| {z }+ cc...c| {z }+ d − c + b − a,n+1n+1 n+1adică2x n =9a11| {z...1} +(9b + c − 8a) · 11| {z ...1}+ b + d − a − c, n ∈ N ∗ . (∗)n+1n+1Acum, să observăm că în(∗) coeficienţii 9a, 9b + c − 8a şi b + d − a − c sunt aceiaşipentru orice n ∈ N ∗ (ei reflectând numai forma lui x n ). Ca urmare dacă trinomuldin membrul doi este pătrat perfect pentru o valoare particulară aluin, atunci vaavea această proprietate pentru orice n ∈ N ∗ .Acestfaptverificându-se direct pentrun =4, vom deduce că numerele (1) − (11) sunt cele căutate.Observaţia 1. i) Cititorul poate observa că, pentru 1 ≤ n ≤ 3, existăpătrateperfecte de forma aa...a| {z }bcc...c| {z }d care nu apar printre cele unsprezece. Un exemplun npentru n =2: 344 2 = 118336. Mai precis, pentru n =2,există18pătrate perfectede forma enunţată, iar pentru n = 3, numărul lor este 12. În cazul n =1,numărullor este mult mai mare, căci problema se reduce la identificarea pătratelor perfectecu patru cifre ale sistemului zecimal.ii) Elevul Aursulesei Tudor, căruia îi mulţumim şi cu acest prilej, a verificat prinintermediul calculatorului faptul că, pentru 4 ≤ n ≤ 14, singurele pătrate perfectede forma aa...a| {z }bcc...c| {z }d sunt exact cele unsprezece prezentate mai sus.n nObservaţia 2. i) Singurele pătrate perfecte de forma aa...abb...bc, ∀n ∈ N ∗sunt: 11| {z...1}55| {z...5}6, n ∈ N ∗ şi 44| {z...4}88| {z...8}9, n ∈ N ∗ .n+1 nn+1 n| {z }n+1| {z }nii) Singurul pătrat perfect de forma aa...a| {z }bb...b| {z }c, ∀n ∈ N ∗ este 44| {z...4}22| {z...2}5,n n+1n n+1n ∈ N ∗ .Aplicaţii1. Să searatecănumărul N = 11| {z...1}22| {z...2}5 este pătrat perfect (OBM -1997 1998juniori, Atena, 1998) [1].Este un caz particular al rezultatului (4); N = 33| {z...3}5 2 .19972. Să se determine cifrele x şi y, x 6= 0,dacă xx...x| {z }6 yy...y4 este pătrat| {z }n nperfect, pentru orice n ∈ N ∗ .Rezolvarea problemei decurge din (8) şi (10), deci avem (x, y) ∈ {(4, 2) , (9, 0)}.118


3. Nu există pătrate perfecte în baza zece de forma aa...a| {z }; a 6= 0, n ≥ 2.nRezultă din cele prezentate mai sus; o altă abordare poate fi găsită în[3].4. Rezultateledela(3)şi(7)suntprezenteîn[3].5. Să searatecă numerele a = 11 ...1| {z }2n− 22 ...2| {z }nşi b = 44 ...4| {z }2n− 88| {z...8}suntnpătrate perfecte, ∀n ∈ N, n ≥ 2.Se arată că a = 11| {z...1}088...8| {z }9 şi se aplică (2), iar b = 44| {z...4}355...5| {z }6 şi sen−1 n−1n−1 n−1aplică (6).6. Să searatecăexistăoinfinitate de numere cu terminaţia 0004 care suntpatrate perfecte.Se poate utiliza egalitatea (10).7. Este numărul a = √ |4 4{z...4}355...5| {z }6 natural? (P. Bătrîneţu - ONM (lista2007 2007scurtă), Piteşti, ediţia 2007 [6]).Observaţia 3. i) În lista scurtă cu problemele propuse la Olimpiada Naţionalăde Matematică, ediţia 2005 [6], E. Velcea a propus problema care face obiectulrezultatului de la (6).ii) Rezultatul de la (4) a constituit o problemă delaConcursul Interjudeţean"Gh.Ţiţeica", ediţia 2004.iii) Autorul acestei note nu a identificat enunţuri legate de rezultatele de la (1),(9) şi (11).În final, propunem următorul exerciţiu (poate cu o alta abordare):Să searatecănuexistă numere în baza zece cu scrierea poziţională aa...a| {z }bb...b| {z },n ncare, pentru ficare număr natural nenul n, să fie pătrate perfecte.Bibliografie1. D. Brînzei ş.a. - 10 ani de Olimpiade Balcanice ale Juniorilor, Paralela 45, 2007.2. N.B. Vasiliev, A.A. Egorov - Zadaci vsesoiuznîi matematiceskih olimpiad-ebvisa,Nauka, Moscova, 1988.3. A.P. Ghioca, L.A. Cojocaru - Matematica gimnazală dincolodemanual, Gil,Zalău, 2005.4. I. Cucurezeanu - Pătrate şi cuburi perfecte de numere întregi, Gil, Zalău, 2007.5. Gazeta Matematică, Seria B, nr. 12/2005, Problema E:13095.6. Romanian Mathematical Competitions, Theta, Bucureşti, 2005.7. Romanian Mathematical Competitions, Theta, Bucureşti, 2007.119


Cercuri semiînscriseşi puncte de tip Gergonne sau NagelTemistocle BÎRSAN 1Fie ABC un triunghi oarecare. Pentru cercurile circumscris, înscris, A-exînscrisetc. folosim notaţiile uzuale: C (0,R), C (I,r), C (I a ,r a ) etc. Punctele de tangenţăadrepteiBC cu cercurile C (I,r) şi C (I a ,r a ) se notează D şi D 0 ;cuE, E 0 şi F , F 0notăm punctele cu semnificaţii similare relativ la dreptele CA şi, respectiv, AB.Este cunoscut faptul că dreptele AD, BE şi CF sunt concurente (într-un punctΓ — punctul lui Gergonne) şi, de asemenea, faptul că dreptele AD 0 , BE 0 şi CF 0 suntconcurente (într-un punct N — punctul lui Nagel).Se asociază triunghiului ABC trei cercuri semiînscrise: C (J 1 ,ρ 1 ), C (J 2 ,ρ 2 ),C (J 3 ,ρ 3 ) (C (J 1 ,ρ 1 ) fiind cercul tangent dreptelor AB şi AC şi tangent interior cerculuicircumscris triunghiului etc.), precum şi trei cercuri ex-semiînscrise: C (J a ,ρ a ),C (J b ,ρ b ), C (J c ,ρ c ) (C (J a ,ρ a ) fiind cercul tangent dreptelor AB şi AC şi tangentexterior cercului C (O, R) etc.). Observăm că avem un singur cerc înscris, dar treicercuri semiînscrise; pe de altă parte,numărul cercurilor exînscrise este egal cu cel alcelor ex-semiînscrise. Privitor la cercurile semiînscrise, un număr de proprietăţi alelor sunt date în [3] şi [1].Ne propunem în această Notăsă"trecem" cele două rezultate mai susmenţionate la cercurile semiînscriseşi ex-semiînscrise.În scopul propus, să notăm D 1 şiD a punctele de tangenţă a cercurilorC (J 1 ,ρ 1 ) şi, respectiv, C (J a ,ρ a ) cuC (O, R); E 1 , E b şi F 1 , F c au semnificaţiianaloage.Odată cu trecerea de la cerculC (I,r) la cele trei cercuri semiînscriseC (J i ,ρ i ) (i = 1, 2, 3), estefiresc să considerăm în rolul cevienelorGergonne AD, BE şi CFcevienele AD 1 , BE 1 şi, respectiv,CF 1 . Similar, în locul cevienelorNagel AD 0 , BE 0 şi CF 0 săconsiderămcevienele AD a , BE b şi, respectiv,CF c legate de cercurile exsemiînscriseC (J a ,ρ a ) etc.Vom arăta că rezultatelor clasice de mai sus le corespund cele din următoareaTeoremă. a) Cevienele AD 1 , BE 1 şi CF 1 sunt concurente în centrul S alomotetiei directe a cercurilor C (O, R) şi C (I,r).1 Prof. dr., Universitatea Tehnică "Gh.Asachi",Iaşi120


) Cevienele AD a , BE b şi CF c sunt concurente în centrul S 0 al omotetiei inversea cercurilor C (O, R) şi C (I,r).Demonstraţie. a) Evident, omotetia HA k ,cuk = ρ 1, transformă cerculC (I,r)rîn C (J 1 ,ρ 1 ),pecândomotetiaHD k01,cuk 0 = R , transformă C (J 1 ,ρρ 1 ) în C (O, R). Ca1urmare, produsul HD k01◦ HA k are centrul pe AD 1 şi raportul kk 0 = ρ 1r · R = R ρ 1 r .Cumtransformă C (I,r) în C (O, R), acest produs coincide cu omotetia directă aacestorcercuri. În consecinţă, AD 1 trece prin S — centrul omotetiei directe a cercurilorC (I,r) şi C (O, R) (situat pe OI şi definit de relaţia SO = R SI). Similar se aratărcă drepteleBE 1 şi CF 1 trec prin S.b) Se procedează lafel. HA t ,cut = ρ ar , transformă C (I,r) în C (J a,ρ a ), iarHD t0a,cut 0 = − R , transformă C (J a ,ρρ a ) în C (O, R). Omotetia produs HD t0a◦ HA t ,acu raportul tt 0 = − R ,coincidecuomotetiainversă a cercurilor C (I,r) şi C (O, R).rAD a conţine centrul S 0 al acestei din urmă omotetii (situat pe OI şi determinat deS 0 O = − R r S0 I). Se arată similar că şi BE b , CF c trec prin S 0 . Q.e.d.Observaţia 1. Demonstraţia standard pentru concurenţa cevienelor Gergonne(sau Nagel) se bazează pe reciproca teoremei lui Ceva. Acest instrument poate fiutilizat şi pentru stabilirea afirmaţiilor a) şi b), darcupreţul unor calcule laborioase.Astfel, dacă notăm X = BC ∩ AD 1 ,segăseşte că BXXC = c2b 2 · p − b (2p = a + b + c).p − cAceastă relaţie şi cu analoagele ei fac posibilă aplicarea reciprocei teoremei lui Cevaşi, deci, dovedirea concurenţei dreptelor AD 1 , BE 1 , CF 1 . Faptul că S este punctullor de concurenţă devine o chestiune de rutină, care cere noi calcule; de exemplu, sepoate utiliza Propoziţia2din[2] şi lista de coordonate triliniare din [4]. În concluzie,este preferabilă demonstraţia dată pebazaprodusuluiadouă omotetii.Observaţia 2. În [3], sub formă deproblemăpropusă cititorilor spre rezolvare,este afirmată concurenţa dreptelor AD 1 , BE 1 , CF 1 (cu alte notaţii), fără afifăcutăvreo precizare asupra punctului lor de concurenţă.Observaţia 3. În [5], într-o interesantă Notă de geometria triunghiului, centrelede omotetie S şi S 0 apar ca puncte de concurenţă ale altor două triplete de drepteasociate unui triunghi dat.Bibliografie1. R. Bairac - Cercuri semiînscrise în triunghi, Delta, 1/2006, 12-15.2. T. Bîrsan - Ceviene izogonale şi puncte de concurenţă remarcabile, 9/2002, 321-326.3. A. Girici - Câteva probleme despre triunghiuri şi cercuri, Kvant, 11/1990, 46-48.4. C. Kimberling - Centrul Points and Central Lines in the Plane of a Triangle,Mathematics Magazine, 67(1994), no.3, 163-187.5. I. V. Maftei - Două puncte remarcabile într-un triunghi, G.M. (B) — 1/2008, 1-4.121


O rafinare a inegalităţii lui JensenFlorin POPOVICI 1Cu o demonstraţie simplă, stabilim un criteriu de monotonie a funcţiilor; ca aplicaţie,prezentăm o rafinare a inegalităţii lui Jensen, despre care credem că estenouă.1. Preliminarii. Fie a, b ∈ R, cua


(0, ∞), are loc inegalitatea lui Jensen rafinată:µ p 1 f (a 1 )+···+ p n f (a n ) p1 a 1 + ...p n a n− f≥p 1 + ···+ p np 1 + ···+ p n≥ (p µ 1 + p 2 ) f (a 2 )+···+ p n f (a n ) (p1 + p 2 ) a 2 + ···+ p n a n− f≥ ···≥ (4)p 1 + ···+ p np 1 + ···+ p n≥ (p µ 1 + ···+ p n−1 ) f (a n−1 )+p n f (a n ) (p1 + ···+ p n−1 ) a n−1 + p n a n− f≥ 0.p 1 + ···+ p np 1 + ···+ p nDemonstraţie. Stabilim prima inegalitate din (4). Dacă a 1 = a 2 ,atunciprimainegalitate din (4) are loc cu egalitate. Dacă a 2 = a n , atunci prima inegalitate din(4) rezultă direct din definiţia convexităţii. Considerăm cazul a 1


Asupra unor inegalităţi geometriceGheorghe IUREA 1Rezultatul principal al notei [1] esteurmătoareaPropoziţie. Fie a, b, c lungimile laturilor unui triunghi; atunci, pentru oricex ≥ 0, au loc inegalităţile:(a + b + c) 3 (x +1) 3 ≥ 27 [(a−b)(1−x)+c (1+x)] [(b−c)(1−x)+a (1+x)] ·· [(c − a)(1− x)+b (1 + x)] , (1)(a+b−c) x + b+c−ap(ax + b)(bx + c)(b + c) x + a + c√ + ax + b+(b+c−a) x + c+a−b (c+a−b) x + a+b−cp + p ≤ 3, (2)(bx + c)(cx + a) (cx + a)(ax + b)(a + b) x + c + b√ + √ ≥bx + ccx + a³ √ax √ √≥ 2 + b + bx + c + cx + a´, (3)(c + a) x + b + aÎn cele ce urmează, vom demonstra că inegalităţile (1), (2) şi (3) au loc pentruorice a, b, c numere reale pozitive.Cu substituţiile α =(a − b)(1− x) +c (1 + x), β =(b − c)(1− x) +a (1 + x)şi γ = (c − a)(1− x) +b (1 + x), observând că α + β + γ = (a + b + c)(1+x),inegalitatea (1) se scrie sub forma (α + β + γ) 3 ≥ 27αβγ (1 0 ). Dacă αβγ < 0,atunci (1 0 ) este evidentă. Dacă αβγ ≥ 0, cumα + β, β + γ şi γ + α sunt nenegative,rezultă că α, β, γ sunt nenegative şi atunci (1 0 ) urmează imediat din inegalitateamediilor (MA ≥ MG). Egalitatea se atinge pentru a = b = c şi x ∈ [0, ∞) oarecare.Notând ax + b = α 2 , bx + c = β 2 , cx + a = γ 2 ,cuα, β, γ > 0, inegalitatea (2)devineα 2 + β 2 − γ 2+ β2 + γ 2 − α 2+ α2 + γ 2 − β 2≤ 3αββγαγcare, după calcule, poate fi scrisă subformaα (α − β)(α − γ)+β (β − α)(β − γ)+γ (γ − α)(γ − β) ≥ 0.Aceasta este însă cunoscuta inegalitate Schur. Egalitatea se atinge când a = b = c.Folosind aceleaşi substituţii, inegalitatea (3) este echivalentă cuβ 2 + γ 2α+ α2 + β 2γ+ α2 + γ 2βcare rezultă prin sumarea inegalităţilor β2α + α2γ + γ2βα + β + γ. Egalitatea are loc pentru a = b = c.≥ 2(α + β + γ) ,≥ α + β + γ şiγ2α + β2γ + α2β ≥Bibliografie1. I. V. Maftei, M. Haivas - Tehnici de stabilire a unor inegalităţi geometrice,Recreaţii <strong>Matematice</strong> 1/2008, 22-23.1 Profesor, Liceul Teoretic "Dimitrie Cantemir", Iaşi124


Metoda deligamentării şi rafinarea unor inegalităţiTitu ZVONARU 1Scopul acestei note este de a prezenta demonstraţii elementare pentru unele inegalităţi,ca şi obţinerea unor rafinări ale acestora. Descrierea metodei deligamentăriipoate fi găsită în[2].Pentru început, o demonstraţie prin metoda deligamentării a unei inegalităţicunoscute:1.ab + c +bc + a +ca + b ≥ 3 , a, b, c > 0.2NesbittSoluţie. Avemab + c − 1 2 = 2a − b − c = a − b2(b + c) 2(b + c) + a − c şi, analog,2(b + c)bc + a − 1 2 = b − c2(c + a) + b − a2(c + a) , ca + b − 1 2 = c − a2(a + b) + c − b . Grupând fracţiile2(a + b)în funcţiedenumărătorii lor, obţinem:a − b2(b + c) + b − a (a − b)(c + a − b − c) (a − b) 2= =2(c + a) 2(b + c)(c + a) 2(b + c)(c + a) ;împreună curelaţiile similare:b − c2(c + a) + c − b2(a + b) = (b − c) 22(c + a)(a + b) , c − a2(a + b) + a − c2(b + c) = (c − a) 22(a + b)(b + c) ,deducem valabilitatea inegalităţii de demonstrat.Metoda deligamentării, folosită în demonstraţia următoarelor inegalităţi, duce şila obţinerea unor rafinări ale acestora. Chiar dacă sunt necesare unele calcule, acesteasunt uşor de condus către rezultatul dorit.2.a(b + c) 2 + b(c + a) 2 + c 3(a + b + c)2≥(a + b) 4(ab + bc + ca), a, b, c > 0.Darij Grinberg şi Cezar LupuSoluţie. Demonstraţia din [1] faceapellainegalitatea lui Cebâşev şi la inegalitatealui Gerretsen. Avema(b + c) 2 − 3a4(ab + bc + ca) = a ¡ 4ab +4bc +4ca − 3b 2 − 3c 2 − 6bc ¢4(ab + bc + ca)(b + c) 2 ==(3ab + ac)(a − b)4(ab + bc + ca)(b + c) 2 + (3ac + ab)(a − c)4(ab + bc + ca)(b + c) 2şi, analog,b(c + a) 2 − 3b4(ab + bc + ca) = (3bc + ab)(b − c)4(ab + bc + ca)(c + a) 2 + (3ab + bc)(b − a)4(ab + bc + ca)(c + a) 2 ,c(a + b) 2 − 3c4(ab + bc + ca) = (3ac + bc)(c − a)4(ab + bc + ca)(a + b) 2 + (3bc + ac)(c − b)4(ab + bc + ca)(a + b) 2 .1 Comăneşti, e-mail: tzvonaru@yahoo.com125


Grupând convenabil, obţinem(3ab + ac)(a − b)4(ab + bc + ca)(b + c) 2 + (3ab + bc)(b − a)4(ab + bc + ca)(c + a) 2 ==a − b4(ab + bc + ca) · (3ab + ac)(c + a)2 − (3ab + bc)(b + c) 2(b + c) 2 (c + a) 2 ,şi cum(3ab + ac)(c + a) 2 − (3ab + bc)(b + c) 2 =3a 3 b + a 3 c +3abc 2 + ac 3 +6a 2 bc++2a 2 c 2 − 3ab 3 − b 3 c − 3abc 2 − bc 3 − 6ab 2 c − 2b 2 c 2 ==3ab ¡ a 2 − b 2¢ + c ¡ a 3 − b 3¢ + c 3 (a − b)+6abc (a − b)+2c 2 ¡ a 2 − b 2¢ ==(a − b) ¡ 3a 2 b +3ab 2 + a 2 c + abc + b 2 c + c 3 +6abc +2ac 2 +2bc 2¢ ==(a−b) ¡ a 2 b+ ab 2 + b 2 c+ bc 2 + a 2 c+ ac 2 +3abc+2a 2 b+2ab 2 +4abc+ c 3 + bc 2 + ac 2¢ ==(a − b) £ (a + b + c)(ab + bc + ca)+2a 2 b +2ab 2 +4abc + c 3 + bc 2 + ac 2¤ ,deducem că(3ab + ac)(a − b)4(ab + bc + ca)(b + c) 2 + (3ab + bc)(b − a)4(ab + bc + ca)(c + a) 2 == (a − b)2 £ (a + b + c)(ab + bc + ca)+2a 2 b +2ab 2 +4abc + c 3 + bc 2 + ac 2¤4(ab + bc + ca)(b + c) 2 (c + a) 2≥ (a − b)2 (a + b + c)(ab + bc + ca)4(ab + bc + ca)(b + c)(c + a) 2 = (a − b)2 (a + b + c)4(b + c) 2 (c + a) 2 .Prin permutări circulare obţinem încă douărelaţii similare. Rezultă următoarearafinare a inegalităţii date:a(b + c) 2 + b(c + a) 2 + c 3(a + b + c)2−(a + b) 4(ab + bc + ca) ≥Ã!≥ a + b + c (a − b) 24 (b + c) 2 (c + a) 2 + (b − c) 2(c + a) 2 (a + b) 2 + (c − a) 2(a + b) 2 (b + c) 2 .3.a 2b 2 + c 2 +Soluţie. Avemşi mai departeb2c 2 + a 2 +c2a 2 + b 2 ≥ab + c +bc + a +≥c , a, b, c > 0.a + bVasile Cârtoajea 2b 2 + c 2 − ab + c = ab (a − b)(b + c)(b 2 + c 2 ) + ac (a − c)(b + c)(b 2 + c 2 ) ,b 2c 2 + a 2 − bc + a = bc (b − c)(c + a)(c 2 + a 2 ) + ab (b − a)(c + a)(c 2 + a 2 ) ,c 2a 2 + b 2 − ca + b = ac (c − a)(a + b)(a 2 + b 2 ) + bc (c − b)(a + b)(a 2 + b 2 )126


ab (a − b)(b + c)(b 2 + c 2 ) + ab (b − a)(c + a)(c 2 + a 2 ) = ab (a − b)(b + c)(c + a)(b 2 + c 2 )(c 2 + a 2 )·= ab (a − b)2 ¡ c 2 + ac + bc + a 2 + ab + b 2¢(b + c)(c + a)(b 2 + c 2 )(c 2 + a 2 )· ¡c 3 + ac 2 + a 2 c + a 3 − b 3 − b 2 c − bc 2 − c 3¢ =≥ ab (a − b)2 ¡ c 2 + ac + bc + ab ¢(b + c)(c + a)(b 2 + c 2 )(c 2 + a 2 ) =ab (a − b) 2 (b + c)(c + a)=(b + c)(c + a)(b 2 + c 2 )(c 2 + a 2 ) = ab (a − b) 2(b 2 + c 2 )(c 2 + a 2 ) .Obţinem următoarea rafinare a inegalităţii în discuţie:a 2b 2 + c 2 +b2c 2 + a 2 +c2a 2 + b 2 − ab + c − bc + a − ca + b ≥ab (a − b) 2≥(b 2 + c 2 )(c 2 + a 2 ) + bc (b − c) 2(c 2 + a 2 )(a 2 + b 2 ) + ca (c − a) 2(a 2 + b 2 )(b 2 + c 2 ) .În încheiere, propunem cititorilor demonstrarea şi, eventual, rafinarea următoarelorinegalităţi:4. xn+1y + z + yn+1z + x + zn+1x + y ≥ xn + y n + z n, x, y, z > 0, n ∈ N.25. a, b, c fiind laturile unui triunghi, are loc inegalitateaaba + b − c + bcb + c − a + cac + a − b ≥ a + b + c.Gabriel DospinescuabIndicaţie.a + b − c − a + b = ... etc.26. a, b, c fiind laturile unui triunghi, avemb 2 + c 2a 3 + abc + c2 + a 2b 3 + abc + a2 + b 2c 3 + abc ≥ 1 ab + 1 bc + 1ca .Titu Zvonaru şi Bogdan Ioniţăb 2 + c 2Indicaţie.a 3 + abc − 12ab − 1 = ... etc.2acBibliografie1. C. Lupu - Asupra inegalităţii lui Gerretsen, R.M.T., 4/2006, 3-100.2. T. Zvonaru - Inegalităţi ligamentate şi neligamentate, Arhimede, 5-6/2003, 8-16.Semnalăm cititorilor reeditarea colecţiei complete a revisteiRECREAŢII ŞTIINŢIFICE (1883-1888),la 125 de ani de la apariţia primului număr, cu respectarea formei în care a fostpublicată iniţial. <strong>Revista</strong> prezintă şi astăzi interes prin culoarea limbii române şiterminologiei folosite, prin conţinutul interesant şi de un înalt nivel ştiinţific, precumşi prin forma grafică frumoasă. Cei interesaţi pot consulta site-ul revisteihttp://www.recreatiistiintifice.ro127


Oproblemă şi ... nouăsoluţiiGheorghe IUREA 1 ,GabrielPOPA 2În numărul 2/2007 al Recreaţiilor <strong>Matematice</strong>, Enache Pătraşcu a propus sprerezolvare problemaG133. Fie 4ABC echilateral şi D un punct astfel încât BD = DC, m(\BDC) =30 ◦ ,iarBC separă A şi D. Dacă E ∈ (BD) cu m(\BAE) =15 ◦ ,săsearatecăCE ⊥ AC.Soluţia autorului problemei (prezentată mai jos) recurge la o construcţie ajutătoareinteresantă, dar greu de găsit. Încercăriledeaabordaproblemaîntr-unmoddiferit au fost încununate de succes într-o măsură maimaredecâtneaşteptam; încele ce urmează potfigăsite nouă soluţii ale problemei, iar cititorul probabil că vamai observa şi altele.Soluţia 1. Notăm cu A 0 simetricul lui A faţă deBC. Observăm că m( \EBA 0 )=m(\EBC) − m( \A 0 BC) =75 ◦ − 60 ◦ =15 ◦ , prin urmare \EBA 0 ≡ \EAA 0 . Deducem căpatrulaterul ABEA 0 este inscriptibil, de unde \EA 0 B ≡ \EAB, adică m( \EA 0 B)=15 ◦ .Obţinem astfel că 4EBA 0 este isoscel cu EB = EA 0 şi de aici rezultă că E se aflăpe mediatoarea segmentului [BA 0 ], deci CE ⊥ BA 0 . Este însăclarcă BA 0 k AC,prin urmare CE ⊥ AC (fig. 1).Fig. 1 Fig. 2 Fig. 3Soluţia 2 (Sergiu Prisacariu, Cristian Lazăr). Fie F ∈ (BD) astfel încâtCF ⊥ AC; atuncim(\BCF) =90 ◦ − 60 ◦ =30 ◦ şi cum m(\CBF) =75 ◦ , deducemcă m(\CFB) =75 ◦ . Rezultăcă CB = CF, de unde CA = CF, adică 4CAF este1 Profesor, Liceul Teoretic "Dimitrie Cantemir", Iaşi2 Profesor, Colegiul Naţional, Iaşi128


dreptunghic isoscel, cu m( [CAF) =45 ◦ . Astfel, m(\BAF) =60 ◦ − 45 ◦ =15 ◦ ,prinurmare m(\BAF) =15 ◦ şi astfel F = E, de unde concluzia problemei (fig. 2).Soluţia 3 (Enache Pătraşcu). Considerăm punctul S pentru care AB = BS,AB ⊥ BS, iarAB separă C şi S. Cum m(\ABE) =m( [SBE) = 135 ◦ , deducem că4ABE ≡ 4SBE (L.U.L.), de unde AE = SE. Însă m( [SAE) =45 ◦ +15 ◦ =60 ◦ ,deci 4ASE este echilateral. Rezultă că 4ABS ≡ 4ACE (SA = AE, AB = AC,iar m( [SAB) =m( [EAC) =45 ◦ ), prin urmare m( [ACE) =m( [ABS) =90 ◦ (fig. 3).Soluţia 4 ( după oideedatădeCătălin Budeanu). Vom calcula laturile4ACE în funcţie de a = AB. Mai întâi, observăm că, dacă {M} = AD ∩ BC, avemcă AD = AM + MD = a√ 3 a+2 2tg15 ◦ = a√ 3 a+2 2 ¡ 2 − √ 3 ¢ = a ¡ 1+ √ 3 ¢ . Apoi,cum AE este bisectoare în 4ABD, atunci BEED = AB√3 − 1AD = şi, pe de altă parte,22 · AB · AD¡ AE =AB + AD cos 15◦ = 2a2 1+ √ 3 ¢ p √2+ 3a ¡ 2+ √ 3 ¢ · = a √ 2. Folosind relaţia lui2Stewart în 4BCD, obţinem √ că CE 2·BD = BC 2·DE+CD2·BE−BE·DE·BD. Am3 − 1văzut mai sus că BE = · ED; dupăcalculederutină, deducem că CE = a.2În concluzie, CA = CE = a şi AE = a √ 2 şi, din reciprova teoremei lui Pitagora,rezultă că EC ⊥ AC (fig. 4).Fig. 4 Fig. 5Soluţia 5. Fie {N} = AD ∩ CE şi G centrul triunghiului ABC; caînsoluţiaprecedentă, avem că BE√3 − 1ED = ,deunde,folosindproporţiile derivate, obţinem2că DEDB = √ 3−1. Aplicăm teorema lui Menelaus în 4BMD cu transversala E −N −C; deducem că BEED · DNNM · MCCBDN=1, de undeNM =2¡√ 3+1 ¢ .Dupăcalculede129


utină, rezultă că DN = a ¡ 3+ √ 3 ¢, DG = a ¡ 3+2 √ 3 ¢, şi atunci DN32DG = √ 3 − 1.În concluzie, DEDB = DN şi din reciproca teoremei lui Thales obţinem că NE k BG,DGprin urmare NE ⊥ AC, tocmai concluzia problemei (fig. 5).Soluţia 6. Folosim teorema sinusurilor în triunghiurile ABE şi BCD, obţinândBEcăsin 15 ◦ = ABsin 30 ◦ , respectiv BCsin 30 ◦ = BD . Se verifică prin calcul faptul căsin 75◦ sin 15 ◦ sin 30◦=sin 30◦ sin 75 ◦ ,deciBE AB = BCBD . Cum AB = BC şi \CBE ≡ \CBD, urmeazăcă4CBE ∼ 4DBC, prin urmare m(\BCE) =30 ◦ ,deundem( [ACE) =90 ◦ (fig. 4).Soluţia 7. Raportăm planul la un reper cu originea în M, unde M = AD ∩³BC; dacă a = AB, atunciA 0, a√ 3´, B³− a 0´ ³ a 0´2 2 , , C2 , , Dµ0, −a ¡ 2+ √ 3 ¢ .2Observăm că 4EAD este isoscel, deoarece m(\EAD) =m(\EDA)=15 ◦ , prin urmareordonata lui E va fi media aritmetică a ordonatelor punctelor A şi D, adică y E = − a 2 .Cum punctele B, E, D sunt coliniare, avem că y E − y B= x E − x B,deundex E =y D − y B x D − x Ba ¡ 1 − √ 3 ¢.PantadrepteiAC este m AC = y A − y C= − √ 3, iar panta dreptei CE2x A − x Ceste m CE = y C − y E= √ 1 şi, cum m AC · m CE = −1, rezultăcă CE ⊥ AC (fig. 4).x C − x E 3Soluţia 8. Folosim reperul din soluţia precedentă, lucrând însă cunumerecomplexe.Pentru simplitate, vom considera că AB =2;atunciC (1), B (−1), A ¡√ 3i ¢ ,D ¡ − ¡ 2+ √ 3 ¢ i ¢ . Cum BE√3 − 1ED = = k, avemz E = z B + kz D= ¡ 1 − √ 3 ¢ − i.21+kRezultă că z E − z C= i, prin urmare CE ⊥ AC (şi, în plus, CE = AC) (fig. 4).z A − z CSoluţia 9. Din BE√3 − 1ED = ,obţinem că −→ −→√CB +3−1−−→CE =2 CD21+ √ =3−12= 2 −→ ¡√ ¢ −−→CB + 3 − 1 CD −→ −→ 1³√ , deci CE · CA = √ 2 −→ −→ ¡√ ¢ −−→ −→CB · CA + 3 − 1 CD · CA´.3+13+1Însă −→ −→ CB · CA = CB · CA · cos 60 ◦ = a22 , −−→ CD · −→ ³ −→ −→CA = CA + AD´· −→ CA = CA 2 −√3+1AD · AC · cos \CAD = − a 2 −→ −→.Astfel, CE · CA =0, de unde CE ⊥ CA (fig. 4).2Notă. Soluţia 6 sugerează următoarea extindere:Se consideră 4ABC isoscel ( AB = BC) şi triunghiul BCD cu A, D în semiplaneopuse faţă deBC, iarm(\ABC) =2m(\BDC). Dacă E ∈ (BD) are proprietateacăsin αsin (α + β + x) = sin x 2sin ¡ ¢,β + x undeα = m(\BAE), β = m(\CBD) şi2x = m(\ABC), atunciCE ⊥ AC.Problema G133 se obţine pentru α =15 ◦ , β =75 ◦ şi x =60 ◦ .130


Sur les matrices magiquesAdrien REISNER 1Toutes les matrices considérées ici appartiennent à l’espace vectoriel réel M 3 (R).Cet espace est de dimension 9. Nous nous proposons d’étudier certaines propriétésde l’ensemble des matrices 3 × 3 dites magiques dont la définition est la suivante:Définition. Une matrice A =(a ij ) ∈ M 3 (R) est dite magique si les huit sommesa i1 + a i2 + a i3 , a 1j + a 2j + a 3j , a 11 + a 22 + a 33 , a 31 + a 22 + a 13 sont égales pouri, j :1, 2, 3. Onappelerasomme magique cette somme commune.Considérons les trois matrices suivantes (évidemment magiques):⎛L = ⎝ 1 1 1 ⎞ ⎛⎞−1 2 −11 1 1⎠ , M = ⎝ 0 0 0 ⎠ , N = M T .1 1 11 −2 1Proposition 1. Une matrice quelconque A est somme d’une matrice symétriqueA 0 et d’une matrice antisymétrique A 00 , la décomposition A = A 0 + A 00 étant unique.Démonstration. On a de façon unique: A = A 0 + A 00 ,oùA 0 = 1 ¡ ¢ A + ATet2A 00 = 1 ¡ ¢ A − AT; A 0 est symétrique et A 00 est antisymétrique (i.e. A 00T = −A 00 ).2Proposition 2. Lasommededeuxmatricesmagiquesestunematricemagique.La transposée d’une matrice magique est magique. Enfin le produit d’une matricemaqique et d’un scalaire est une matrice magique. Si A est une matrice magique lesdeux matrices A 0 et A 00 définies plus haut sont elles mêmes magiques.On se propose ⎛ de construire toutes les matrices magiques antisymétriques. Eneffet, si A 00 = ⎝ 0 −γ β ⎞γ 0 −α⎠, alors: −γ + β = −α + γ = −β + α = γ − β =−β α 0⎛α − γ = β − α =0. On en déduit la solution générale: A 00 = λ ⎝ 0 1 −1 ⎞−1 0 1 ⎠, où1 −1 0λ est un scalaire arbitraire.On se propose de construire toutes les matrices magiques symétriques. De mêmeque précédemment toutes les matrices symétriques A 0 1 =(a⎛ ij ) vérifiant: s =trA 0 1 =a 11 + a 22 + a 33 =0sont de la forme suivante: A 0 1 = µ ⎝ 1 −1 0 ⎞−1 0 1 ⎠, oùµ est0 1 −1un scalaire arbitraire. Si s 6= 0, la forme générale des matrices symétriques magiquess’obtient en ajoutant s 3 aux éléments a ij de la matrice précédente. On en déduit lasolution générale:⎛A 0 = µ ⎝ 1 −1 0 ⎞ ⎛−1 0 1 ⎠ + ν ⎝ 1 1 1 ⎞1 1 1⎠ ,0 1 −1 1 1 11 Centre de Calcul E.N.S.T., Paris; e-mail: adrien.reisner@enst.fr131


où µ, ν sont des réels arbitraires. Compte tenu de la Proposition 1, on obtientimmédiatement la forme générale des matrices magiques:⎛A = λ ⎝ 0 1 −1 ⎞ ⎛−1 0 1 ⎠ + µ ⎝ 1 −1 0 ⎞ ⎛−1 0 1 ⎠ + ν ⎝ 1 1 1 ⎞1 1 1⎠ =1 −1 0 0 1 −1 1 1 1= 1 2 (λ − µ) M − 1 (λ + µ) N + νL.2Les trois matrices M, N et L étant linéairement indépendentes on a, donc, leThéorème 3. L’ensemble des matrices 3×3 magiques est un sous-espace vectorielde M 3 (R). Ce sous-espace vectoriel a pour dimension 3, unebaseétantforméeparles trois matrices L, M et N.Etant donné les deux matrices A = αM + βN + γL et B = α 0 M + β 0 N + γ 0 Lnous avons:Proposition 4. Le produit AB est une matrice magique si et seulement si on aαβ 0 = βα 0 =0. La matrice L est la seule matrice (à un facteur scalaire près) quisoit magique et produit de deux matrices magiques.Démonstration. Les relations évidentes: M 2 = N 2 = ML = LM = NL =LN =0, L 2 =3L et MN + NM =12I − 4L conduisent immédiatement à:AB =(αM + βN + γL) ¡ α 0 M + β 0 N + γ 0 L ¢ =⎛=3γγ 0 L + ⎝ 2βα0 +6αβ 0 −4βα 0 2βα 0 − 6αβ 0 ⎞−4βα 0 8βα 0 −4βα 0 ⎠2βα 0 − 6αβ 0 −4βα 0 2βα 0 +6αβ 0et par suite à l’équivalence: AB est une matrice magique ⇔ αβ 0 = βα 0 =0. Onobtient les quatre cas suivants:• α = β =0 ⇒ λL ¡ α 0 M + β 0 N + γ 0 L ¢ =3γγ 0 L (1)• α = α 0 =0 ⇒ (βN + γL) ¡ β 0 N + γ 0 L ¢ =3γγ 0 L (2)• β = β 0 =0 ⇒ (αM + γL)(α 0 M + γ 0 L)=3γγ 0 L (3)• α 0 = β 0 =0 ⇒ γ 0 L (αM + βN + γL) =3γγ 0 L (4)Les deux cas (1), (4) n’en forment qu’un seul. Les cas (2), (3) sont identiques àun échange de matrices près. On en déduit immédiatement la deuxième partie de laProposition 4.Proposition 5. Le produit d’une matrice magique par une combinaison linéairede L et I est une matrice magique.Démonstration. De façon évidente la matrice:(αM + βN + γL)(α 0 I + γ 0 L)=αα 0 M + βα 0 N +(γα 0 +3γγ 0 ) Lest magique.Proposition 6. A = αM + βN + γL est inversible si et seulement si αβγ 6= 0.De plus, dans le cas où αβγ 6= 0la matrice A −1 est elle même magique.Démonstration. La proposition est immédiate puisque det (αM + βN + γL) =−36αβγ. Dans le cas où αβγ 6= 0il vient: A −1 = 1 µ 336 β M + 3 α N + 4 γ L qui estbien une matrice magique.132


Cettepropositionsegénéraliseaveclethéorèmesuivant:Théorème 7. Soit A = αM + βN + γL une matrice magique. Alors, pour toutn ∈ N la matrice A 2n+1 est magique. Si αβγ 6= 0, pour tout p ∈ N la matriceA −(2p+1) est magique.Démonstration. Avec A = αM+βN+γL on obtient A 2 =12αβI+ ¡ 3γ 2 −4αβ ¢ Let la Proposition 5 permet alors par une récurrence évidente sur n ∈ N de démontrerles implications suivantes: A 2n−1 est une matrice magique ⇒ A 2n = kI + k 0 L ⇒A 2n +1 est une matrice magique. On conclut alors grâce à la proposition précédentepuisque A −(2p+1) = ¡ A −1¢ 2p+1.Remarques. Mis à part le cas où αβ =0— voir la Propozition 4 — les matricesA 2n ne sont pas magiques.Si αβ =0les matrices A et A n =3 n−1 γ n L avec n>1 sont magiques.Si αβ 6= 0on obtient immédiatement par récurrence sur n ∈ N:A 2n =(12αβ) n I + 1 h(3γ) 2n − (12αβ) ni L3n’est pas magique.A 2n+1 =(12αβ) n (αM + βN)+γ (3γ) 2n Lest une matrice magique.Si αβγ 6= 0ces mêmes formules sont vérifiées aussi pour n


Concursul de matematică “Al. Myller”Ediţia a VI-a, Iaşi, martie 2008Clasa a VII-a1. Numerele reale distincte x, y, z au propietatea că x 3 − x = y 3 − y = z 3 − z.Să searatecă x + y + z =0.2. a) Să searatecă, dintre cinci numere naturale oarecare, se pot alege treinumere cu suma divizibilă cu3.b) Să searatecă, dintre 17 numere naturale oarecare, se pot alege nouă numerecu suma divizibilă cu9.3. FieAD înălţimea triunghiului ascuţitunghic ABC. Considerăm mulţimea Ma punctelor X ∈ (AD) cu proprietatea că \ABX = \ACX.a) Să searatecămulţimea M este nevidă.b) Dacă M conţine cel puţin două elemente, să se demonstreze că mulţimea Mconţine o infinitate de elemente.Cristian Lazăr4. Fie segmentul AB şi semidrepta (Ox, unde O ∈ (AB) şi A, B /∈ (Ox. Perpendiculareleîn A şi B pe dreapta AB intersectează bisectoarele (Oy şi (Oz aleunghiurilor [AOx şi [BOx în punctele M, respectivN. Perpendiculara din A pe (Oyintersectează perpendiculara din B pe (Oz în punctul P .Săsearatecă punctele M,N, P sunt coliniare.Mircea FianuClasa a VIII-a1. Considerăm cubul ABCDA 0 B 0 C 0 D 0 şi M, N, P mijloacele muchiilor AB, AD,respectiv AA 0 .Să se determine măsura unghiului dintre dreapta A 0 C 0 şi dreapta deintersecţie a planelor (MNP) şi (BCC 0 ).2. Fie a, b numere întregi distincte cu proprietatea că există n număr real astfelîncât a 3 − a = b 3 − b = n. Săsearatecă n =0.3. Se dau şase puncte în plan, oricare trei necoliniare. Considerăm zece segmente,fiecare având capetele în câte două dintre aceste puncte. Să searatecăexistă celpuţin un triunghi având ca laturi trei dintre cele zece de segmente.4. Fie SABC un tetraedru regulat. Punctele A 1 , B 1 , C 1 aparţin muchiilor (SA),(SB), (SC), respectiv, astfel încât A 1 B 1 = B 1 C 1 = C 1 A 1 . Săsearatecă planele(A 1 B 1 C 1 ) şi (ABC) sunt paralele.Clasa a IX-a[x]1. Determinaţi numărul soluţiilor ecuaţiei{x} = 2007x2008 . Mihail Bălună2. Rezolvaţi în mulţimea numerelor întregi ecuaţia x 6 + x 5 +4=y 2 .Ioan Cucurezeanu134


3. Fie ABCDE un pentagon convex. Demonstraţi căaria(ABC)aria(ABCD) + aria(CDE)aria(BCDE) < 1.Dan Ismailescu4. Fie C 1 , C 2 două cercuri concentrice distincte şi [AB] un diametru al cerculuiC 1 .Considerăm două puncte variabile M ∈ C 1 , N ∈ C 2 , nesituate pe dreapta AB.a) Arătaţi că există şi sunt unic determinate punctele P , Q, situate pe drepteleMA, respectiv <strong>MB</strong>,astfelîncâtN să fie mijlocul segmentului [PQ].b) Arătaţi că sumaAP 2 + BQ 2 este constantă, unde P , Q sunt definite la a).Mihai Piticari, Mihail BălunăClasa a X-a1. Fie O centrul cercului circumscris triunghiului ABC şi A 1 punctul de pe cercdiametral opus lui A. Notăm cu G, G 1 centrele de greutate al triunghiurilor ABC şiA 1 BC şi cu P intersecţia dreptelor AG 1 şi OG. Săsearatecă PGPO = 2 3 .Gabriel Popa, Paul Georgescu2. Săsearatecănuexistănumereîntregia, b, c astfel încât (a+bi √ 3) 17 = c+i √ 3.Dorin Andrica, Mihai Piticari3. Să se determine poligoanele convexe, inscriptibile, cu proprietatea că oricetriunghi determinat de trei dintre vârfurile acestora este isoscel.Gheorghe Iurea4. Fie r un număr real cu proprietatea că ¡ 2 n r − 1 4 , 2n r + 4¢ 1 ∩ Z 6= ∅, pentruorice n ∈ N. Săsearatecă r este număr întreg.Ciprian BaghiuClasa a XI-a1. Fie A ∈ M 4 (R) astfel încât det(A 2 − I 4 ) < 0. Săsearatecăexistă α ∈ R, cu|α| < 1, astfel încât matricea A + αI 4 să fie singulară.Mihai Haivas2. Fie A, B, S ∈ M 3 (C), S fiind o matrice nesingulară încâtB = S −1 AS. Săsearate că tr(B 2 )+2tr(B ∗ )=(tr(A)) 2 .Mihai Haivas3. Fie a>1 un număr real. Pentru fiecare număr natural nenul n, k(n) este celmai mic număr natural k pentru care (n +1) k ≥ an k k(n).Săsecalculeze limn→∞ n .Neculai Hârţan4. Fie f : R → R o funcţie continuă peQ, cu proprietatea că f(x)


2. Determinaţi numerele n ∈ N, n ≥ 3 şi a ∈ R pentru care polinomul X n +aX−1are un divizor de foma X 2 + αX + β cu α, β ∈ Z.Mihail Bălună1R3. Determinaţi funcţiile crescătoare f :[0, 1] → R pentru care¯ f (x) e nx dx¯ ≤0≤ 2008, pentruoricen ∈ N.Mihai Piticari4. Fie A un inel finit în care numărul elementelor inversabile este egal cu numărulelementelor nilpotente. Să searatecănumărul elementelor inelului este o putere alui 2. (Un element x ∈ A se numeste nilpotent dacă există k natural cu x k =0.)Dinu ŞerbănescuConcursul de matematică “Florica T. Câmpan”Etapa judeţeană, 16-17 februarie 2008Clasa a IV-a1. a) Găsiţi regula de formare a şirului3,8,13,... şi scrieţi termenul de pe locul31.b) După un concurs de matematică, un elev nu şi-a amintit rezultatul unei probleme.Totuşi, şi-a adus aminte că numărul are şase cifre, începe cu 1 şi dacă primacifră semutălasfârşit, atunci numărul obţinut va fi de trei ori mai mare decât celiniţial. Care a fost rezultatul problemei?2. Mergândcumaşina, un şofer observă la ora 09:10 că pekilometrajuldelabord apare numărul 12921. La ora 11:00, pe kilometraj apare următorul număr carecoincide cu răsturnatul său. La ce oră vaobservaşoferuldinnouunastfeldenumăr,presupunând că sedeplaseazăcuviteză constantă?Gabriel Mîrşanu, Recreaţii <strong>Matematice</strong> 1/20013. Pe o foaie este scris numărul A = 1234xy. Cinci elevi joacă următorul joc:fiecare dintre primii patru citeşte numărul, îşi fixează câte o regulă de transformarealuişi scrie pe tablă numărul trans<strong>format</strong>. Al cincilea, care cunoaşte doar primelepatru cifre ale lui A, trebuie să ghiceascăregulafiecăruia dintre colegi şi să încercesă aflenumărul. Ştiind că primii patru au scris pe tablă numerele 123500, 123470,123460, 120000, se cere:a) Care sunt regulile de transformare observate de al cincilea elev?b) Poate al cincilea elev să aflecuexactitatenumărul? Care sunt valorile posibileale numărului A?Petru AsafteiClasa a V-a1. La un concurs se acordă cinci puncte pentru premiul I, trei puncte pentru aldoilea şi două puncte pentru al treilea. Aflaţi numărul de premii primite de eleviiunei şcoli, ştiind că auobţinut în total 25 de puncte şi cel puţin cîte două premiidinfiecare categorie.2. Un număr natural se numeşte simpatic dacă este <strong>format</strong> din cifre distinctenenule, a căror sumă se divide cu 10.136


a) Determinaţi cel mai mic şi cel mai mare număr simpatic.b) Precizaţi câte numere de trei cifre sunt simpatice şi divizibile cu 4.3. Dacă a ∈ N ∗ şi b ∈ N ∗ ,notăm a∗b = a b +b a (de exemplu, 3∗2 =3 2 +2 3 =17).a) Determinaţi numărul n ∈ N ∗ astfel încât 1 ∗ 1+2∗ 1+3∗ 1+···+ n ∗ 1=54.b) Comparaţi numerele 3 ∗ 18 şi 2 ∗ 27.c) Aflaţi ultima cifră anumărului 2 ∗ (2 ∗ 2008).Adrian ZanoschiClasa a VI-a1. Un părinte îşi împarte averea astfel: primul copil primeşte 10 000 lei pluso cincime din rest; al doilea copil primeşte 20 000 lei plus o cincime din noul rest;al treilea copil primeşte 30 000 lei plus o cincime din noul rest şi aşa mai departe.Să seaflesumaîmpărţită depărinte, precum şi numărul copiilor, ştiind că toţi aumoşteniri egale.Mihai Gârtan, Recreaţii <strong>Matematice</strong> 1/20022. Cătălin este faianţar şi trebuie să paveze podeaua unei încăperi în formă dedreptunghi având lungimea de 3 metri şi lăţimea de 2 metri, folosind dale pătraticecu latura de 50 centimetri. El are la dispoziţie 6 dale roşii, 6 albastre, 6 galbene şi 6verzi, iar cerinţa este ca orice două dale de aceeaşi culoare să nuseatingă.a) Indicaţiunexempludepavarecorectă.b) Cătălin sparge o dală roşie şi primeşte în loc una verde. Poate acum procedaîn aşa fel încât să respecte cerinţa? Justificaţi răspunsul.Doru Buzac3. Un număr natural N se scrie în baza 10 folosind 6 cifre nenule şi distincte.Se ştie că, oricum am schimba ordinea cifrelor numărului N, numărul N precum şinumerele obţinute sunt toate multipli de p, unde p este un număr prim.a) Determinaţi câte numere se pot obţine din N prin schimbarea ordinii cifrelor.b) Dacă p =3,daţi exemplu de un număr N care să verifice condiţiile din enunţ.c) Dacă p 6= 3, demonstraţi că nuexistănumereN care să verificecondiţiile dinenunţ.Radu SavaClasa a VII-a1. Se consideră şirurile definite prin: a 1 = 91204; a 2 = 9012004; a 3 = 900120004;... ; b 1 = 91504; b 2 = 9015004; b 3 = 900150004; ...a) Aflaţi numărul de cifre al sumei a n + b n , unde n ∈ N ∗ ;b) Arătaţi că √ a n + b n /∈ Q, ∀n ∈ N ∗ ;c) Demonstraţi că √ a n ∈ Q, ∀n ∈ N ∗ ,însă √ b n /∈ Q, ∀n ∈ N ∗ .Constantin Chirilă, Recreaţii <strong>Matematice</strong> 1/20012. Două blocuri de locuinţe care au înălţimea de 21 m fiecare sunt situate peun teren plat. La ora 10, umbra primului bloc proiectată pe cel de-al doilea areînălţimea de 15 m, iar umbra celui de-al doilea bloc pe pământ are lungimea de 42m. Ce înălţime are umbra primului bloc pe cel de-al doilea la ora 11, dacă umbracelui de-al doilea bloc pe pământ este de 31,50 m?3. Pardoseala unei băi de dimensiuni L =45dm, l =35dm este acoperităcuplăcide gresie în formă depătrat cu latura de 1 dm. Dacă seîndepărtează câteoplăcuţă137


din cele patru colţuri, se poate acoperi suprafaţa rămasă cuplăci dreptunghiulare dedimensiuni 1 dm × 2 dm?Clasa a VIII-a1. a) Fie suma1S = p √1+ 12 − 1 + 1p √3+ 32 − 1 + ···+ 1p √2007 + 20072 − 1 .Aflaţi cel mai mic număr natural nenul n pentru care numărul S · √n este natural.b) Dacă a este lungimea ipotenuzei şi b, c lungimile catetelor unui triunghi dreptunghic,demonstraţi că 2a >b+ c + h a , unde h a este lungimea înălţimii corespunzătoareipotenuzei.Claudiu Ştefan Popa2. Câte plane pot fi duse la egală distanţă de patru puncte necoplanare date?Justificaţi răspunsul.3. În cetatea NN a numerelor naturale se organizează o mare petrecere în cinsteanumărului 0. La poarta castelului bate unul din locuitorii cetăţii.— Sunt numărul 83. Îmipermiteţi să intru la petrecere? întreabă acesta.— La petrecere sunt invitate doar numerele fantastice, îi răspunse o voce de parteacealaltă.— Dar ce înseamnă număr fantastic? întreabă numărul 83.—Săvă explic, spune vocea stranie. Dacă n este un număr natural mai maredecât 1 şi notăm A n = {x ∈ N | (x, n) 6= 1}, numărul n se numeşte fantastic dacăpentru orice două numerex, y aparţinând mulţimii A n ,sumalorx + y este tot unelement al mulţimii A n .Aţi priceput?—Amînţeles, răspunde lămurit vizitatorul.a) Stabiliţi voi dacă numărul 83 este invitat la petrecere. Aceeaşi cerinţă şi pentrunumărul 2008.b) Găsiţi toate numerele pare invitate la petrecere.Alexandru NegrescuEtapa interjudeţeană, 22-23 martie 2008Clasa a IV-a1. George este mai mic decât Andrei cu o pătrime din vârsta lui Andrei. Pesteun an, Andrei va fi mai mare decât George cu o pătrime din vârsta lui George. Cevârstă au acum Andrei şi George?2. Se consideră împărţirea:a) Daţi un exemplu de astfel de împărţire.b) Câte împărţiri de acest tip se pot efectua? Justificaţirăspunsul!3. Un elev de clasa a IV-a are în total 100 de fructe,nuci şi mere. El schimbă cu un prieten câte nouă nucipentru două mere, terminând toate nucile după unnumărde schimburi şi rămânând în final cu 44 de mere.a) Câte nuci a avut iniţial elevul?∗ ∗ ∗ ∗ ∗ ∗∗ ∗ ∗ ∗ 2 1== ∗ ∗∗ ∗== ∗∗=b) Câte schimburi s-au făcut şi câte mere a primit de la prietenul său?138


Clasa a V-a1. Un număr se numeşte fiul unui alt număr dacă este <strong>format</strong> cu două dintrecifrele numărului iniţial, numit tată. Dintre numerele de trei cifre cu ultima cifră 0,aflaţi toţi taţii cu 891 mai mari decât unul dintre fiii lor.2. Se consideră următorul tablou cu 200 de linii.a) Ce număr se află în mijlocul ultimei linii atabloului?b) Câte numere conţine tabloul?c) De câte ori apare numărul 100 în acest tablou?22 4 22 4 6 4 22 4 6 8 6 4 2...........................3. La concursul "Florica T. Câmpan", etapa locală, au luat parte toţi elevii declasaaV-adintr-oşcoală. Elevii din clasa a V-a D au obţinut următoarele rezultate:prima problemă au rezolvat-o 9 elevi, a doua problemă au rezolvat-o 7 elevi, a treiaproblemă au rezolvat-o 5 elevi, a patra problemă au rezolvat-o 3 elevi, iar a cinceaproblemă a rezolvat-o un singur elev. Toţi elevii clasei, în afarădePetrică, au rezolvatacelaşi număr de probleme, în timp ce Petrică a rezolvat cu una mai mult decît colegiisăi. Poate să fie el premiant al concursului, dacă premianţii concursului au fost eleviicare au rezolvat 4 sau 5 probleme?Clasa a VI-a½ 20081. Se consideră mulţimea A =7 , 20098 , 2010 ¾9 ,... . Determinaţi cardinalulmulţimii A ∩ N.2. Fie dreapta AB, O un punct între A şi B şi, de aceeaşi parte a dreptei,semidreptele [OA 1 , [OA 2 ,..., [OA n ,înaceastă ordine, astfel încât m( \AOA 1 )=a,m( A\1 OA 2 )=a +2,..., m( \A n OB) =a +2n, undea ∈ N, n ∈ N, n ≥ 2. Determinaţinumărul unghiurilor şi măsura fiecăruia dintre ele.3. a) Arătaţi că, oricum am alege cinci numere naturale, există printre ele treicu suma divizibilă cu3.b) Arătaţi că, oricum am alege 25 de numere naturale, există printre ele nouă cusuma divizibilă cu9.Clasa a VII-a1. a) Se consideră numerele 1, 3 − √ 2, 3+ √ 2 şi 5. După un pas, fiecare numărse înlocuieşte cu media aritmetică a celorlaltor trei. Este posibil ca, după unnumărde paşi, să obţinem numerele 5 − 2 √ 2, 3, 3+2 √ 2 şi 2?b) Să searatecădacă numerele a, b şi √ a + √ b sunt raţionale, atunci √ a şi √ bsunt raţionale.2. Fie ∆ABC, AB < AC şi D ∈ (AC). Fie AE bisectoarea unghiului \BAC,E ∈ (BD), F mijlocul lui [AD], {O} = AE ∩ BF, {G} = DO ∩ AB. SăsearatecăGD k BC ⇔ AB = CD.Daniela Tamaş, Recreaţii <strong>Matematice</strong> 2/20063. Ionel şi Gigel au trasat cu creta pe parchet, în două colţuri diferite ale uneicamere, câte un segment de dreaptă cu capetele la marginea pereţilor, obţinând139


două triunghiuri dreptunghice. Dacă celedouă triunghiuri au aceeaşi arie şi acelaşiperimetru, să searatecă ele sunt congruente.Clasa a VIII-a1. Să se afle valoarea fracţiei x + y , ştiind că 0


a) Arătaţi că funcţia F :(0, ∞) → R, F (x) =2 √ x (ln x − 2), este o primitivăpentru funcţia f.b) Demonstraţi că orice primitivă G afuncţiei f este crescătoare pe [1, ∞).c) Aflaţi aria suprafeţei cuprinse între graficul lui f, axaOx şi dreptele de ecuaţiix = 1 şi x = e.e Subiect propus pentru Bacalaureat-2008Problema 2. Fie (A, +, ·) un inel de caracteristică 3(3x =0, ∀x ∈ A ) şi a ∈ Aun element oarecare. Fie M = © x ∈ A | x 3 = ax = xa ª şi b ∈ M cu proprietatea căb comută cu orice element din A.a) Demonstraţi că b − x ∈ M, oricare ar fi x ∈ M.b) Să searatecă, dacă a ∈ A are în plus proprietatea că există n ∈ N ∗ pentrucare a n =1, atunci x +(b − x) 2n+1 ∈ M, oricare ar fi x ∈ M.Ion Bursuc, SuceavaProblema 3. Dacă f :[0, 1] → R este funcţia continuă, atunciZ 10x (f (x)+f (1 − x)) dx =Z 10f (x) dx.Dumitru Crăciun, FălticeniProblema 4. Se consideră numerele prime distincte p şi q, astfelîncâtp−q =2r,cu r ≥ 3 prim.a) Arătaţi că p − q | p q − q p dacă şi numai dacă p − q | q p−q − 1.b) Demonstraţi că p − q | p q − q p dacă şi numai dacă q 2 ≡ 1 (mod (p − q)).Cătălin Ţigăeru, SuceavaPrima problemă fiind una cunoscută, prezentăm doar soluţiile celorlaltor trei.Problema 2. a) Observăm că (α − x) 3 = α 3 − 3α 2 x +3αx 2 − x 3 = α 3 − x 3 =α 3 − ax. Cum α ∈ M, atunci α 3 = αa = aα, deundededucemcă (α − x) 3 =aα − ax = αa − xa = a (α − x) =(α − x) a.b) Demonstrăm prin inducţie că x 2m+1 = a m x, pentru orice x ∈ M şi m ∈ N ∗ .Pentru m =1,relaţia este adevărată. Presupunem că, dacă x ∈ M, atunci x 2k−1 =a k−1 x; atunci x 2k+1 = x 2k−1 x 2 = a k−1 xx 2 = a k−1 x 3 = a k−1 ax = a k x, ceea ceîncheie demonstaţia afirmaţiei iniţiale.Deoarece α−x ∈ M, atunci pentru orice număr m ∈ N ∗ ,rezultăcă (α − x) 2m+1 =a m (α − x). Luândm = n,obţinem că (α − x) 2n+1 = α−x, de unde x+(α − x) 2n+1 =α ∈ M, ceea ce trebuia demonstrat.RProblema 3. Făcănd o schimbare de variabilă, se arată că bf (x) dx =aR= bf (a + b − x) dx, ∀f :[a, b] → R continuă. Aplicând de două ori acest rezultat,obţinem că 1 RaRf (x) dx = 1 Rf (1 − x) dx, respectiv 1 x (f (x)+f (1 − x)) dx =001RR(1 − x)(f (x)+f (1 − x)) dx, de unde 2 1 Rx (f (x)+f (1 − x)) dx = 1 f (1 − x) dx +0014100


1RRf (x) dx. Deducem că 2 1 Rx (f (x)+f (1 − x)) dx =2 1 f (x) dx, adicătocmaiconcluziaproblemei.000Problema 4. a) Putem scrie că p q − q p =(p − q) ¡ p q−1 + qp q−2 + ···+ q q−1¢ −q q (q p−q − 1); deoarece (p − q, q q )=1, deducem că p − q | p q − q p dacă şi numai dacăp − q | q p−q − 1.b) Rezultatul de la punctul a) se reformulează astfel: p − q | p q − q p dacă şi numaidacă q p−q ≡ 1 (mod (p − q)). Dar (p − q, q) =1,deciq ϕ(p−q) ≡ 1 (mod (p − q))(teorema lui Euler); deoarece ϕ (p − q) =ϕ (2r) =r − 1, deducem că q r−1 ≡ 1(mod (p − q)) şi q 2r ≡ 1 (mod (p − q)). Dar (r − 1, 2r) =2, deci p − q | p q − q p dacăşi numai dacă q 2 ≡ 1 (mod (p − q)).Particularizând, se pot obţine o serie de probleme interesante; iată câteva exemple:34 | 67 101 − 101 67 , 34 | 137 103 − 103 137 , 22 | 881 859 − 859 881 , 6 | 2011 2017 − 2017 2011 .1. La data de 15 ianuarie 1883 a apărut primul număr al revistei Recreaţii Ştiinţifice(1883—1888). În anul acesta, 2008, se împlinesc 125 de ani de la apariţiaacesteia.Scrieţi 2008 folosind numărul 125 şi (numai) operaţiile de adunare şi împărţire!Care este numărulmaximdeoperaţii cu care puteţi face această scriere? Dar celminim? (Nu se acceptă termeni nuli!)2. Se dau trei cifre şi un rezultat. Indicaţi operaţiile necesare pentru a restabiliegalitatea!1 1 1 = 82 2 2 = 83 3 3 = 84 4 4 = 85 5 5 = 86 6 6 = 87 7 7 = 88 8 8 = 89 9 9 = 8.Notă. Răspunsurile pot fi găsite la pag. 160.142


Soluţiile problemelor propuse în nr. 2 / 2007Clasele primareP.134. De la apartamentul meu cobor 7 etaje, apoi urc 4 etaje şi observ că suntla etajul 9. La ce etaj locuiesc?(Clasa I )Dragoş Iacob,elev,IaşiSoluţie. 9+(7− 4)=9+3=12.Eulocuiesclaetajul12.P.135. În trei vase sunt 36 nuci. Dacă din primul vas se iau 3 nuci şi din altreilea o nucă şi se pun în al doilea vas, atunci în fiecare vas va fi acelaşi număr denuci. Câte nuci au fost la început în fiecare vas?(Clasa I )Înv. Rica Bucătariu, IaşiSoluţie. Din 36 = 12 + 12 + 12 obţinem: în primul vas erau 12 + 3 = 15 nuci; înal doilea vas 12 − 3 − 1=8nuci, iar în al treilea vas 12 + 1 = 13 nuci.P.136. Aflaţi vârsta tatălui meu, ştiind că esteunnumăr cuprins între 35 şi 41,dublul lui între 73 şi 77, iar triplul lui este cuprins între 112 şi 118.(ClasaaII-a)Iurie Juc, elev, IaşiSoluţie. Numerele cuprinse între 35 şi 41 sunt 36, 37, 38, 39 şi 40. Numărul 36nu verifică adouacondiţie, iar numărul 37 nu verifică a treia condiţie. Numărul 38verifică toate condiţiile, iar dacă vârstaestemaimareca38, atunci dublul vârsteidepăşeşte pe 77. Tatăl are 38 ani.P.137. Dorin, Oana şi Claudia se pregătesc pentru Concursul "Florica T. Câmpan".Oana a rezolvat 5 probleme. Dorin a rezolvat un număr de probleme în plusfaţă de Oana, egal cu numărul de probleme rezolvate în plus de Oana faţă deClaudia.Câte probleme au rezolvat împreună cei trei copii?(ClasaaII-a)Inst. Maria Racu, IaşiSoluţie. Cei trei copii au rezolvat împreună 5+(5+a)+(5− a) =15probleme.P.138. Doi taţi şi trei fii au împuşcat fiecare câte un iepure. Când i-au numărat,au văzut că au doar patru iepuri. De ce?(Clasa a III-a)Inst. Elena Niţă, IaşiSoluţie. Echipa de vânători este <strong>format</strong>ă din doi feciori, tatăl lor şi bunicul celordoi feciori.P.139. Mutaţi un singur chibrit pentru a obţine o egalitate:(Clasa a III-a)Nicolae Ivăşchescu, CraiovaSoluţie. Mutăm chibritul orizontal de la semnul de operaţie plus şi obţinem:P.140. Descoperă regula de completare a jetoanelor10011112213 14 ...98729981143


şi calculează câtenumerediferitesuntscrisepeacestejetoanepelocuriledejos.(Clasa a III-a)Lenuţa Zaharia, elevă, IaşiSoluţie. Regula este dată de: 1 × 0=0, 1 × 1=1,..., 9 × 9=81. Numerelediferite scrise pe locurile de jos sunt generate de produsele:1 × 0, 1 × 1, 1 × 2,...,1 × 9 10 numere2 × 5, 2 × 6,...,2 × 9 5numere3 × 5, 3 × 7, 3 × 8, 3 × 9 4numere4 × 5, 4 × 7, 4 × 8, 4 × 9 4numere5 × 5, 5 × 6,...,5 × 9 5numere6 × 7, 6 × 8, 6 × 9 3numere7 × 7, 7 × 8, 7 × 9 3numere8 × 8, 8 × 9 2numere9 × 9 1număr.În total sunt scrise 37 numere diferite.P.141. Fiul observă că, atunci când îi mai trebuia un an până lajumătateavârstei din prezent, tatăl avea vârsta de 12 ori mai mare decât a sa, iar când va avea11 ani, vârsta lui va fi de 4 ori mai mică decât a tatălui. Să se afle vârsta fiului înprezent.(ClasaaIV-a)Petru Asaftei, IaşiSoluţie. Diferenţa dintre vârsta tatălui şi vârsta fiului este 4 × 11 − 11 = 33 ani.Figurarea mărimilor din problemă:(*) — vârsta fiului în prezent(**) — vârsta fiului când mai avea un an până lajumătatea vârstei din prezent.Valoarea unui segment este 33 : 11 = 3 ani. Vârsta fiului în prezent este (3 + 1) ×2=8ani.P.142. Paginile unei cărţi sunt numerotate de la 1 la 336. Din această carteserup, la întâmplare, 111 foi. Să searatecă:a) sumanumerelordepefoilerămase nu se împarte exact la 10;b) produsul numerelor de pe foile rămase se împarte exact la 3.(ClasaaIV-a)Maria Frangoi, elevă, IaşiSoluţie. a) În total cartea are 336 : 2 = 168 foi. Numărul foilor rămase este168 − 111 = 57. Suma numerelor de pe fiecare foaie este un număr impar. Sumaunui numărimpardenumereimpareesteunnumăr impar, deci nu se poate împărţiexact la 10.b) În şirul 1, 2, 3,..., 336 avem 112 numere care se împart exact la 3. Acesteasunt: 1 × 3, 2 × 3, 3 × 3,..., 112 × 3. Pe paginile celor 111 foi putem avea cel mult111 numere care se împart exact la 3. Pe paginile rămase vom întâlni cel puţin unnumăr care se împarte exact la 3, decişi produsul numerelor se împarte exact la 3.144


P.143. Aşezaţi numerele 2, 3, 4, . . . , 10 în pătratul alăturat astfelîncât, pe fiecare linie, suma numerelor din primele două casetesăfieegală cunumărul din ultima casetă. În câte moduri pot fi aşezate acestenumere?(ClasaaIV-a)Ionela Bărăgan, elevă, IaşiSoluţie. Suma numerelor de pe primele două coloane este egală cu2 6 8suma numerelor de pe ultima coloană. Suma tuturor numerelor este54, deci suma numerelor de pe ultima coloanăeste27. Singurasituaţie4 5 9care satisface condiţia de pe ultima coloană este8+9+10=27.Un3 7 10exemplu de aşezare este prezentat alăturat. Numerele 8, 9 şi 10 pot fi aşezate în3 × 2 × 1=6moduri pe ultima coloană. Deoarece⎧⎧⎨⎨⎩2+6=6+2=84+5=5+4=93+7=7+3=10şi⎩3+5=5+3=82+7=7+2=94+6=6+4=10înseamnă căavem(8 × 2) × 6=96moduri de aşezare a celor 9 numere.Clasa a V-aV.81. Demonstraţi că putem completa cu numere naturale într-o infinitate demoduri căsuţele libere din figura de mai jos, astfel încât să sepoată efectua corectoperaţiile indicate:,Soluţie.Amalia Cantemir, elevă, IaşiNotă. Explicaţia este aceea că "schema" se poate descompune în câteva cicluri"nule", în sensul că suma numerelor care sunt scrise pe traseu şi care se adună esteegală cu suma numerelor care se scad:I II IIIV.82. Într-o fermă suntgăini, oi şi vaci, în total 324 de picioare şi un numărimpar de capete:145


a) Să searatecăînfermănupotfi101 găini.b) Să searatecănumărul oilor nu poate fi egal cu numărul vacilor.Petru Asaftei, IaşiSoluţie. a) Notăm cu g, o, v numărul găinilor, oilor, respectiv vacilor. Se ştie că2g +4o +4v = 324, de unde g +2(o + v) =162, deci g trebuie să fienumăr par.b) Cum g + o + v este număr impar, iar g este par, rezultă că o + v este numărimpar, prin urmare o şi v au în mod necesar parităţi diferite.V.83. Să se demonstreze că 13 | abc dacă şi numai dacă 13 | 3 · ab − c.Otilia Nemeş, Ocna MureşSoluţie. Avem:13 | abc ⇔ 13 | 10 · ab + c ⇔ 13 | 13 · ab − ¡ 10 · ab + c ¢ ⇔ 13 | 3 · ab − c.V.84. Determinaţi cel mai mic şi cel mai mare număr natural de 90 de cifre,divizibile cu 90 şi având suma cifrelor 90.Carmen Daniela Tamaş, BârladSoluţie. Un număr cu suma cifrelor 90 este oricum divizibil cu 9; pentruafidivizibil şi cu 10, eltrebuiesăsetermineîn0. Celmaimarenumăr cu proprietateadorită vafi99| {z...9}00| {z...0}, iar cel mai mic 100...0| {z }899...9| {z }0.10 8078 9V.85. Fie a, b ∈ N; săsearatecădacă ultima cifră anumărului a 2 + b 2 este 9,atunci ultima cifră alui(a + b) 2 este tot 9. Reciprocaesteadevărată?Ioan Săcăleanu, HârlăuSoluţie. Ultima cifră aunuipătrat perfect poate fi 0, 1, 4, 5, 6 sau 9. DacăU ¡ a 2 + b 2¢ =9, obligatoriu U ¡ a 2¢ =0, U ¡ b 2¢ =9(sau invers) sau U ¡ a 2¢ =4,U ¡ b 2¢ =5(sau invers). În primul caz , vom avea U (a) =0, U (b) ∈ {3, 7} (sauinvers), deci U (a + b) ∈ {3, 7} şi atunci U³(a 2´+ b) =9. În al doilea caz, vom aveaU (a) ∈ {2, 8}, U (b) =5(sau invers), deci U (a + b) ∈ {3, 7} şi din nou obţinem căU³(a 2´+ b) =9.Reciproca este falsă; de exemplu, pentru a =2, b =1avem că (a + b) 2 =9,însăa 2 + b 2 =5.V.86. a) Să serezolveînnumerenaturaleecuaţia x 2 + y 2 = 625.b) Să searatecăecuaţia x 2 + y 2 = 2007 nu are soluţii în N 2 .Valerica Benţa, IaşiSoluţie. a) Scădem din 625, pe rând, fiecare pătrat perfect care nu-l depăşeşte;rezultatul este tot pătrat perfect în cazurile 625 − 49 = 576, 625 − 225 = 400, 625 −400 = 225 şi 625−576 = 49. Obţinem soluţiile (x, y) ∈ {(7, 24) ; (15, 20) ; (20, 15) ; (24, 7)}.b) Un pătrat perfect dă laîmpărţirea prin 4 fie restul 0, fierestul1, deci x 2 +y 2 poate fi M 4 , M 4 +1sau M 4 +2.Cum2007 = M 4 +3,ecuaţia dată nuaresoluţiiîn N 2 .V.87. Să searatecă 7 51 > 3 89 .146Nela Ciceu, Bacău


Soluţia 1 (a autoarei). Avem:7 51 = ¡ 7 3¢ 17= 343 17 > 342 17 =(9· 38) 17 > (9 · 36) 17 =3 68 · 2 34 ==3 68 · 2 2 · ¡2 8¢ 4> 368 · 3 · 256 4 > 3 69 · 243 4 =3 69 · ¡3 5¢ 4=3 89 .Soluţia 2 (M. Haivas). Inegalitatea se scrie echivalent¡74 ¢ 51 4> ¡ 3 7¢ 89 7⇔ (2401) 51 4> (2187) 89 7,ceea ce este evident adevărat deoarece 2401 > 2187, iar 51 4 > 897 .Clasa a VI-aVI.81. Ştiind că 13 | 2a +3b +4c +5d, arătaţi că 13 | 43a +45b +47c +49d şi13 | 46a +30b − 64c − 54d ( a, b, c, d ∈ N).Norbert-Traian Ioniţă, elev, IaşiSoluţie. Avem:13 | 2(2a +3b +4c +5d)+13(3a +3b +3c +3d) ⇒ 13 | 43a +45b +47c +49d;13 | 13 (4a +3b − 4c − 3d) − 3(2a +3b +4c +5d) ⇒ 13 | 46a +30b − 64c − 54d.VI.82. Fie A = 3 m · 5 n , m, n ∈ N. Notăm cu a, b, c numărul divizorilornumerelor A, 3A, respectiv5A. Ştiind că a şi b sunt direct proporţionale cu 3 şi 4,iar b şi c sunt invers proporţionale cu 15 şi 16, să se determine A.Mihai Haivas, IaşiSoluţie. Avem că a =(m +1)(n +1), b =(m +2)(n +1),iarc =(m +1)(n +2).Din a 3 = b obţinem că m =2,apoidin15b =16c deducem că n =3, prin urmare4A =3 2 · 5 3 = 1125.VI.83. Dacă p este număr prim, iar n ∈ N ∗ ,săsearatecă p 4n − 3 nu estepătrat perfect.Mirela Marin, IaşiNotă. Domnul Titu Zvonaru atrage atenţia asupra faptului că ipoteza că p estenumăr prim nu este importantă, fiind suficient să considerăm p ∈ N \{0, 1}; acestlucru a fost observat în redacţie în momentul selectării problemei spre publicare, dars-a preferat păstrarea enunţului dat de autor. Prezentăm mai jos soluţiile d-lui TituZvonaru.Soluţia 1. Urmărim ultima cifră anumărului p 4n − 3:U (p) 0 1 2 3 4 5 6 7 8 9U ¡ p 4¢ 0 1 6 1 6 5 6 1 6 1U ¡ p 4n¢ 0 1 6 1 6 5 6 1 6 1U ¡ p 4n − 3 ¢ 7 8 3 8 3 2 3 8 3 8Deducem astfel că p 4n − 3 nu poate fi pătrat perfect.Soluţia 2. Are loc dubla inegalitate ¡ p 2n − 1 ¢ 2 2,fapt adevărat pentru p ≥ 2. Astfel p 4n − 3 este strict cuprins între două pătrateperfecte consecutive, prin urmare nu poate fi pătrat perfect.147


VI.84. Pentru n ∈ N ∗ , definim A n = 1 + 11 + 111 + ···+11...1| {z }.Arătaţi că:a) 3| A n dacă şi numai dacă 3 - n − 1;nb) 10 n +1+ 9n10 < A 2n< 10 n +1+ 10nA n11 , ∀n ≥ 3. Temistocle Bîrsan, IaşiSoluţie. a) Restul împărţirii unui număr prin 3 este acelaşi cu restul împărţiriiprin 3 a sumei cifrelor sale. Rezultă că oricum am considera trei termeni consecutividin A n , ei vor fi de forma M 3 , M 3 +1şi M 3 +2(nu neapărat în această ordine)şiatunci suma lor se va divide cu 3.Dacă n =3k, grupând câte trei termenii lui A n , deducem că A. n 3. Dacă n =3k +1, atunci A n = M 3 +11...1| {z }= M 3 +1. Dacă n =3k +2, atunci A n =3k+1M 3 +11...1 = M 3 +(M 3 +1)+(M 3 +2)=M 3 şi de aici rezultă concluzia| {z }3k+1+11...1| {z }3k+2de la a).b) Observăm căA 2n =A n +11...1| {z }n+1³++ ···+11...1| {z }2n11 · 10 n +11...1| {z }n=A n (10 n +1)+n · 11 ...1´+ ···+| {z }n.³= A n +³11| {z...1}n10 n +11...1| {z }n´+·10 n +11...1| {z }nRezultă că A 2n=1+10 n 11 ...1+ n · şi rămâne să arătăm că 9A n A n 10∀n ≥ 3. Scrise dezvoltat, aceste inegalităţi revin la9 + 99 + 999 + ···+9...9| {z }n11 + 110 + 1100 + ···+110...0| {z }n−1´=< 10 + 100 + 1000 + ···+10...0| {z };n< 10 + 110 + 1110 + ···+11...1


să searatecă 4ABC este isoscel.Gheorghe Iurea, IaşiSoluţie. a) Folosind inegalitatea triunghiului în 4CDM,P ADM = AD + AM + DM < AD + AM +(DC + MC)== AD + DC +(AM + MC)=P ADC .b) Să presupunem prin absurd că AB 6= AC; să zicem căAB < AC şi fie M ∈ (AC) astfel încât AM = AB. Avemcă4ABD ≡ 4AMD (L.U.L), deci P ABD = P ADM


a) dacă a 3 − b 3 = a + b, atuncia 2 + b 2 > 1;b) dacă a 3 + b 3 = a − b, atuncia 2 + b 2 < 1.Ionel Nechifor, IaşiSoluţie. Mai întâi, săobservăm că nu putem avea a = b, altfel din ambele ipotezear rezulta că a = b =0.Avem:a) a3 − b 3a + b =1⇒ a3 + b 3a + b > 1 ⇒ a2 − ab + b 2 > 1 ⇒ a 2 + b 2 > 1;b) a3 + b 3a − b =1⇒ a3 − b 3a − b < 1 ⇒ a2 + ab + b 2 < 1 ⇒ a 2 + b 2 < 1.VII.83. Determinaţi numerele întregi a, b, c, d pentru care ac + bd =1,iarad + bc =2.Gheorghe Iurea, IaşiSoluţie. Scăzândmembrucumembrurelaţiile din ipoteză, deducem că ad + bc −ac−bd =1,deci(a − b)(d − c) =1, de unde a−b = d−c =1sau a−b = d−c = −1.În primul caz, substituind a = b +1şi d = c +1în prima dintre relaţiile iniţiale,avem succesiv:ac + bd =1⇔ c (b +1)+b (c +1)=1⇔ 2bc + b + c =1⇔⇔ 4bc +2b +2c +1=3⇔ (2b +1)(2c +1)=3⇔⇔ (b, c) ∈ {(1, 0) ; (0, 1) ; (−1, 2) ; (−2, −1)} .Obţinem soluţiile (a, b, c, d) ∈ {(−1, −2, −1, 0) ; (0, −1, −2, −1) ; (1, 0, 1, 2) ; (2, 1, 0, 1)}.Similar, în al doilea caz găsim soluţiile(a, b, c, d) ∈ {(−2, −1, 0, −1) ; (−1, 0, −1, −2) ; (0, 1, 2, 1) ; (1, 0, 1, 2)} .VII.84. Fie pătratul ABCD cu latura de lungime a, iarE, F , G puncte pelaturile [BC], [CD], respectiv [AB] astfel încât CE = a 4 , CF = a 3 ,iarBG = a 2 .Săse arate că drepteleAE, BF şi CG sunt concurente.Claudiu Ştefan Popa, IaşiSoluţie. Fie {P } = AC ∩ BF; dinasemănarea 4CFP ∼4ABP , deducem că CPPA = CFAB = 1 CP. Atunci3 PA · AGGB ·BEEC = 1 3 · 11 · 3 =1şi din reciproca teoremei lui Ceva urmează1concluzia.VII.85. Fie O intersecţia diagonalelor patrulaterului ABCD. Dacă A ABD =A ABC = A COD ,săsearatecă CDAB − ABCD =1.Doru Buzac, IaşiSoluţie. Notăm S 1 = A AOD , S 2 = A AOB , S = A DOC .Cum A ABD = A ABC , rezultă că ABCD este trapez cuAB k CD, prin urmare A BOC = S 1 . Din ipoteză vomavea că S = S 1 + S 2 . Notăm c = CD şi atunci, cumAB4OAB ∼ 4OCD, deducem că S = c 2 . Pe de altăparte,S 2150


µ 2S 1 OD · dist (A, BD)=S 2 OB · dist (A, BD) = c şi astfel S S1= , prin urmare S 1 = √ S · S 2 .AmS 2 S 2obţinut că S = √ S · S 2 + S 2 , deciSS 2=1+r SS 2,adică c 2 =1+c. Rezultăcăc =1+ 1 (c este tocmai numărul de aur), de unde concluzia problemei.cVII.86. Fie A un punct pe manta unei mese de biliard circulare cu raza de 1 m.ObilăpleacădinA şi ajunge înapoi în A lovind manta de cel puţin trei ori; reflexiabileisefaceconsiderândcăaceastaloveşte un perete plan tangent la cerc în punctulde contact. Să searatecăexistă o infinitate de traiectorii posibile şi să sedeterminetraiectoria de lungime minimă.Cristian Lazăr, IaşiSoluţie. Considerând un poligon regulat cu n laturi, n ≥ 4, înscris în cerculdat, vârfurile acestuia pot fi punctele de contact cu manta ale bilei într-o traiectoriedorită; poligonul poate fi chiar unul stelat! Lungimea minimă a traiectoriei se atingeîn cazul triunghiului echilateral înscris în cerc, al cărui perimetru va fi 3 √ 3m.VII.87. Otablă are forma unui dreptunghi 4 × 5, <strong>format</strong>din20 de pătrăţele1 × 1. Avem la dispoziţie două jetoane, fiecare putând acoperi câte un pătrăţel. Încâte moduri putem aşeza jetoanele pe tablă, astfel încât ele să nuseaflenicipeaceeaşi linie, nici pe aceeaşi coloană? Generalizare.Gabriel Popa, IaşiSoluţie. Numărăm întâi în câte moduri putem aşeza jetoanele pe tablă, în absenţarestricţiei din enunţ. Dacă jetoanele ar fi numerotate, ar exista 20·19 modalităţi20 · 19de aşezare a lor; cum nu contează ordinea,avem = 190 modalităţi de aşezare.2Dintre acestea, 10 conţin cele două jetoane pe prima linie, 10 pe a doua linie etc.,deci 40 de aşezări au jetoanele pe aceeaşi linie. Apoi, există 6 aşezări cu jetoanele peprima coloană etc.,deci6 · 5=30de aşezări cu jetoanele pe aceeaşi coloană.În final, avem 190 − 40 − 30 = 120 aşezări ce verifică enunţul.Generalizare. În cazul unei table m × n, avemmn (mn − 1)2modalităţi de aşezare.−mn (n − 1)2−mn (m − 1)2= mn2(m − 1) (n − 1)Clasa a VIII-aVIII.81. Considerăm fixate numerele a, b ∈ Z ∗ , m, n ∈ N ∗ , m 6= n şi fie funcţiaf : N → Z, f (x) =ax+b. Dacă f (1)+f (2)+···+f (m) =f (1)+f (2)+···+f (n),să se calculeze suma S = f (1) + f (2) + ···+ f (m + n).¡ Dan Nedeianu, Dr.Tr.Severinm 2 + m ¢ ¡a n 2 + n ¢ aSoluţie. Din relaţia din ipoteză deducem că+mb =+nb.22Trecem totul într-un membru şi simplificăm prin m−n 6= 0; rezultăcă a (m + n +1)+151


2b =0.Atunci(m + n)(m + n +1)af (1) + f (2) + ···+ f (m + n) = +(m + n) b =2(m + n)(−2b)= +(m + n) b =0.2VIII.82. Să searatecă |−3xy + x + y| ≤ 1, ∀x, y ∈ [0, 1].Ovidiu Pop, Satu MareSoluţie. Avem de arătat că −1 ≤−3xy + x + y ≤ 1, ∀x, y ∈ [0, 1]. Primainegalitate este echivalentă cu0 ≤ x (1 − y)+y (1 − x)+(1− xy), adevărată pentrux, y ∈ [0, 1] (toţi cei trei termeni sunt pozitivi). A doua inegalitate este echivalentăcu 0 ≤ (1 − x)(1− y)+2xy, dinnouadevărată pentrux, y ∈ [0, 1].VIII.83. Să searatecănuexistă x, y ∈ Z pentru care 147x 2 =1+4y − 3y 2 .Mihai Crăciun, i PaşcaniSoluţia 1. Scriem relaţia din enunţ subforma3h(7x) 2 + y 2 =4y +1. Oricepătrat perfect este sau M 4 ,sauM 4 +1;pentrux, y ∈ Z, paranteza pătrată esteM 4 ,M 4 +1 sau M 4 +2 şi atunci membrul stâng este M 4 , M 4 +3 sau M 4 +2. Cummembrul drept este M 4 +1,urmeazăconcluzia.Soluţia 2 (Ioan Stanciu, elev, Craiova). Dacă există x, y cu proprietăţilecerute, în mod necesar 1+4y − 3y 2 ≥ 0. Deducem că y ∈ {0, 1}, valori pentru carex nu este întreg.VIII.84. Laturile a, b, c ale unui triunghi verifică egalitatea 2 ¡ a 8 + b 8 + c 8¢ =¡a 4 + b 4 + c 4¢ 2.Săsearatecă triunghiul este dreptunghic.Corina Elena Vişan, CraiovaSoluţie. Avem succesiv:a 8 + b 8 + c 8 − 2a 4 b 4 − 2b 4 c 4 − 2c 4 a 4 =0⇔⇔ ¡ a 4 + b 4 − c 4¢ 2− 4a 4 b 4 =0⇔ ¡ a 4 + b 4 +2a 2 b 2 − c 4¢¡ a 4 + b 4 − 2a 2 b 2 − c 4¢ =0⇔³ ¡a⇔2 + b 2¢ 4´³ 2 ¡− c a 2 − b 2¢ 2− c4´=0⇔⇔ ¡ a 2 + b 2 + c 2¢¡ a 2 + b 2 − c 2¢¡ a 2 − b 2 + c 2¢¡ a 2 − b 2 − c 2¢ =0şi de aici concluzia.VIII.85. Dacă a, b, c sunt numere reale pozitive, să searatecăpqa2 + b 2 − ab + b 2 + c 2 − bc √ 3= p a 2 + c 2 ⇔ 1 √3a + c = 2 b .Liviu Smarandache, CraiovaSoluţie. Fie [OA], [OB], [OC] trei segmente de lungimia, b, respectiv c, astfelîncâtm(\AOB) =60 ◦ ,iarm(\BOC) =30 ◦ .Avem:pqa2 + b 2 − ab + b 2 + c 2 − bc √ 3= p a 2 + c 2 ⇔⇔ p a 2 + b 2 − 2ab cos 60 ◦ + p b 2 + c 2 − 2bc cos 30 ◦ == p a 2 + c 2 ⇔ AB + BC = AC ⇔ A, B, C coliniare ⇔152


⇔ab sin 60◦2+⇔ A AOB + A BOC = A AOC ⇔bc sin 30◦2= ac2 ⇔ ab√ 3+bc =2ac ⇔ 1 a + √3c = 2 b .VIII.86. Opiramidă hexagonală regulată VABCDEF are muchia bazei AB =4cm şi înălţimea VO =4 √ 2 cm. Fie M mijlocul lui VD, {P } = AD ∩ BF, iar{Q} = PM ∩ (VCF). Săsearatecă:a) dreptele VP şi DQ sunt concurente; b) DQ ⊥ (VBF).Gabriel Popa, IaşiSoluţie. a) Cum (VAD)∩(VCF)=VO, PM ⊂ (VAD),iar {Q} = PM ∩ (VCF), înseamnăcă Q ∈ VO. Astfeldreptele VP şi DQ sunt ambele incluse în planul (VAD),cu Q ∈ Int VPD şi de aici urmează concurenţa dorită.b) Observăm că BF ⊥ AD şi BF ⊥ VO (VO⊥ (ABC)),de unde BF ⊥ (VAD), prin urmare BF ⊥ DQ. Vom maiarăta că DQ ⊥ VP şiatuncivarezultacă DQ ⊥ (VBF).Avem că PO =2cm, deci VP = √ VO 2 + OP 2 =6cm, iarPD = PO + OD =6cm. Deducem că 4PDV este isoscel,iar mediana bazei PM va fi şi înălţime. Astfel, Q va fi ortocentru în 4VPD,prinurmare DQ ⊥ VP.VIII.87. Considerăm prisma triunghiulară regulată ABCA 0 B 0 C 0 şi cubulAMCNA 0 M 0 C 0 N 0 ,undeM este punct interior triunghiului ABC. Fie E, F , E 0 ,F 0 mijloacele muchiilor [AB], [BC], [A 0 B 0 ],respectiv[A 0 C 0 ].a) Aflaţi măsura unghiului dintre dreptele EF 0 şi E 0 F .b) Aflaţi măsura unghiului <strong>format</strong> de planele (MCC 0 ) şi (ECC 0 ).Claudiu Ştefan Popa, IaşiSoluţie. a) Fie l = AB, h = AA 0 ;cumAC = AM √ 2şi AM = AA 0 = h, deducem că l = h √ 2. În 4A 0 B 0 C 0 echilateral,avem că C 0 E 0 = l√ 32= h√ 62 ,iardin4CC0 F dreptunghicobţinem că C 0 F = h√ 62 .Seobservăuşor că EFC0 F 0este paralelogram, deci EF 0 k FC 0 , EF 0 = FC 0 . Rezultă căm( EF\0 ,E 0 F )=m( FC\0 ,E 0 F )=m( \C 0 FE 0 ). Din motive desimetrie, E 0 F = EF 0 = FC 0 = h√ 6şi astfel 4FC 0 E 0 este echilateral, prin urmare2măsura unghiului dorit este de 60 ◦ .b) Unghiul planelor este E\0 C 0 M 0 ,acărui măsură estem( A\0 C 0 M 0 )−m( A\0 C 0 E 0 )=45 ◦ − 30 ◦ =15 ◦ .Clasa a IX-aIX.81. Fie a, b ∈ R. Dacăecuaţia x 2 +ax+b+2 = 0 areambelerădăcini întregi,arătaţi că numărul 2a 2 + b 2 este natural compus.Dorin Mărghidanu, CorabiaSoluţie. Dacă x 1 ,x 2 ∈ Z sunt soluţiile ecuaţiei date, atunci a = − (x 1 + x 2 ) şi153


= x 1 x 2 − 2, de unde rezultă că a, b ∈ Z, deci 2a 2 + b 2 ∈ N. Înplus,2a 2 + b 2 =2(x 1 + x 2 ) 2 +(x 1 x 2 − 2) 2 = ¡ x 2 1 +2 ¢¡ x 2 2 +2 ¢şi concluzia se impune.Notă. Într-o formă puţin modificată, problema a apărut în RMT 2/2007, cunumărul IX.216, sub semnătura aceluiaşi autor.IX.82. Determinaţi funcţiile f : R → R pentru caref ¡ x 4 + y 3 + z 2 + t ¢ = f (x)+f ¡ y 2¢ + f ¡ z 3¢ + f ¡ t 4¢ , ∀x, y, z, t ∈ R.Lucian Tuţescu şi Liviu Smarandache, CraiovaSoluţie. Pentru x = y = z = t =0,găsim f (0) = 0. Dacă y = z =0, atuncif ¡ x 4 + t ¢ = f (x)+f ¡ t 4¢ , ∀x, t ∈ R. Pentru t = −x 4 ,găsim că f ¡ x 16¢ = −f (x),∀x ∈ R. Cum f ¡ x 4¢ = f (x), x ∈ R, deducem că f ¡ x 16¢ ³ ¡x4= f¢ 4´= f ¡ x 4¢ =f (x), ∀x ∈ R. Prin urmare f (x) =−f (x), ∀x ∈ R, adică f (x) =0, ∀x ∈ R.IX.83. Pentru a ≥ 9, să se demonstreze că are loc inegalitateaq3+ √ q3a +9≥ 1+ 1+ √ a.Marian Tetiva, BârladSoluţie. Pentru a =9avem egalitate; să arătăm că are loc strict inegalitateadin enunţ pentrua>9. Prin ridicare la pătrat şi cu notaţia a = x 2 , x>3, obţinemsuccesiv:1+ √ q3a +9> 2 1+ √ a + √ a ⇔ 1+ p 3x 2 +9> 2 √ 1+x + x ⇔⇔ p 3x 2 +9− (x +3)≥ 2 ¡√ 1+x − 2 ¢ 2x (x − 3) 2(x − 3)⇔ √ > √ ⇔3x2 +9+x +3 1+x +2x⇔ √3x2 +9+x +3 > 1√ ⇔ x ¡√ 1+x +1 ¢ > p 3x 2 +9+3.1+x +2Această dinurmă inegalitate rezultă adunând x>3 şi x √ 1+x> √ 3x 2 +9 (carerevine la √ x 3 + x 2 > √ 3x 2 +9, evident pentru x>3).IX.84. Fie ABC un triunghi. Determinaţi numerele întregi a, b, c nenule, primeîntre ele două câte două, astfel încât punctele M, N, P să fie coliniare, unde M, N,P sunt determinate prin condiţiile −−→ AM = a −→ −−→ −→ −→ −→AB; CN = bCA; CP = cBC.Ioan Săcăleanu, HârlăuSoluţie. Exprimând vectorii −−→ NP şi −−→ MN în funcţie de −→ −→AC şi BA, obţinem că−−→NP =(b + c) −→ AC + c −→ −−→ −→ −→BA şi MN =(1− b) AC − aBA. Deoarece punctele M, N, Psunt coliniare, rezultă că vectorii −−→ NP şi −−→ MN sunt coliniari; folosind relaţiile precedente,găsim condiţia ab + ac + bc = c. De aici rezultăcă c | ab şi cum c este primcu a şi b, deducem că c ∈ {−1, 1}. Pentru c =1găsim soluţiile a = −3, b = −2sau a = −2, b = −3. Pentru c = −1 găsim soluţiile a =1, b ∈ Z (în acest cazM = P = B) saua ∈ Z, b =1(în acest caz N = A şi P = B).IX.85. Fie ABC un triunghi ascuţitunghic şi D =pr BC A, E =pr CA B, F =pr AB C.Demonstraţi echivalenţa afirmaţiilor următoare:(i) 4ABC este isoscel;154


(ii) DB + EC + FA = DC + EA + FB;1(iii)DB + 1EC + 1FA = 1DC + 1EA + 1FB .Examinaţi cazurile în care 4ABC este obtuzunghic sau dreptunghic.Temistocle Bîrsan, IaşiSoluţie. Implicaţiile (i) ⇒ (ii) şi (i) ⇒ (iii) sunt triviale. Pentru implicaţiileinverse acestora, utilizăm relaţiile BD = c cos B, DC = b cos C etc. şi teoremacosinusului. Astfel, avem:(iii) ⇔ X 1c cos B = X 1b cos C ⇔ X 2ab 2 − c 2 − a 2 = X 2ac 2 − a 2 − b 2care, după transformări, este echivalentă cu(a − b)(b − c)(c − a)(a + b + c) 2 =0.Prin urmare, (iii) este echivalentă cu(i).Analog se arată că (ii) este echivalentă cu(a − b)(b − c)(c − a)(a + b + c) =0,deci cu (i).În cazul în care triunghiul este obtuzunghic şi A b este unghiul obtuz, avem echivalenţacondiţiilor:(j) 4ABC este isoscel de vârf A,(jj) DB + EC − FA = DC − EA + FB,1(jjj)DB + 1EC − 1FA = 1DC − 1EA + 1FB .Dacă 4ABC este dreptunghic în A, atunci condiţia (iii) nu se mai poate formula,iar (ii) devine DB + b = DC + c, careesteechivalentăcufaptulcă 4ABC estedreptunghic şi isoscel de vârf A.Clasa a X-aX.81. Să serezolveînR × R × R sistemulx − y 2/3 = z 1/3 ; x 4/3 − y = z 2/3 ; z 5/3 − y 4/3 = z.Vasile Chiriac, BacăuSoluţie. Notând x 1/3 = t, y 1/3 = u, z 1/3 = v, sistemul devine t 3 − u 2 = v;t 4 − u 3 = v 2 ; t 5 − u 4 = v 3 .Avemcă t 8 = ¡ u 3 + v 2¢ 2 ¡= u 2 + v ¢¡ u 4 + v 3¢ , de undegăsim că uv (u − v) 2 =0. Prin urmare, u =0sau v =0sau u = v. Analizândacestecazuri, gasim soluţiile (t, u, v) :(1, 0, 1); (−1, 0, −1); (0, 0, 0); (1, 1, 0); (0, −1, −1);³ √1+ 5, 1+√ 5, 1+√ 5´ ³ √1 − 5şi , 1 − √ 5, 1 − √ 5´. Corespunzător, obţinem2 2 22 2 2soluţiile (x, y, z) ale sistemului dat.X.82. Solve the equationae −x + b ¯¯e−x − 3¯¯ = ax 3 + b ¯¯x3 − 2¯¯ + a, a > b > 0.Zdravko Starc, Vršac, SerbiaSoluţie. Scriem ecuaţia sub formaa ¡ e −x − 2 ¢ + b ¯¯e−x ¡− 3¯¯ = a x 3 − 1 ¢ + b ¯¯x3 − 2¯¯ .Considerând funcţia f : R → R, f (t) =at + b |t − 1|, ecuaţia devine f (e −x − 2) =f ¡ x 3 − 1 ¢ . Se verifică uşor că f este strict crescătoare, deci injectivă. Ecuaţiei datăse reduce la e −x − 2=x 3 − 1, adică x 3 − e −x +1 = 0. Cum funcţia g : R → R,155


g (x) =x 3 − e −x +1 este strict crescătoare şi g (0) = 0, concluzionăm că ecuaţiax 3 − e −x +1 = 0 are soluţia unică x =0. Prin urmare, ecuaţia dată aresoluţia unicăx =0.X.83. În exteriorul triunghiului ABC se construiesc triunghiurile isoscele BMA,ANC şi CPB de baze AB, AC şi respectiv BC, astfel încât m( \MAB) = 15 ◦ ,m(\NAC)=45 ◦ ,iarm(\PBC)=30 ◦ .Săsearatecă m( \MPN)=60 ◦ .Angela Ţigăeru, SuceavaSoluţie. Vom nota afixul fiecărui punct cu litera micăce îi corespunde. Deoarece −−→ MA se obţine din −−→ <strong>MB</strong> în urma5πunei rotaţii în jurul lui M de unghi6 ,avemcă a − m =³(b−m) cos 5π 6 +i sin 5π ´, de unde m= 2a+b√ 3−bi62+ √ 3 − i .Analoggăsim afixele punctelor P şi N, anumep = 2b + c − c√ 3i3 − √ ,3irespectiv n = c − ai1 − iconcluzia problemei.. Prin calcule,m − pn − p = ¡√ 3 − 1 ¢ ³ cos π 3 + i sin π 3´şi de aiciX.84. Fie ABC un triunghi în care (tg B − 1) (tg C − 1) = 2. Dacă M şi Nsunt picioarele înălţimilor din B, respectivC, săsearatecăsegmenteleBM, CNşi MN se pot constitui în laturi ale unui triunghi.Cătălin Calistru, IaşiSoluţie. Problema este înrudită cu VI.30, publicată de acelaşi autor în RecMat1/2002. Ca şi acolo, cheia rezolvării este aceea de a arăta că m( A)=45 b ◦ ; atuncitriunghiurile ABM şi ACN vor fi dreptunghice isoscele, cu BM = AM şi CN = AN,deci segmentele din enunţ seconstituieînlaturiale4AMN.Deoarece tg A +tgB +tgC =tgA tg B tg C (identitate cunoscută), relaţia dinipotezăesteechivalentăcu1+tg B+tg C =tgB tg C,i.e. (tg A − 1) (tg B tg C − 1) =0. Se observăuşor că al doilea factor nu se poate anula, deci rămâne că tg A =1,adică m( A)=45 b ◦ .X.85. Se prelungeşte diametrul [MN] al unui cerc C cu segmentul [NP] congruentcu [MN]. Fie d perpendiculara în P pe MN şi R ∈ d, oarecare. Tangenteleduse prin R la C intersectează tangentaînM la C în S şi T .Săsearatecă centrulde greutate al 4RST este un punct fix.Adrian Reisner, ParisSoluţie. Raportăm planul la un reper ortogonal cu origineaîn M, având dreapta MN ca axă Ox şi tangenta în Mla C drept axă Oy. Fie a raza cercului C; atunci M (0, 0),2N (a, 0), P (2a, 0), iarR (2a, λ), cuλ variabil şi fie S (0,s).Ecuaţia dreptei RS este (λ − s) x − 2ay +2as = 0 şi,impunând condiţia că d (O, RS) = a 2 ,obţinem:¯ a2(λ − s)+2as¯¯q= a q(λ − s) 2 +4a 2 2 ⇔ |λ +3s| = (λ − s) 2 +4a 2 ⇔ 2s 2 +2λs − a 2 =0.156


Dacă T (0,t), seobţine pentru t ecuaţia 2t 2 +2λt−a 2 =0.Dinrelaţiile µ lui Viète, deducemcă t+s = −λ. Centrul de greutate al 4RST este G, λ + s + t 2a +0+0,µ 3 32ai.e. G3 , 0 , deci este un punct fix.Clasa a XI-a{x}XI.81. Dacă m ∈ Z, săsestudiezeexistenţa limitei limx→m sin πx .Dan Popescu, SuceavaSoluţie. Observăm că sin πx =(−1) [x] sin (πx − π [x]) = (−1) [x] sin π {x}, ∀x ∈{x}R. Atunci limx→mx>msin πx = lim π {x} 1x→mx>msin π {x} (−1) [x] π = (−1)m . Pentru calculul limiteiπ{x}la stânga, dacă m =2k, k ∈ Z, obţinem căsin πx = 1 = −∞, iardacă0−{x}sin πx = 10+m =2k +1, k ∈ Z, atunci limx→2kx


u 2 n +9. Deoarece (u n ) este convergent şi lim6u n =3, deducem că f (x) ≥ x lim u n,n→∞ n→∞deci f (x) ≥ 3x, ∀x ∈ [0, ∞).XI.84. Determinaţi numerele a ∈ R pentru care există o funcţie continuăf : R → R astfel încât (f ◦ f)(x) =a 2 f (x) − 2a 4 x, ∀x ∈ R.Andrei Nedelcu, IaşiSoluţie. Pentru a = 0 există f : R → R, f (x) = 0 care verifică enunţul.Vom arăta că nuexistă astfel de funcţii pentru a 6= 0. Presupunând contrariul, dinipoteză seobţine că f este injectivă, prin urmare f va fi strict monotonă. Rezultă căf ◦ f este strict crescătoare; din x 0, y (1) = 2 > 0, condiţia y ≥ 0 pentru x ∈ [0, 1] se158


scrie sub forma 1+x >a, ∀x ∈ (0, 1), decia ≤x − x2 suph (x), unde h :(0, 1) → R,x∈(0,1)h (x) = 1+xx − x 2 .Găsim cu uşurinţă a ≤ 3+2√ 2, prin urmare y = ax 2 +(1− a) x+1,a ∈ ¡ −∞, 3+2 √ 2 ¤ R\{0}. Cum aria cerută înenunţesteA = 1y (x) dx = 9 − a06 ,obţinem că A este minimă pentrua maxim, deci a =3+2 √ 2,iarA min = 3 − √ 2.3XII.82. Determinaţi primitivele funcţiei ¡ f :(1, ∞) → R,f (x) = x3 5x 4 +3 ¢ (ln x − 1)(x 4 − 1) 3 .Dan Nedeianu, Dr. Tr. SeverinSoluţie. Cu substituţia x 4 − 1=t, avemdecalculatZ1 (5t +8)(ln(t +1)− 4)16t 3 dt == 1 µ Z Z 5t +8 5t +8−416 t 3 dt +t 3 ln (t +1)dt .Calculând a doua integrală prinpărţi şi înlocuind t cu x 4 − 1, găsim căZx 4f (x) dx =(x 4 − 1) 2 − x8 +3x 44(x 4 − 1) 2 ln x + 1 16 ln ¡ x 4 − 1 ¢ + C.XII.83. Să se determine funcţiile continue f : R → R pentru careZ xf (x) =|x| + e −t f (x − t) dt, ∀x ∈ R.0Dumitru Mihalache, BârladSoluţie. Cu schimbarea de variabilă x − t = u, aducem ecuaţia funcţionalăRla forma f (x) =|x| + x Re −x+u f (u) du. Notând x e u f (u) du = F (x), deducem că0e x f (x) =F 0 (x), ∀x ∈ R şi ecuaţia devine e −x F 0 (x) −(e −x F (x) =|x|, ∀x ∈ R, sauîncă (e −x F (x)) 0 −= |x|, ∀x ∈ R. Considerând G (x) =x22 ,x


a 0 a 2 Y 2n−1 + ···+ a 2n0 a 2n+1 . Rezultă că y i ∈ Z şi y i | a 2n0 a 2n+1 , ∀i = 1, 2n +1.Cum a 0 a 2n+1 este impar, atunci a 2n0 a 2n+1 este impar, deci y i , i = 1, 2n +1 suntnumere impare. Din prima relaţie Viète, 2n+1 Py i = −a 1 , rezultă că a 1 este impar.Apoi, din P i


Soluţiile problemelor pentru pregătirea concursurilordin nr. 2/2007A. Nivel gimnazialG126. Să se determine numerele naturale care au proprietatea că media geometricăa tuturor divizorilor lor este un număr natural....,Petru Minuţ, Iaşin nSoluţie. Fie 1 = d 1 < d 2 < ··· < d k = n divizorii lui n; atunci , ,d 1 d 2nd keste şirul aceloraşi divizori, scris descrescător, prin urmare d 1 d 2 ···d k =n kd 1 d 2 ···d k, de unde (d 1 d 2 ···d k ) 2 = n k . Media geometrică a tuturor divizorilor luin este k√ d 1 d 2 ···d k = √ n şi este număr natural dacă şi numai dacă n este pătratperfect.G127. Dacă a, b, c, x, y, z, t sunt numere reale pozitive, să se demonstrezeinegalitatea1ax+by+cz + 1ay+bz+ct + 1az+bt+cx + 1at+bx+cy ≥ 8√ √3 a2 +b 2 +c 2p x 2 +y 2 +z 2 +t . 2D. M. Bătineţu-Giurgiu, BucureştiSoluţie. Observăm că produsul(a + b + c)(x + y + z + t) este, după desfacereaparantezelor, tocmai suma numitorilor din membrul stâng ai inegalităţii de demonstrat.Notăm acest membru stâng cu S; folosind inegalitatea dintre media aritmeticăşi cea armonică, obţinem16(a + b + c)(x + y + z + t) S ≥ 16 ⇔ S ≥(a + b + c)(x + y + z + t) .Însă a + b + cra 2 + b 2 + c 2≤şi x + y + z + trx 2 + y 2 + z 2 + t 2≤(inegalitatea3344dintre media aritmetică şi cea pătratică) şi atunci concluzia problemei urmează. Egalitatease atinge când a = b = c şi x = y = z = t.G128. Fie a, b, c numere reale pozitive astfel încât abc =1şi fie t ∈ [1, 5]. Săse arate căaa 2 + t + bb 2 + t + cc 2 + t ≤ 3t +1 .Titu Zvonaru, Comăneşti şi Bogdan Ioniţă, BucureştiSoluţie. Deoarece abc =1,există numerele reale pozitive x, y, z astfel încâta = x y , b = y z , c = z x .Avemcă⇒aa 2 + t =aa 2 + t +xyx 2 + ty ≤bb 2 + t +xy2xy +(t − 1) y 2 =x2x +(t − 1) y = x 22x 2 +(t − 1) xy ⇒cc 2 + t ≤ x 22x 2 +(t−1) xy + y 22y 2 +(t−1) yz + z 22z 2 +(t−1) zx ≤≤(x + y + z) 22(x 2 + y 2 + z 2 )+(t − 1) (xy + yz + zx) .161


Prin urmare, este suficient să demonstrăm că(x + y + z) 22(x 2 + y 2 + z 2 )+(t − 1) (xy + yz + zx) ≤ 3t +1 .După efectuarea calculelor, acesta se reduce laevident adevărată.(5 − t) ¡ x 2 + y 2 + z 2 − xy − yz − zx ¢ ≥ 0,Notă. Pentru t =2se obţine o problemă propusă la concursul Baltic Way 2005.½G129. Să se determine y ∈ R ∗ pentru care {x} + x + 1 ¾= {xy} + 1 , ∀x ∈ R.yy(Cu {·} am notat partea fracţionară.)Alexandru Negrescu, elev, BotoşaniSoluţie. Pentru x = 1 ½ ¾ ½ ¾ 1 2y obţinem că + = 1 ½ ¾ · ¸ 2 1y y y ,deci = . Cum½ ¾¸½ ¾ ¸y y2·12·1∈ [0, 1), iar ∈ Z, deducem că = =0, prin urmare 1 ∈ (0, 1) şiy yy yy2y ∈ N. Astfel,2 y ∈ (0, 2) ∩ N, deci 2 =1şi atunci y =2.y ½Pentru y =2, egalitatea din enunţ devine{x} + x + 1 ¾= {2x} + 1 22 , ∀x ∈R, iar·aceasta este adevărată întrucât revine la cunoscuta identitate a lui Hermite[x]+ x + 1 ¸=[2x], ∀x ∈ R.2¡ G130. Fie a, b, c lungimile laturilor unui triunghi ABC. Dacă a 2007 + b 2007 >2 2007 +1 ¢ c 2007 ,săsearatecăunghiul C b este ascuţit.Lucian Tuţescu, CraiovaSoluţie. Vom arăta că c este cea mai micălatură a triunghiului, de unde concluziaeste imediată. Să presupunem prin absurd că c ≥ b; atuncia 2007 + c 2007 ≥ a 2007 + b 2007 > ¡ 2 2007 +1 ¢ c 2007 ⇒ a 2007 > (2c) 2007 ⇒ a>2c,de unde a>2c ≥ b + c, ceea ce contrazice inegalitatea triunghiului.procedează dacă am presupune că c ≥ a.Analog seG131. Fie n, k ≥ 2 numere naturale şi mulţimea M = {− (n − 1) ,...,−2, −1, 1,2,...,n}. Săsearatecă M se poate partiţiona în k submulţimi având fiecare aceeaşisumă a elementelor dacă şi numai dacă n se divide cu k.Marian Tetiva, BârladSoluţie. Condiţia este necesară: dacă M admite o partiţie ca în enunţ, atuncisuma elementelor sale (care este n) va fi egală cusk (s fiind suma elementelor dinfiecare clasă apartiţiei).Pentru a demonstra suficienţa, vom construi efectiv o partiţie în cazul în care162


n = ks, cus ∈ N. Considerăm mulţimile:M 1 = {2, 3,...,s,s+1, −1, −2,...,− (s − 1) , −s} ;M 2 = {s +2,...,2s, 2s +1, − (s +1),...,− (2s − 1) , −2s} ;..M k−1 = {(k − 2) s +2,...,(k − 1) s, (k − 1) s +1, − ((k − 2) s +1),...,− (k − 1) s} ;M k = {1, (k − 1) s +2,...,ks,− ((k − 1) s +1),...,− (ks − 1)}şi este evident că M = M 1 ∪M 2 ∪···∪M k , M i ∩M j = ∅, ∀i 6= j, iar suma elementelororicărei mulţimi M i este s.G132. În fiecare câmp unitate al unei livezi m × n se află câteunmăr. Unnumăr de k arici pornesc, pe rând, din câmpul stânga-sus al livezii şi se mişcă sprecâmpul din dreapta-jos. La fiecare mişcare, un arici se poate deplasa cu un câmp,spre dreapta sau în jos, fără aieşi din livadă. Ariciul poate să culeagămărul dincâmpul pe care îl vizitează, dacă nu a fost cules deja de alt arici. Care este numărulminim k, pentru care k arici pot să culeagătoatemerele?Iurie Boreico, elev, ChişinăuSoluţie. Numerotăm câmpurile (x, y), cux ∈ {1, 2,...,m}; y ∈ {1, 2,...,n},începând din colţul stânga-sus. Fiecare mişcare a unui arici duce la mărirea coordonateix sau y acâmpuluipecareseaflăcu1, adică suma coordonatelor creştecu 1 la fiecare mişcare a unui arici. În particular, un arici poate să viziteze celmult un câmp de pe diagonala x + y = k. Cea mai lungă diagonală are lungimeamin (m, n) (diagonalele cu această lungimesuntx + y = m +1, x + y = m +2, ...,x + y = n +1 dacă, de exemplu, asumăm că m ≤ n), prin urmare avem nevoie de celpuţin min (m, n) arici care să culeagă toate merele de pe această diagonală; rezultăk ≥ min (m, n). Pedealtăparte,unnumăr de min (m, n) arici sunt suficienţi: încazul m ≤ n, putem considera că primul arici merge spre dreapta până lamarginealivezii, apoi coboară pânăladestinaţie; al doilea merge o unitate în jos, apoi spredreapta până la marginea livezii, după care coboară; al treilea merge două unităţi înjos ş.a.m.d.Răspunsul este deci k =min(m, n).G133. Fie 4ABC echilateral şi D un punct astfel încât BD = DC, m(\BDC) =30 ◦ ,iarBC separă A şi D. Dacă E ∈ (BD) cu m(\BAE) =15 ◦ ,săsearatecăCE ⊥ AC.Enache Pătraşcu, FocşaniNotă. A se vedea nota Oproblemă şi. . . nouă soluţii din acest număr al revistei,pag. 128.G134. Se consideră patrulaterul convex ABCD înscris într-un cerc de rază √ 6cm, având m( A)=60 b ◦ şi m( B)=45 b ◦ . Săsearatecă aria patrulaterului este celmult egală cu3 √ 6 cm 2 .Constantin Apostol, Rm. SăratSoluţie (Gabriel Popa). Cu teorema sinusurilor în 4ABC şi în 4ABD,obţinem că AC =2R sin 45 ◦ =2 √ 3 cm, respectiv BD =2R sin 60 ◦ =3 √ 2 cm.Dacă α = m( AC, \ BD), aria patrulaterului este S = 1 AC · BD · sin α şi este maximă2163


când α =90 ◦ . În cazul nostru, cum AC 2 + BD 2 > (2R) 2 (relaţia revine la 30 > 24),deducem că există un patrulater cu diagonalele perpendiculare şi de lungimi 2 √ 3 cmşi 3 √ 2 cm, prin urmare maximul ariei se atinge şi este S max = 1 2 · 2√ 3 · 3 √ 2=3 √ 6cm 2 .G135. Fie tetraedrul ABCD cu AB = CD, AC = BD, AD = BC. Săsearatecă celpuţin două dintre unghiurile diedre <strong>format</strong>e de faţa (ABC) cu feţele (BCD),(ACD), (ABD) sunt ascuţite.Dan Brânzei, IaşiSoluţie (Răzvan Ceucă, elev, Iaşi). Este evidentcă cele trei diedre nu pot fi toate neascuţite; săpresupunem prin absurd că două dintre ele sunt neascuţite(anume cele de muchii BC şi AC, alecăror măsurivor fi α, respectiv β, cuα, β ≥ 90 ◦ ), iar al treilea,anume cel de muchie AB, aremăsura γ


opusă (vezi,deexemplu,L.Niculescuşi V. Boskoff - Probleme practice de geometrie,Ed. Tehnică, 1990). Cum noi dorim să arătăm că diagonala AQ a paralelogramuluiASQT este simediană, ar fi destul să demonstrăm că ST este antiparalelă laBC,deci că ATAB = ASAB. Acest lucru este însă evident, deoarece AT =AC 2cosA , iarAS =AC şi astfel rezolvarea este încheiată.2cosAL127. Fie A 1 A 2 A 3 A 4 A 5 A 6 un hexagon inscriptibil. Să searatecăr A1 A 2 A 3+ r A4 A 5 A 6+ r A1 A 3 A 6+ r A3 A 4 A 6= r A3 A 4 A 5+ r A1 A 2 A 6+ r A2 A 3 A 6+ r A3 A 5 A 6,unde r XY Z este raza cercului înscris în 4XY Z.Cătălin Calistru, IaşiSoluţie. Dacă R este raza cercului circumscris hexagonului, este cunoscutărelaţiar XY Z =4R sin X 2 sin Y 2 sin Z ,undeXY Z este un triunghi având vârfurile comune2cu hexagonul. Vom demonstra întâi următoareaLemă. Dacă A 1 A 2 A 3 A 4 este un patrulater inscriptibil, atuncir A1 A 2 A 3+ r A1 A 3 A 4= r A1 A 2 A 4+ r A2 A 3 A 4.Într-adevăr, cu notaţiile din figură, vom avea:³r A1 A 2 A 3+ r A1 A 3 A 4=4R sin α 2 sin β 2 sin γ + δ2 ++sin γ 2 sin δ 2 sin α + β ´;2³r A1A 2A 4+ r A2A 3A 4=4R sin β 2 sin δ 2 sin α + γ +2+sin α 2 sin γ 2 sin β + δ ´2şi dezvoltând sin γ + δ =sin γ 2 2 cos δ 2 +sinδ 2 cos γ etc., obţinem concluzia lemei.2Aplicând lema patrulaterelor inscriptibile A 1 A 2 A 3 A 6 şi A 3 A 4 A 5 A 6 şi sumândmembru cu membru egalităţile obţinute, găsim tocmai relaţia de demonstrat.Notă. Aceeaşi soluţie a fost dată deVlad Emanuel, student, Bucureşti.L128. Să searatecă între medianele unui triunghi are loc inegalitatea³Y ´ hY8 ma´³Xm2a m 2 b ≥ (ma + m b )ih2 X m 2 am 2 b − X im 4 a .Dorel Băiţan şi I.V.Maftei, BucureştiSoluţie. Avem, folosind cunoscutele x 2 + y 2 + z 2 ≥ xy + xz + yz, sin A 2 ≤ ab + cşi 4R sin A 2 sin B 2 sin C = r, că2a 2 b 2 + b 2 c 2 + c 2 a 2 ≥ abc (a + b + c) ≥ (b + c)sin A 2 (c + a)sinB 2 (a + b)sinC 2 · 2p ==sin A 2 sin B 2 sin C · 2p · (a + b)(b + c)(c + a) =2= r 2SS(a + b)(b + c)(c + a) = (a + b)(b + c)(c + a) (1)4R r 2R165


Apoi, să observăm că are loc identitateaa 4 +b 4 +c 4 +(a + b + c)(−a + b + c)(a − b + c)(a + b − c) =2 ¡ a 2 b 2 + b 2 c 2 + c 2 a 2¢ ,de unde obţinem căa 4 + b 4 + c 4 +16S 2 =2 ¡ a 2 b 2 + b 2 c 2 + c 2 a 2¢ (2)Pentru triunghiul de laturi a, b, c, există un triunghi dual, având laturile m a , m b ,m c .Relaţia (1) aplicată în triunghiul dual ne dă că 2 ¡P m 2 am 2 b¢≥e Se RQ (ma + m b )şi putem scrie, folosind (2), căeS SeR =em a m b m c= 16 S e2= 2 ¡P ¢m 2 am 2 Pb − m4a.4Se 4m a m b m c 4m a m b m cCombinând aceste relaţii se obţine concluzia problemei.Notă. Principial aceeaşi soluţie a dat Marius Olteanu, inginer, Rm. Vâlcea.L129. În planul raportat la un reper cartezian xOy considerăm vectorii legaţiîn O: v 1 (a 1 ,b 1 ), v 2 (a 2 ,b 2 ), v 3 (a 3 ,b 3 ). SăsearatecăexistăuntetraedruOABCregulat, de muchie 1 şi astfel încât −→ −→ −→OA, OB, OC se proiectează peplanulxOy înv 1 , v 2 ,respectivv 3 dacă şi numai dacă se verifică simultanrelaţiile:3 ¡a22 1 + a 2 2 + a 2 ¢3 − a1 a 2 − a 1 a 3 − a 2 a 3 = 3 ¡b22 1 + b 2 2 + b 2 ¢3 − b1 b 2 − b 1 b 3 − b 2 b 3 =1;32 (a 1b 1 + a 2 b 2 + a 3 b 3 ) − (a 1 b 2 + a 2 b 1 + a 1 b 3 + a 3 b 1 + a 2 b 3 + a 3 b 2 )=0.Irina Mustaţă, studentă, BremenSoluţie. Completăm reperul din plan la unul în spaţiu Oxyz şi fie A (a 1 ,b 1 ,c 1 ),B (a 2 ,b 2 ,c 2 ), C (a 3 ,b 3 ,c 3 ) astfel încât OABC este tetraedru regulat de muchie 1.Din OA = OB = OC =1deducem că a 2 1 + b 2 1 + c 2 1 = a 2 2 + b 2 2 + c 2 2 = a 2 3 + b 2 3 + c 2 3 =1, iardinm(\AOB) =m(\BOC) =m( [COA) =60 ◦ rezultă, via produs scalar, căa 1 a 2 + b 1 b 2 + c 1 c 2 = a 1 a 3 + b 1 b 3 + c 1 c 3 = a 2 a 3 + b 2 b 3 + c 2 c 3 = 1 . Aceste egalităţi2pot fi scrise sub formă matriceală astfel:⎛⎝ a ⎞ ⎛1 b 1 c 1a 2 b 2 c 2⎠ ⎝ a ⎞ ⎛1 a 2 a 3b 1 b 2 b 2⎠ = ⎝ 1 ⎞1 12 21 121 ⎠2. (1)1 1a 3 b 2 c 3 c 1 c 2 c 3 2 21⎛Fie X = ⎝ a ⎞ ⎛1 b 1 c 1a 2 b 2 c 2⎠, iarA = ⎝ 1 ⎞⎛⎞1 132 21 1212⎠; vomaveaA −1 2− 1 2− 1 2= ⎝− 1 32 2− 1 2⎠,1 1a 3 b 2 c 3 2 21− 1 2− 1 32 2deciX ¡ X T A −1¢ = I 3 (2)Evident, de aici avem că X T A −1 X = I 3 şi, după efectuarea calculelor, se vor obţineexact cele trei condiţii din enunţul problemei.Să arătăm acum că acestecondiţii sunt suficiente, adică sădemonstrăm că putemgăsi c 1 , c 2 , c 3 care să dea restul condiţiilor din egalitatea (2). Ecuaţiile în care apar166


c 1 , c 2 , c 3 sunt:µ 3a1c 12 − a 22 − a µ3+ c 2 − a 122 + 3a 22 − a µ3+ c 3 − a 122 − a 22 + 3a 3=0;2µ 3b1c 12 − b 22 − b µ3+ c 2 − b 122 + 3b 22 − b µ3+ c 3 − b 122 − b 22 + 3b 3=0;23 ¡c22 1 + c 2 2 + c 2 ¢3 − c1 c 2 − c 1 c 3 − c 2 c 3 =1.Primele două sunt ecuaţii omogene cu 3 necunoscute, despre care se ştie că auovariabilăliberă, deci o soluţie netrivială ( ec 1 , ec 2 , ec 3 ). Cum a treia ecuaţieestesimetricăşi omogenă înc 1 , c 2 , c 3 , putem înmulţi ec 1 , ec 2 , ec 3 cu un factor k astfel încât egalitateasă fie îndeplinită şi rezolvarea problemei este încheiată.L130. Să searatecăpentruoricex, y ≥ 1 are loc inegalitatea³(xy − x − y) 2 + 6 √ ´3 − 10 xy ≥ 6 √ 3 − 9.Gabriel Dospinescu, Paris şi Marian Tetiva, BârladSoluţie. Luăm x = a +1, y = b +1,cua, b ≥ 0; inegalitatea de demonstratdevine³a 2 b 2 + 6 √ ´3 − 10 (a + b + ab) − 2ab ≥ 0.Cum a + b ≥ 2 √ ab şi 6 √ 3 − 10 > 0, ar fi suficient să arătăm că³a 2 b 2 + 6 √ ´³3 − 10 2 √ ´ab + ab − 2ab ≥ 0.Cu notaţia t = √ ab ≥ 0, amaveadejustificatcă³t 4 + 6 √ ´ ³3 − 10 t 2 +2 6 √ ´3 − 10 t ≥ 0 ⇔ f (t) ≥ 0,unde f :[0, ∞) → R, f (t) =t 3 + ¡ 6 √ 3 − 10 ¢ t +2 ¡ 6 √ 3 − 10 ¢ . Derivata acesteifuncţiei este f 0 (t) =3³t 2 − ¡√ 3 − 1 ¢ 2´,carearecasingurărădăcină pozitivă pe√3 − 1. Euşor de văzut că acesta este punct de minim pentru f pe intervalul [0, ∞),prin urmare f (t) ≥ f ¡√ 3 − 1 ¢ =0, ∀t ≥ 0, ceea ce încheie demonstraţia.Nota autorilor. De fapt, avem că f (t) = ¡ t − √ 3+1 ¢ 2 ¡ √ ¢t +2 3 − 2 , ceea ceconduce la concluzia dorită f (t) ≥ 0, ∀t ≥ 0, însă această descompunere este maigreu de văzut.Notă. Soluţii asemănătoare celei prezentate s-au primit de la Vlad Emanuel,student, Bucureşti, precum şi de la dl. Marius Olteanu, inginer, Rm. Vâlcea.L131. Să se afle valoarea minimă anumărului real k astfel încât, oricare ar fia, b, c reale pozitive cu a + b + c = ab + bc + ca, săaibă loc inegalitateaµ 1(a + b + c)a + b + 1b + c + 1 c + a − k ≤ k.Andrei Ciupan, elev, BucureştiSoluţii. Înµ particular, inegalitatea din enunţ trebuie să aibălocpentrua = b =3c =1;astfel,32 − k ≤ k ⇔ k ≥ 9 8 . Vom arăta că 9 este valoarea minimă căutată8167


aluik; pentru aceasta, ar trebui să arătăm căµ 1(a + b + c)a + b + 1b + c + 1 ≤ 9 (a + b + c +1),c + a 8oricare ar fi a, b, c ∈ R + cu a + b + c = ab + bc + ca. Observăm căa + b + c +1= (a + b + c)2 +(a + b + c)= (a + b + c)2 + ab + bc + ca=a + b + ca + b + c(a + b)(b + c)+(b + c)(c + a)+(c + a)(a + b)= ;a + b + c1 9(b + c)(c + a)astfel, ar fi suficient să demonstrăm că (a + b + c) ≤ , deoarecea + b 8(a + b + c)scriind încă două inegalităţi similare şi sumându-le, obţinem chiar ceea ce dorim.Această ultimă inegalitate se scrie succesiv:9(a + b)(b + c)(c + a) ≥ 8(a + b + c) 2 ⇔ 9(a + b)(b + c)(c + a) ≥≥ 8(a + b + c)(ab + bc + ca) ⇔ 9(a + b)(b + c)(c + a) ≥≥ 8(a + b)(b + c)(c + a)+8abc ⇔ (a + b)(b + c)(c + a) ≥ 8abc,fapt care rezultă din inegalitatea mediilor.Notă. Soluţie corectă adatdl.Marius Olteanu, inginer, Rm. Vâlcea.L132. Fie a, b, c, x, y, z ∈ R şi A = ax+by+cz, B = ay+bz+cx, C = az+bx+cy.Dacă |A − B|≥1, |B − C|≥1 şi |C − A|≥1, arătaţi că ¡ a 2 + b 2 + c 2¢¡ x 2 + y 2 + z 2¢ ≥ 4 3 .Adrian Zahariuc, elev, BacăuSoluţie. Deoarece distanţa pe axa reală între oricare două dintrenumereleA,B şi C este cel puţin 1, distanţa dintre cel mai mare şi cel mai mic dintre ele estecel puţin 2, deci cel puţin unul dintre ele se află ladistanţa de cel puţin 1 faţă deorigine. Putem presupune că |A| =max{|A| , |B| , |C|} ≥ 1. Folosind identitatea luiLagrange şi inegalitatea CBS, obţinem:¡a 2 + b 2 + c 2¢¡ x 2 + y 2 + z 2¢ =(ax + by + cz) 2 +(ay − bx) 2 +(bz − cy) 2 +(cx − az) 2 ≥≥ (ax + by + cz) 2 +(ay + bz + cx − bx − cy − az)23= |A| 2 +|B − C|23≥ 4 3 .L133. Determinaţi funcţiile f : N → N pentru care2f(n +3)f(n +2)=f(n +1)+f(n)+1, ∀n ∈ N.Gheorghe Iurea, IaşiSoluţie. Notăm a n = f (n), n ∈ N; atunci2a n+3 a n+2 = a n+1 + a n +1. Cum2a n+4 a n+3 = a n+2 +a n+1 +1, deducem că 2a n+3 (a n+4 − a n+2 )=a n+2 −a n , ∀n ∈ N.Prin urmare, 2a n+3 |a n+4 − a n+2 | = |a n+2 − a n |, ∀n ∈ N. Deoarece 2a n+3 a n+2 =a n+1 + a n +1≥ 1, rezultăcă a n 6=0, ∀n ≥ 2. Dacăexistă n 0 ∈ N cu a n0 +2 6= a n0 ,folosind relaţiile anterioare găsim că a n0 +4 6= a n0 +2, a n0 +6 6= a n0 +4, ...,şi atunci|a n0+2 − a n0 | > |a n0+4 − a n0+2| > |a n0+6 − a n0+4| > ···> 0,contradicţie. Prin urmare, a n+2 = a n , ∀n ∈ N. Notând a 1 = a 3 = a 5 = ···= a ∈ N,a 2 = a 4 = a 6 = ··· = b ∈ N, găsim 2ab = a + b +1, de unde a =1, b =2sau a =2,b =1,decif (n) =½ 1, n par2, n imparsau f (n) =168½ 2, n par1, n impar .


L134. Avem un colier cu n mărgele, numerotate consecutiv 1, 2,...,n, unden ≥ 3. În câte moduri putem să le colorăm cu trei culori, astfel încât oricare douămărgele consecutive să aibă culori diferite?Iurie Boreico, elev, ChişinăuSoluţie. Notăm cu a n numărul modalităţilor de colorare şi vom calcula a n recursiv.Evident că a 2 = a 3 =6. Fie n ≥ 4; putem alege culoarea mărgelei 1 în3 moduri, iar culorile mărgelelor 2, 3,...,n în câte două moduri, obţinând astfel3 · 2 n−1 modalităţi de colorare în care mărgelele 1 şi 2, 2 şi 3, ..., n − 1 şi n auculori diferite. Mai avem însă ocondiţie: ca mărgelele n şi 1 să aibăculoridiferite;atunci 3 · 2 n−1 = a n + b n ,undeb n este numărul colorărilor de tipul descris mai suspentru care mărgelele 1 şi n au aceeaşi culoare. Observăm că b n = a n−1 , suprimareamărgelei n dând o corespondenţă bijectivă întrenumărul colorărilor corespunzătoare,prin urmare a n−1 + a n =3· 2 n−1 .Avem că a n + a n+1 =3· 2 n şi, prin scădere, a n+1 − a n−1 =3· 2 n−1 . Deducem căa 2k+1 =3 ¡ 2 2k−1 +2 2k−3 + ···+2 3¢ + a 3 =6 ¡ a 2k−2 + ···+2 2 +1 ¢ =6 22k − 12 2 − 1 =2 2k+1 − 2. Cuma 2k + a 2k+1 =3· 2 2k ,vomaveacă a 2k =2 2k +2.Răspunsul poatefi scris sub forma a n =2 n +2· (−1) n .L135. Se consideră un poligon cu 3n laturi, n ≥ 2, înscris într-un cerc de rază1. Arătaţi că celmult3n 2 dintre segmentele având capetele în vârfurile poligonuluiau lungimea strict mai mare decât √ 2.Bianca-Teodora Iordache, elevă, CraiovaSoluţie. Evident că oricum am alege 4 puncte pe cercul de rază 1, existădouă printreacesteasituatelaodistanţă cel mult egală cu √ 2. Considerăm grafulG (X, U), undeX este mulţimea vârfurilor poligonului iniţial, iar două vârfurivorfi unite printr-o muchie dacă şi numai dacă distanţa dintre ele este strict mai maredecât √ 2. Conform observaţiei iniţiale, oricum am alege 4 vârfuri ale grafului, existădouă care nu sunt unite printr-o muchie, deci G nu conţine subgrafuri complete deordin 4. Aplicăm acum următorul rezultat:Teorema lui Turan. Dacă G =(X, U) este un graf neorientat cu n vârfuri cenu conţine subgrafuri complete de ordin p, iarr este restul împărţirii lui n la p − 1,atunci|U| ≤ p − 2p − 1 · n2 − r 2 r (r − 1)+ .2 2În cazul nostru avem 3n vârfuri, p =4, r =0, prin urmare |U| ≤ 3n 2 , deci celmult 3n 2 distanţe <strong>format</strong>e cu vârfurile poligonului iniţial sunt strict mai mari ca √ 2.Notă. Soluţie asemănătoare a dat Vlad Emanuel, student, Bucureşti.169


Clasele primareProbleme propuse 1P.154. Dorina are 15 baloane roşii şi albastre. Câte baloane roşii poate avea,dacă numărul acestora este mai mic decât numărul baloanelor albastre şi este celpuţin egal cu 3?(Clasa I )Inst. Maria Racu, IaşiP.155. Dintr-o carte lipsesc câteva pagini, de la numărul 71 la numărul 94. Câtefoi lipsesc din această carte?(Clasa I )Ionela Bărăgan, elevă, IaşiP.156. La concursul "Desene pe asfalt", elevii claselor I-IV de la Şcoala "OtiliaCazimir" au acumulat 50 de puncte şi cel puţin 2 premii din fiecare categorie. Careeste cel mare număr de premii pe care-l pot primi elevii, dacă pentru premiul I s-auacordat 10 puncte, pentru premiul al II-lea s-au acordat 6 puncte, iar pentru premiulal III-lea s-au acordat 4 puncte?(ClasaaII-a)Înv. Elena Porfir, IaşiP.157. Prin golirea unui singur vas, ales dintre cele de mai jos, putem face carestul vaselor să aibăcantităţi egale de lichid. Care vas trebuie golit?(ClasaaII-a)Amalia Cantemir, elevă, IaşiP.158. Aflaţi trei numere naturale ştiind că, adunându-le două câte două, obţinem100, 89, respectiv, 141.(Clasa a III-a)Inst. Maria Racu, IaşiP.159. Se consideră numerele: a =1+4+7+10+···+ 2008, b =2+5+6++ ···+ 2009, c =3+6+8+···+ 2010. Arătaţi că sumaa + b + c se împarte exactla 3, fără săcalculaţi această.(Clasa a III-a)Iuliana Moldovanu, elevă, IaşiP.160. Numărul a este de forma xy0, iarnumărul b este de forma uv. Săseaflea şi b ştiind că a + b =22zeci.(Clasa a III-a)Dragoş Toma,elev,IaşiP.161. Fie a şi b două numere naturale astfel încât diferenţa lor este de 5 ori maimică decâtsumalor.Săsearatecănumărul cel mai mare se împarte exact la 3, iarcel mai mic se împarte exact la 2.(ClasaaIV-a)Diana Tănăsoaie, elevă, IaşiP.162. Maria are 9 săculeţi cu monede. Cel puţinunsăculeţ cântăreşte unkilogram. În orice grupare de 5 săculeţi, cel puţin 3 săculeţi au aceeaşi masă, iar înorice grupare de 6 săculeţi, cel mult 5 săculeţi au aceeaşi masă. Care este cel maimare număr de săculeţi de un kilogram pe care îl poate avea Maria?(ClasaaIV-a)Petru Asaftei, Iaşi1 Se primesc soluţii până ladatade1iunie2009.170


P.163. Jumătatea produsului a două numere naturale consecutive împărţită cu3, nu poate da niciodată restul2.Recreaţii Ştiinţifice, Anul I (1883), nr. 4, pag. 119Clasa a V-aV.95. Două numere naturale se scriu în baza 10 folosind doar cifrele 1, 4, 6, şi 9.Poate fi unul dintre numere de 2008 ori mai mare decât celălalt?Cătălin Budeanu, IaşiV.96. Determinaţi k, n ∈ N ∗ astfel încât(1 + 1 · n)+(2+2· n)+···+(k + k · n) =3· 4 · 5 · 6.Petru Asaftei, IaşiV.97. Arătaţi că numărul N =17 n +21 n +25 n , n ∈ N, nu poate fi pătrat perfect.Virginia Grigorescu, CraiovaV.98. Fie n ∈ N ∗ .Să se demonstreze că numărul N = 5050 ...505 (2n +1 cifre)se scrie ca sumă a4n +2pătrate perfecte distincte.Veronica Plăeşu şi Dan Plăeşu, IaşiV.99. Se consideră numărul N = 1 + 11 + 101 + 1001 + ···+|100{z...01}.n cifrea) Pentru n ∈ N, n ≥ 5, arătaţi că 5 | N ⇔ 5 | n.b) Precizaţi care dintre propoziţiile "3 | n ⇒ 3 | N" şi "3 | N ⇒ 3 | n" esteadevărată pentruoricen ≥ 3.Temistocle Bîrsan, IaşiV.100. Determinaţi numerele naturale nenule a şi b pentru care există n ∈ Nastfel încât a 3n +2= şi 3a +2b


VI.98. Determinaţi cel mai mic număr natural n cu proprietatea că numărulzerourilor în care se termină numărul (n + 10)! este cu 2008 mai mare decât numărulzerourilor în care se termină n! (unde n! =1· 2 · 3 ···n).Cătălin Budeanu, IaşiVI.99. Un patrulater convex are două laturi opuse congruente şi diagonalelecongruente. Arătaţi că patrulaterul este trapez isoscel sau dreptunghi.Ioan Săcăleanu, HârlăuVI.100. Fie 4ABC cu m( A) b ≥ 90 ◦ . Săsearatecă m( B)=2m( b C) b dacă şinumai dacă există M ∈ [BC] astfel încât AB = AM = MC.Petru Asaftei, IaşiVI.101. Fie ABC un triunghi dreptunghic cu m( A)=90 b ◦ şi CD bisectoareaunghiului C, b D ∈ (AB). Perpendiculara din D pe bisectoarea unghiului B b intersecteazăipotenuzaBC în E. Dacă P este punctul de intersecţie a bisectoarelorunghiurilor triunghiului ABC, iarM este punctul de intersecţie dintre EP şi AC,arătaţi că \MPA ≡ \PBE.Nela Ciceu, Bacău şi Titu Zvonaru, ComăneştiClasa a VII-aVII.95. Fie ABCD pătrat, M un punct oarecare pe (AB), iarN ∈ (BC) esteastfel încât MN ⊥ MD.Arătaţi că AM · AB + CN · CB = DM 2 .Ovidiu Pop, Satu Mare şi Gh. Szöllösy, Sighetul MarmaţieiVII.96. Fie [AD] mediană în4ABC, M mijlocul lui [AD], {E} = BM ∩ AC,iar punctul F pe dreapta AB este astfel încât CF k AD. Demonstraţi că puncteleD, E şi F sunt coliniare.Mirela Marin, IaşiVII.97. Fie C 1 (O 1 ,r 1 ) şi C 2 (O 2 ,r 2 ), r 1


Clasa a VIII-aVIII.95. Pentru a, b, c ∈ R ∗ ,notăm α = a b + b c + c a , β = a c + c b + b a . Calculaţinumărul x = a3b 3 + b3c 3 + c3în funcţie α şi β.a3 Elena Nicu, Malu-Mare (Dolj)VIII.96. Rezolvaţi în numere naturale ecuaţia x 2 + y 2 + xy = x 2 y 2 .Mihail Bencze, BraşovVIII.97. Fie d 1 , d 2 , d 3 , d lungimile diagonalelor feţelor, respectiv diagonalei unuiparalelipiped dreptunghic. Dacă d 2 1 = 2d2 2d 2 3d 2 2 + ,săsearatecă paralelipipedul are od2 3muchie de lungime cel puţin egală cu d√ 33 . Gheorghe Molea, Curtea de ArgeşVIII.98. Fie VABCD piramidăpatrulaterăregulată. Notăm u=m( (VBC),(ABC)), \v = m( (VBC) \ , (VCD)) şi t = m( (VBC) \ , (VAD)). Arătaţi că u + v + t>180 ◦ .Claudiu Ştefan Popa, IaşiVIII.99. Pentru n ∈ N ∗ ,considerăm A = © 1 2 , 2 2 , 3 2 ,...,n 2ª . Determinaţi n,ştiind că existăofuncţie f : A → A astfel încât f (x) − f (y) = √ x − √ y, ∀x, y ∈ A.Cristian Lazăr, IaşiVIII.100. Rezolvaţi în în N 2 ecuaţia x 2 − 8 n + 1287 = 0.Mihai Crăciun, PaşcaniVIII.101. Se calculează suma cifrelor pentru fiecare dintre numerele de la 1 lan, n>10. Pentru fiecare sumă dintre cele n se calculează din nou suma cifrelor,repetându-se aceastăoperaţie pânăcândobţinem n numere <strong>format</strong>e din câte o singurăcifră. Să seaflen, ştiind că înmulţimea astfel obţinută cifrele1, 2, 3 şi 4 se repetăde câte 101 orifiecare,iarcifrele5, 6, 7, 8 şi 9 de câte 100 ori fiecare.Mihai Haivas, IaşiClasa a IX-aIX.91. Fie a, b, c, p ∈ R, p>0. Dacă ¯¯ax ¯2 + bx + c¯¯ ≤ p, ∀x ∈ [−1, 1], atunci¯cx 2 + bx + a¯¯ ≤ 2p, ∀x ∈ [−1, 1].Dorin Mărghidanu, CorabiaIX.92. Fie n ∈ N, n ≥ 3, iarα, β ∈ R astfel încât n + α + β 6= 0.Arătaţi căn−2(1 + α) ···(n + α)X− (1 + α) ···(n − 1+α)+ (−1) i+1 (1 + α) ···(n − i − 1+α)×n + α + β×β (β +1)···(β + i − 1) + (−1) n β ···(β + n − 2) = (−1) n β (β +1)···(β + n − 1).n + α + βGheorghe Costovici, IaşiIX.93. Fie 4ABC dreptunghic cu m( A)=90 b ◦ şi ABAC = 3 ,iarD mijlocul lui2[AC]. Notăm cu E punctul de intersecţie a cercurilor C 1 (A, AD) şi C 2 (B,BC), aflatde aceeaşi parte a dreptei AB ca şi punctul C. Determinaţi măsura unghiului [CAE.Cătălin Ţigăeru, Suceava173i=1


IX.94. În 4ABC, I este centrul cercului înscris, iar {M} = AI ∩ BC. Demonstraţică bisectoarea unghiului \AMC, BI şi AC sunt trei drepte concurente dacă şinumai dacă m( A) b = 120 ◦ .Vlad Emanuel, student şi Andrei Cozma, elev, BucureţiIX.95. Dacă x i ∈ [0,a], i = 1,n şi x n+1 = x 1 , demonstraţi cănXx i+1 (a − x i )


XI.93. Studiaţi convergenţa şirului (u n ) n≥1definit prin u 1 ≥ 0, u n+1 = u n +1u 2 n +1 ,∀n ∈ N ∗ .Gheorghe Costovici şi Adrian Corduneanu, IaşiXI.94. Să se demonstreze că pentru orice n ∈ N ∗ ,există numerele distincte³ 4´n.x 1 ,x 2 ,...,x n ∈ (1, 2), aşa încât x 1 x 2 ···x n =eDan Plăeşu, Iaşi¸n α − n , unde·³XI.95. Calculaţi lim 1+ 1 ´ ³n→∞ n α + 1+ 1 ´ 1 ³2 αn α + ···+ 1+ 1 ´ 1n αα ≥ 1 este fixat. (În legătură cu L83 din RecMat-1/2005.)Marius Olteanu, Rm. VâlceaClasa a XII-aXII.91. Prove that 1 R(1 + x) e (1+x)ex dx = e e − 1.0Zdravko Starc, Vr˘sac, SerbiaXII.92. Fie b>a>0, iarf :[a, b] → R o funcţie continuă pe[a, b] şi derivabilăRpe (a, b); săsearatecăexistă c ∈ (a, b) astfel încât b cf (x) dx = c (b − c) f (c).aDan Nedeianu, Dr. Tr. Severinsin x 2cos1XII.93. Demonstraţi că există c ∈ (2,π) pentru care dx ≤ .1 x cConstantin Micu, Melineşti (Dolj)XII.94. Calculaţi lim n 2n R x a + b√ dx, unde a ∈ (0, ∞) şi b ∈ R.n→∞n x2a+4+1Liviu Smarandache, CraiovaπR2XII.95. Fie (A, +, ·) un inel în care 0 6= 1şi 1+1+1+1+1=0. Săsearatecă, dacă x 3 y 2 = y 2 x 3 , ∀x, y ∈ A, atunci inelul este comutativ.I.V. Maftei, Bucureşti şi Mihai Haivas, IaşiSemnalăm cititorilor reeditarea colecţiei complete a revisteiRECREAŢII ŞTIINŢIFICE (1883-1888)la 125 de ani de la apariţia primului număr, cu respectarea formei în care a fostpublicată iniţial. <strong>Revista</strong> prezintă şi astăzi interes prin culoarea limbii române şiterminologiei folosite, prin conţinutul interesant şi de un înalt nivel ştiinţific, precumşi prin forma grafică frumoasă. Cei interesaţi pot consulta site-ul revisteihttp://www.recreatiistiintifice.rode unde se poate prelua gratuit.175


Probleme pentru pregătirea concursurilorA. Nivel gimnazialG146. Fie x, y, z ∈ (0, ∞) astfel încât xyz =1.Arătaţi căxy 3x 4 + y + z + yz 3y 4 + z + x + zx 3z 4 + x + y ≥ 1.Liviu Smarandache şi Lucian Tuţescu, CraiovaG147. Fie n ∈ N, n ≥ 2, fixat,iara, b, c sunt numere naturale astfel încâth n − 1ina +(n +1)b +2nc = n 2 +1.Arătaţi că n − ≤ a + b + c ≤ n.2Gheorghe Iurea, IaşiG148. Fie a 1 a 2 ...a p ∈ N. Săsearatecăoricenumăr natural are un multiplude forma a 1 a 2 ...a p a 1 a 2 ...a p ...a 1 a 2 ...a p 0 ...0.Marian Panţiruc, IaşiG149. a) Determinaţi două numere prime p, q astfel încât p


mijloacele arcului mic BC, respectiv arcului mare BC. SăsearatecădrepteleAM,DN şi BC sunt concurente.Gabriel Popa, IaşiB. Nivel licealL146. În plan se consideră drepteled 1 , d 2 ,..., d n+1 , oricare două neparalele.Notăm cu α k = m( d k\,d k+1 ), α k ≤ 90 ◦ , k = 1,n. Pe d 1 se consideră unsegmentde lungime 2 care se proiectează ped 2 ,apoisegmentulobţinut se proiectează ped 3 şi tot aşa, până cândped n+1 se obţine un segment de lungime 1. Ştiind cătg ¡ min © α i | i = 1,n ª¢ = p √ n4 − 1, determinaţi unghiurile α k , k = 1,n.Cristian Săvescu, student, BucureştiL147. Se consideră un poligon convex cu n laturi, n ≥ 4, având proprietatea căoricare două diagonale nu sunt paralele şi oricare trei nu sunt concurente în punctediferite de vârfurile poligonului. Se notează cun i numărul punctelor de intersecţie adiagonalelor interioare poligonului şi cu n e cel al punctelor de intersecţie exterioarepoligonului.a) Să searatecăexistă exact opt poligoane care verifică relaţia n i >n e .b) Să searatecăexistă exact trei poligoane pentru care n i + n e = kn 2 , k ∈ N ∗ .Mihai Haivas, IaşiL148. Pe latura (AB) a triunghiului ABC considerăm punctul D astfel încâtAB =4AD. De aceeaşi parte a laturii AB ca şi punctul C, luăm un punct Pastfel încât \PDA ≡ \ACB şi PB =2PD. Demonstraţi că patrulaterul ABCP esteinscriptibil.Nela Ciceu, Bacău şi Titu Zvonaru, ComăneştiL149. Să se determine poziţia punctului P pe directoarea parabolei P, astfelîncât aria triunghiului PT 1 T 2 săfieminimă, unde T 1 şi T 2 sunt punctele de contactcu parabola ale tangentelor duse din P la P.Adrian Corduneanu, IaşiL150. Fie tetraedrul A 1 A 2 A 3 A 4 ,iarP un punct în interiorul său. Notăm cuA ij ∈ (A i A j ) proiecţiile ortogonale ale lui P pe muchiile A i A j ale tetraedrului.Demonstraţi căV PA12 A 13 A 23+ V PA12 A 14 A 24+ V PA13 A 14 A 34+ V PA23 A 24 A 34≤ 1 4 V A 1 A 2 A 3 A 4.Când se atinge egalitatea?Marius Olteanu, Rm. Vâlceah L151. Să se demonstreze că nu există numere naturale n şi k astfel încât¡2+ √ ¢ 2n+1i h ¡4+ √ ¢ ki3 = 15 .Cosmin Manea şi Dragoş Petrică, PiteştiL152. Pentru a, b, c ∈ R şi x ∈ R + ,demonstraţi inegalitatea9a 2 + b 2 + c 2 ≤ 3(x +1) 2 (a + b + c) 4h3(x 2 +1)(a 2 + b 2 + c 2 )+2x(a + b + c) 2i ≤ 1 (ab + bc + ca) 2 a 2 + 1 b 2 + 1 c 2 .I. V. Maftei şi Dorel Băiţan, Bucureşti177


L153. Găsiţi toate funcţiile f : R → R cu proprietatea căf ¡ x 2 + xy + yf (y) ¢ = xf (x + y)+f 2 (y) , ∀x, y ∈ R.Adrian Zahariuc, student, PrincetonL154. Fie P ∈ R [X] un polinom de gradul n şi p : R → R funcţia polinomialăasociată. Ştiind că mulţimea {x ∈ R | p (x) =0} are k elemente (distincte), iarfuncţia f : R → R, f (x) =|p (x)| este derivabilă peR, h arătaţi că numărul maxim denirădăcini complexe nenule ale lui P este egal cu 2 − 2k.2Vlad Emanuel, student, BucureştiL155. Fie A, B ∈ M 2 (C) două matrice astfel încât matricea AB − BA să fieinversabilă. Să searatecă urma matricei (I 2 + AB)(AB − BA) −1 este egală cu1.Florina Cârlan şi Marian Tetiva, BârladTraining problems for mathematical contestsA. Junior highschool levelG146. Let x, y, z ∈ (0, ∞) such that xyz =1.Provethatxy 3x 4 + y + z + yz 3y 4 + z + x + zx 3z 4 + x + y ≥ 1.Liviu Smarandache and Lucian Tuţescu, CraiovaG147. Let n ∈ N, n ≥ 2 be a fixed number and let a, b, c be natural numbersh n − 1isuch that na +(n +1)b +2nc = n 2 +1. Show that n − ≤ a + b + c ≤ n.2Gheorghe Iurea, IaşiG148. Let a 1 a 2 ...a p ∈ N. Show that every natural number has a multiple ofthe form a 1 a 2 ...a p a 1 a 2 ...a p ...a 1 a 2 ...a p 0 ...0.Marian Panţiruc, IaşiG149. a) Determine two prime numbers p, q so that p


) Show that we cannot find a numbering for the upper basis so that i + a. i 9,∀i ∈ {1, 2,...,2008}.Gabriel Popa and Gheorghe Iurea, IaşiG152. In the isosceles triangle ABC (AB = AC), B 0 , C 0 denote the feet ofthe altitudes from B, respectively C. If AB =2B 0 C 0 , determine the angles of thetriangle.Nela Ciceu, BacăuandTituZvonaru,ComăneştiG153. In the triangle ABC, M is the midpoint of the side [BC], m(\ABC) =30 ◦and m(\ACB) = 105 ◦ . The perpendicular from C on AM cuts AB at Q. Calculatrethe value of the ratio QAQB .Neculai Roman, Mirceşti (Iaşi)G154. Let D be the midpoint of the side [BC] in the equilateral triangle ABCof side length 1, andletP beamovingpointon[CD]. Denote by M and N theprojections of the points B, respectively C on AP Find the area of the geometriclocus described by the segment [MN].Mariu Olteanu, Rm. VâlceaG155. Let C be the circumcircle of the acute-angled triangle 4ABC. DenotebyP the intersection point of the tangents to the circle at B and C, {D} = AP ∩ C,while M and N are the midpoints of the small arc BC, respectively of the big arcBC. Show that the straight lines AM, DN and BC meet at a point.Gabriel Popa, IaşiB. Highschool levelL146. The straight lines d 1 , d 2 ,..., d n+1 , are considered in the plane such thatany two lines are not parallel. We denote by α k = m( d k\,d k+1 ), α k ≤ 90 ◦ , k = 1,n.Asegmentoflength2 is cosidered on d 1 that is projected on d 2 , then the obtainedsegment is projected on d 3 and so on, until a segment of length 1 is obtained ond n+1 . Knowing that tan ¡ min © α i | i = 1,n ª¢ = p √ n4 − 1, determine the angles α k ,k = 1,n.Cristian Săvescu, student, BucureştiL147. A convex polygon with n sides, n ≥ 4, isconsideredsuchthatanypairofdiagonals are not parallel and and any three diagonals do not meet at other pointsexcept the vertices of the polygon. Let us denote by n i the number of intersectionpoints of the diagonals inside the polygon and by n e the number of intersection pointsof the diagonals outside the polygon.a) Show that exactly eight polygons exist such that the inequality n i >n e issatisfied.b) Show that exactly three polygons exist such that n i + n e kn 2 , k ∈ N ∗ .Mihai Haivas, IaşiL148. ApointD is considered on the side (AB) of the triangle ABC such thatAB =4AD. In the same halfplane as point C with respect to the side AB, wetakeapointP such that \PDA ≡ \ACB and PB =2PD. Prove that the quadrilateralABCP is inscriptible, that si it admits a circumscribed circle.Nela Ciceu, BacăuandTituZvonaru,Comăneşti179


L149. Determine the position of the point P on the directrix line of the parabolaP, so that the area of the triangle PT 1 T 2 be minimum, where T 1 and T 2 are thecontact points with P of the tangents drawn from P to P.Adrian Corduneanu, IaşiL150. Let us consider the tetrahedron A 1 A 2 A 3 A 4 , and a point P inside it.Denote by A ij ∈ (A i A j ) the orthogonal projections of P on the edge(s) A i A j of thetetrahedron. Prove thatV PA12A 13A 23+ V PA12A 14A 24+ V PA13A 14A 34+ V PA23A 24A 34≤ 1 4 V A 1A 2A 3A 4.When the equality is attained?Marius Olteanu, Rm. Vâlceah ¡2+ √ ¢ 2n+1iL151. Prove than no natural numbers n and k exist such that 3 =h ¡4+ √ ¢ ki15 .Cosmin Manea and Dragoş Petrică, PiteştiL152. For a, b, c ∈ R and x ∈ R + , prove the inequality9a 2 + b 2 + c 2 ≤ 3(x +1) 2 (a + b + c) 4h3(x 2 +1)(a 2 + b 2 + c 2 )+2x(a + b + c) 2i ≤ 1 (ab + bc + ca) 2 a 2 + 1 b 2 + 1 c 2 .I. V. Maftei and Dorel Băiţan, BucureştiL153. Find all functions f : R → R with the property thatf ¡ x 2 + xy + yf (y) ¢ = xf (x + y)+f 2 (y) , ∀x, y ∈ R.Adrian Zahariuc, student, PrincetonL154. Let P ∈ R [X] a polynomial of degree n and p : R → R its associatedpolynomial function. Knowing that the set {x ∈ 6 R | p (x) =0} consists of k (distinct)elements, and the function f : R → R,f(x) =| p (x)| is differentiable h on R, shownithat the maximum number of nonzero complex roots of P equals 2 − 2k.2Vlad Emanuel, student, BucureştiL155. Let A, B ∈ M 2 (C) be two matrices such that the matrix AB − BA isinvertible. Show that the trace of the matrix (I 2 + AB)(AB − BA) −1 is equal to 1.Florentina Cârlan and Marian Tetiva, Bârlad180


Concursul omagial "Recreaţii Ştiinţifice"Acest concurs este organizat cu prilejul împlinirii a 125 de ani de la apariţia revistei"Recreaţii Ştiinţifice" (1883-1888), prima revistă ştiinţifică (predominant matematică)din ţară adresată tineretului.Organizatorii concursului: Asociaţia "Recreaţii matematice".Premiile prevăzute de concurs:Premiul I — 200 lei (un premiu)Premiul II — 100 lei (două premii)Premiul III — 50 lei (trei premii)Participanţii la concurs: orice elev al şcolilor de orice nivel.Obligaţiile concurenţilor:— se cere rezolvarea celor cinci probleme enunţate mai jos, selectate din revisteleRecreaţii Ştiinţifice şi Recreaţii matematice;— elevii vor trimite soluţiile prin poştă (plic simplu timbrat) pe adresa:Asociaţia "Recreaţii matematice"str. Aurora, nr. 3, sc. D, ap. 6700474, Iaşicu menţiunea (pe plic): Concursul "Recreaţii Ştiinţifice";— data limită de participare este 1.02.2009;—concurenţii vor redacta îngrijit soluţiile problemelor trimise (câte o singurăproblemă pe foaie, cu enunţ, figură etc.).Acordarea premiilor se face pe baza punctajelor obţinute de concurenţi:— fiecare problemă estenotată maxim cu 10 puncte;—seacordă câte 2 puncte suplimentare pentru alte soluţii ale problemei, generalizărietc;— se depunctează soluţiile incomplete sau redactate neîngrijit;—concurenţii trebuie să obţină· minim 42 puncte, pentru premiul I,· minim 35 puncte, pentru premiul II,· minim 25 puncte, pentru premiul III.Sponsor: Fundaţia culturală "Poiana"(director,Dan Tiba).Premianţii concursului vor fi anunţaţi în nr. 1/2009 al revistei Recreaţii matematice(ce va apărea în martie 2009).Problemele concursului1. Ion şi Constantin merg la cumpărături cu soţiile lor, Maria şi Elena. Fiecaredin aceste patru persoane cumpără unnumăr de obiecte ce le plăteştepefiecarecuatâţia lei câte obiecte a cumpărat. Ion cumpără nouă obiecte mai mult decât Elenaşi fiecare soţ cheltuieşte cu 21 lei mai mult decât soţia sa. Care este soţia lui Ion şicare este a lui Constantin? Care este numărul de obiecte cumpărate de fiecare dintreaceste persoane? Care este suma cheltuită de fiecare dintre ele?181


2. Să se rezolve sistemul de ecuaţii(x +2y)(x +2z) =a,(y +2x)(y +2z) =b,(z +2x)(z +2y) =c (0


Pagina rezolvitorilorBRAŞOVColegiul Naţional de In<strong>format</strong>ică "Gr.Moisil".Clasa a IX-a (prof. POPOVICIFlorin). DARIE Flavius: VII.88, VIII(88-90), IX(88,89).CRAIOVAColegiul Naţional "Carol I". Clasa a VIII-a (prof. STANCA Monica). STAN-CIU Ioan: VII(81,82), VIII(84-86), G(126,129,130,133).IAŞIŞcoala nr. 11 "Otilia Cazimir". ClasaaIII-a(inst. HUZUM Lina). ANICĂILaura: P(144-148); HUZUM Andrei: P(144-148); MĂRIUŢA Simina: P(144-148);STOIAN Ioana: P(144-148).Şcoala nr. 14 "Gh. Mârzescu". ClasaaIII-a(inst. NUŢĂ Elena). BACIU Tudor:P(144-148); CHIRILUŢĂ George-Ştefan: P(144-148); POSTUDOR Georgiana-Mădălina: P(144-148); STOICA Adriana: P(144-148).Şcoala nr. 26 "G. Coşbuc". Clasa a III-a (inst. RACU Maria). APACHIŢEIAura Georgiana: P(144-149); BURA Emma-Andreea: P(144-149); FILIP Ingrid-Ştefania: P(144-149); GHEORGHIŢĂ Narcis-Eugen: P(144-149); HRISCU Ovidiu-Constantin: P(144-150); HUZA Mădălina: P(144-150); LEŞOVSCHI Alexandra-Ioana: P(144-149); LUPU Roxana-Elena: P(144-149); MARICIUC Dragoş-Claudiu:P(144-149); MAXIM Alexandra-Camelia: P(144-149); TUDOSE Ema-Alina: P(144-150); ŢUCĂ Cosmin: P(144-149); VASILE Bogdan-Andrei: P(144-149). Clasa a IIIa(înv. HRIMIUC Valeria). BRUMĂ Andrei-Alexandru: P(144-150); DU<strong>MB</strong>RAVĂBianca: P(144-150); HARAPCIUC Eduard-Gabriel: P(144-150); MANTALEA Alex-Adrian: P(144-150); OLARU Alexandra: P(144-150).Colegiul Naţional. Clasa a V-a (prof. POPA Gabriel). STOLERU GeorgianaIngrid: V(88-90,93,94).Colegiul Naţional "C. Negruzzi". ClasaaVII-a(prof. SAVA Radu). IONIŢĂNorbert Traian: VII(81-86), G126.SUCEAVAŞcoala cu clasele I-VIII, nr. 3. Clasa a III-a (înv. TABARCEA Silvestru).FECHET Ştefan: P(136-139,141,143); ClasaaIV-a(inst. NECHITA Daniela). FE-CHET Mircea: P(134-141,143).Premii acordate rezolvitorilorŞcoala nr. 14 "Gh. Mârzescu", IaşiBACIU Tudor (cl. a III-a): 2/2007(10pb), 1/2008(6pb), 2/2008(5pb),CHIRILUŢĂ George-Ştefan (cl. a III-a): 2/2007(10pb), 1/2008(6pb), 2/2008(5pb),STOICA Adriana (cl. a III-a): 2/2007(10pb), 1/2008(6pb), 2/2008(5pb).Şcoala nr. 26 "G. Coşbuc", IaşiGHEORGHIŢĂ Narcis-Eugen (cl. a III-a): 2/2007(5pb), 1/2008(6pb), 2/2008(5pb).183


<strong>Revista</strong> semestrială RECREAŢII MATEMATICE este editată deASOCIAŢIA “RECREAŢII MATEMATICE”. Apare la datele de 1 martie şi1 septembrie şi se adresează elevilor, profesorilor, studenţilor şi tuturor celorpasionaţi de matematica elementară.În atenţia tuturor colaboratorilorMaterialele trimise redacţiei spre publicare (note şi articole, chestiuni demetodică, probleme propuse etc.) trebuie prezentate îngrijit, clar şi concis; eletrebuie să prezinte interes pentru un cerc cât mai larg de cititori. Se recomandă catextele să nu depăşească patru pagini. Evident, ele trebuie să fie originale şi sănu fi apărut sau să fi fost trimise spre publicare altor reviste. Rugăm ca materialeletehnoredactate să fie însoţite de fişierele lor.Problemele destinate rubricilor: Probleme propuse şi Probleme pentrupregătirea concursurilor vor fi redactate pe foi separate cu enunţ şi demonstraţie/rezolvare(câte una pe fiecare foaie) şi vor fi însoţite de numele autorului, şcoalaşi localitatea unde lucrează/învaţă.Redacţia va decide asupra oportunităţii publicării materialelor primite.În atenţia elevilorNumele elevilor ce vor trimite redacţiei soluţii corecte la problemele dinrubricile de Probleme propuse şi Probleme pentru pregatirea concursurilorvor fi menţionate în Pagina rezolvitorilor. Se va ţine seama de regulile:1. Pot trimite soluţii la minimum cinci probleme propuse în numărulprezent şi cel anterior al revistei; pe o foaie va fi redactată soluţia unei singureprobleme.2. Elevii din clasele VI-XII au dreptul să trimită soluţii la problemelepropuse pentru clasa lor, pentru orice clasă mai mare, din două clase mai mici şiimediat anterioare. Elevii din clasa a V-a pot trimite soluţii la problemele propusepentru clasele a IV-a, a V-a şi orice clasă mai mare, iar elevii claselor I-IV pottrimite soluţii la problemele propuse pentru oricare din clasele primare şi orice clasămai mare. Orice elev poate trimite soluţii la problemele de concurs (tip G şi L).3. Vor fi menţionate următoarele date personale: numele şi prenumele,clasa, şcoala şi localitatea.4. Plicul cu probleme rezolvate se va trimite prin poştă (sau va fi adusdirect) la adresa Redacţiei:Prof. dr. Temistocle BîrsanStr. Aurora, nr. 3, sc. D, ap. 6,700 474, IaşiJud. IAŞIE-mail: t_birsan@yahoo.com


CUPRINSO sută de ani de la naşterea lui GHEORGHE GHEORGHIEV .....................................95ILIE POPA – 100 de ani de la naştere....................................................................................97Simpozion dedicat revistei "Recreaţii Ştiinţifice" (1883-1888)...........................................99ARTICOLE ŞI NOTEF. REICHER – Despre calendar........................................................................................... 109M. TETIVA – Câteva probleme de teoria numerelor a căror rezolvarese bazează pe identităţi ......................... 112C. ŢIGĂERU – O caracterizare a punctului Mathot ......................................................... 115D. POPESCU – Unsprezece pătrate perfecte ...................................................................... 117T. BÎRSAN – Cercuri semiînscrise şi puncte de tip Gergonne sau Nagel..................... 120F. POPOVICI – O rafinare a inegalităţii lui Jensen ......................................................... 122Gh. IUREA – Asupra unor inegalităţi geometrice ............................................................. 124T. ZVONARU – Metoda deligamentării şi rafinarea unor inegalităţi.............................. 125CHESTIUNI METODICEGh. IUREA, G. POPA – O problemă şi … nouă soluţii.................................................. 128CORESPONDENŢEA. REISNER – Sur les matrices magiques......................................................................... 131CONCURSURI ŞI EXAMENEConcursul de matematică "Al. Myller", ed. a VI-a, 2008.................................................. 134Concursul de matematică "Florica T. Câmpan", 2008 ....................................................... 136Concursul "Student pentru o zi", Suceava........................................................................... 140PROBLEME ŞI SOLUŢIISoluţiile problemelor propuse în nr. 2/2007........................................................................ 143Soluţiile problemelor pentru pregătirea concursurilor din nr. 2/2007 ............................... 161Probleme propuse................................................................................................................... 170Probleme pentru pregătirea concursurilor ............................................................................ 176Training problems for mathematical contests ..................................................................... 178Concursul omagial "Recreaţii Ştiinţifice" ....................................................................... 181Pagina rezolvitorilor ............................................................................................................ 183ISSN 1582 – 17656 lei

Hooray! Your file is uploaded and ready to be published.

Saved successfully!

Ooh no, something went wrong!